EMRCS Flashcards

1
Q

Which of the following nerves supplies the skin on the palmar aspect of the thumb?

	Ulnar
	Median
	Radial
	Musculocutaneous
	None of the above
A

Answer: B

The median nerve supplies cutaneous sensation to this region.

How well did you know this?
1
Not at all
2
3
4
5
Perfectly
2
Q

The superior aspect of the vagina drains to which of the following lymph node groups?

	Superficial inguinal nodes
	Para-aortic nodes
	Internal iliac nodes
	Meso rectal nodes
	Obturator nodes
A

Ans: C

The lymph vessels from the superior aspect of the vagina join the internal and external iliac nodes, those from the inferior aspect of the vagina drain to the superficial inguinal nodes.

How well did you know this?
1
Not at all
2
3
4
5
Perfectly
3
Q

Which of the following structures is not transmitted by the jugular foramen?

	Hypoglossal nerve
	Accessory nerve
	Internal jugular vein
	Inferior petrosal sinus
	Vagus nerve
A

Ans: A

Contents of the jugular foramen:

Anterior: inferior petrosal sinus
Intermediate: glossopharyngeal, vagus, and accessory nerves
Posterior: sigmoid sinus (becoming the internal jugular vein) and some meningeal branches from the occipital and ascending pharyngeal arteries

The jugular foramen may be divided into three compartments:
Anterior compartment transmits the inferior petrosal sinus
Middle compartment transmits cranial nerves IX, X and XI
Posterior compartment transmits the sigmoid sinus

How well did you know this?
1
Not at all
2
3
4
5
Perfectly
4
Q

A 19 year old female is admitted with suspected meningitis. The House Officer is due to perform a lumbar puncture. What is the most likely structure first encountered when the needle is inserted?

	Ligamentum flavum
	Denticulate ligament
	Dural sheath
	Pia Mater
	Supraspinous ligament
A

Ans: E

Lumbar punctures are performed to obtain cerebrospinal fluid. In adults, the procedure is best performed at the level of L3/L4 or L4/5 interspace. These regions are below the termination of the spinal cord at L1.

During the procedure the needle passes through:
The supraspinous ligament which connects the tips of spinous processes and the interspinous ligaments between adjacent borders of spinous processes
Then the needle passes through the ligamentum flavum, which may cause a give as it is penetrated
A second give represents penetration of the needle through the dura mater into the subarachnoid space. Clear CSF should be obtained at this point

How well did you know this?
1
Not at all
2
3
4
5
Perfectly
5
Q

A 45 year old motor cyclist sustains a tibial fracture and is noted to have anaesthesia of the web space between his first and second toes. Which of the nerves listed below is most likely to be compromised?

	Superficial peroneal nerve
	Deep peroneal nerve
	Sural nerve
	Long saphenous nerve
	Tibial nerve
A

Ans: B

The deep peroneal nerve lies in the anterior muscular compartment of the lower leg and can be compromised by compartment syndrome affecting this area. It provides cutaneous sensation to the first web space. The superficial peroneal nerve provides more lateral cutaneous innervation.

How well did you know this?
1
Not at all
2
3
4
5
Perfectly
6
Q

A 63 year old lady is undergoing an axillary sentinel lymph node biopsy as part of her breast cancer treatment. Which of the structures listed below is most likely to be encountered?

	Subclavian artery
	Intercostobrachial nerve
	Upper cord of the brachial plexus
	Lower cord of the brachial plexus
	Axillary nerve
A

Answer: B

This can be a challenging question. A particularly careless surgeon could encounter all of these. However, during a routine level 1 axillary exploration which is where the majority of sentinel nodes will be located, the nerves most commonly encountered are the intercostobrachial nerves.

How well did you know this?
1
Not at all
2
3
4
5
Perfectly
7
Q

A patient undergoes a femoral hernia repair and at operation the surgeon decides to enter the abdominal cavity to resect small bowel. She makes a transverse incision two thirds of the way between umbilicus and the symphysis pubis. Which of the structures listed below is least likely to be divided?

	Rectus abdominis
	External oblique aponeurosis
	Peritoneum
	Fascia transversalis
	Posterior lamina of the rectus sheath
A

Answer: E

An incision at this level lies below the arcuate line and the posterior wall of the rectus sheath is deficient at this level.

How well did you know this?
1
Not at all
2
3
4
5
Perfectly
8
Q

What is the lymphatic drainage of the ovaries?

	Internal iliac nodes
	Common iliac nodes
	Para-aortic nodes
	Para uterine nodes
	Inguinal nodes
A

Ans: C

The lymphatic drainage of the ovary follows the gonadal vessels and drainage is therefore to the para-aortic nodes.

How well did you know this?
1
Not at all
2
3
4
5
Perfectly
9
Q

A 35 year old farm labourer injures the posterior aspect of his hand with a mechanical scythe. He severs some of his extensor tendons in this injury. How many tunnels lie in the extensor retinaculum that transmit the tendons of the extensor muscles?

	One
	Three
	Four
	Five
	Six
A

Answer: E

There are six tunnels, each lined by its own synovial sheath.

How well did you know this?
1
Not at all
2
3
4
5
Perfectly
10
Q

A 23 year old man undergoes an orchidectomy. The right testicular vein is ligated; into which structure does it drain?

	Right renal vein
	Inferior vena cava
	Common iliac vein
	Internal iliac vein
	External iliac vein
A

Ans: B

The testicular venous drainage begins in the septa and these veins together with those of the tunica vasculosa converge on the posterior border of the testis as the pampiniform plexus. The pampiniform plexus drains to the testicular vein. The left testicular vein drains into the left renal vein. The right testicular vein drains into the inferior vena cava.

How well did you know this?
1
Not at all
2
3
4
5
Perfectly
11
Q

Which of the muscles listed below is not innervated by the median nerve?

	Flexor pollicis brevis
	Lateral two lumbricals
	Pronator teres
	Opponens pollicis
	Adductor pollicis
A

Ans: E

Adductor pollicis is innervated by the ulnar nerve.
Medial two lumbricals innervated by the ulnar nerve.

How well did you know this?
1
Not at all
2
3
4
5
Perfectly
12
Q

A 44 year old lady is undergoing an abdominal hysterectomy and the ureter is identified during the ligation of the uterine artery. At which site does it insert into the bladder?

	Posterior
	Apex
	Anterior
	Base
	Superior aspect of the lateral side
A

Ans: D

The ureters enter the bladder at the upper lateral aspect of the base of the bladder. They are about 5cm apart from each other in the empty bladder. Internally this aspect is contained within the bladder trigone.

How well did you know this?
1
Not at all
2
3
4
5
Perfectly
13
Q

A 23 year old man is involved in a fight outside a nightclub and sustains a laceration to his right arm. On examination, he has lost extension of the fingers in his right hand. Which of the nerves listed below is most likely to have been divided?

	Median
	Musculocutaneous
	Radial
	Ulnar
	Axillary
A

Ans: C

The radial nerve supplies the extensor muscle group.

How well did you know this?
1
Not at all
2
3
4
5
Perfectly
14
Q

What is the correct embryological origin of the stapes?

	First pharyngeal arch
	Second pharyngeal arch
	Third pharyngeal arch
	Fourth pharyngeal arch
	Fifth pharyngeal arch
A

Ans: B

Embryological origin stapes = 2nd pharyngeal arch

The incus is most likely to arise from the first arch.

How well did you know this?
1
Not at all
2
3
4
5
Perfectly
15
Q

A 72 year old man develops a foot drop after a revision total hip replacement. Which nerve is likely to have been affected?

	Sciatic
	Femoral
	Obturator
	Superior gluteal
	Inferior gluteal
A

Ans: A

Whilst many of these nerves can be injured in hip surgery, for a foot drop to develop in this context (i.e. revision THR), the sciatic nerve is the most likely.

How well did you know this?
1
Not at all
2
3
4
5
Perfectly
16
Q

An injured axillary artery is ligated between the thyrocervical trunk of the subclavian and subscapular artery. Subsequent collateral circulation is likely to result in reversal of blood flow in which of the vessels listed below?

	Circumflex scapular artery
	Transverse cervical artery
	Posterior intercostal arteries
	Suprascapular artery
	Profunda brachii artery
A

Ans: A

It’s an easy question really, we just made the wording difficult (on purpose). It is asking about the branches of the axillary artery and knowledge of the fact that there is an extensive collateral network around the shoulder joint. As a result, the occlusion of the proximal aspect of the circumflex humeral inflow (from the axillary artery) ceases and there is then retrograde flow through it from collaterals.

The circumflex scapular artery is a branch of the subscapular artery and normally supplies the muscle on the dorsal aspect of the scapula. In this instance, flow is reversed in the circumflex scapular and subscapular arteries forming a collateral circulation around the scapula.

How well did you know this?
1
Not at all
2
3
4
5
Perfectly
17
Q

A 20 year old lady presents with pain on the medial aspect of her thigh. Investigations show a large ovarian cyst. Compression of which of the nerves listed below is the most likely underlying cause?

	Sciatic
	Genitofemoral
	Obturator
	Ilioinguinal
	Femoral cutaneous
A

Ans: C

The cutaneous branch of the obturator nerve is frequently absent. However, the obturator nerve is a recognised contributor to innervation of the medial thigh and large pelvic tumours may compress this nerve with resultant pain radiating distally.

How well did you know this?
1
Not at all
2
3
4
5
Perfectly
18
Q

A 73 year old man presents with a tumour at the central aspect of the posterior third of the tongue. To which of the following lymph node groups is it most likely to metastasise?

	Submental
	Submandibular
	Ipsilateral deep cervical nodes
	Contralateral deep cervical nodes
	Bilateral deep cervical nodes
A

Ans: E

Posterior third tumours of the tongue commonly metastasise to the bilateral deep cervical lymph nodes

Tumours of the posterior third of the tongue will typically metastasise early and bilateral nodal involvement is well recognised, this is most often true of centrally located tumours and those adjacent to the midline as the lymph vessels may cross the median plane at this location.

How well did you know this?
1
Not at all
2
3
4
5
Perfectly
19
Q

A 6 month old child is brought to the surgical clinic because of non descended testes. What is the main structure that determines the descent path of the testicle?

	Processus vaginalis
	Cremaster
	Mesorchium
	Inguinal canal
	Gubernaculum
A

Ans: E

The gubernaculum is a ridge of mesenchymal tissue that connects the testis to the inferior aspect of the scrotum. Early in embryonic development the gubernaculum is long and the testis are located on the posterior abdominal wall. During foetal growth the body grows relative to the gubernaculum, with resultant descent of the testis.

How well did you know this?
1
Not at all
2
3
4
5
Perfectly
20
Q

A 21 year old man undergoes surgical removal of an impacted 3rd molar. Post operatively, he is noted to have anaesthesia on the anterolateral aspect of the tongue. What is the most likely explanation?

	Injury to the hypoglossal nerve
	Injury to the inferior alveolar nerve
	Injury to the lingual nerve
	Injury to the mandibular branch of the facial nerve
	Injury to the glossopharyngeal nerve
A

Ans: C

The lingual nerve is closely related to the third molar and up to 10% of patients undergoing surgical extraction of these teeth may subsequently develop a lingual neuropraxia. The result is anaesthesia of the ipsilateral anterior aspect of the tongue. The inferior alveolar nerve innervates the teeth themselves.

How well did you know this?
1
Not at all
2
3
4
5
Perfectly
21
Q

What is the most important structure involved in supporting the uterus?

	Round ligament
	Broad ligament
	Uterosacral ligaments
	Cardinal ligaments
	Central perineal tendon
A

Ans: E

The central perineal tendon provides the main structural support to the uterus. Damage to this structure is commonly associated with the development of pelvic organ prolapse, even when other structures are intact.

How well did you know this?
1
Not at all
2
3
4
5
Perfectly
22
Q

A 43 year old typist presents with pain at the dorsal aspect of the upper part of her forearm. She also complains of weakness when extending her fingers. On examination triceps and supinator are both functioning normally. There is weakness of most of the extensor muscles. However, there is no sensory deficit. Which of the following nerves has been affected?

	Anterior interosseous
	Median
	Posterior interosseous
	Palmar cutaneous
	Ulnar
A

Ans: C

The radial nerve may become entrapped in the arcade of Frohse which is a superficial part of the supinator muscle which overlies the posterior interosseous nerve. This nerve is entirely muscular and articular in its distribution. It passes postero-inferiorly and gives branches to extensor carpi radialis brevis and supinator. It enters supinator and curves around the lateral and posterior surfaces of the radius. On emerging from the supinator the posterior interosseous nerve lies between the superficial extensor muscles and the lowermost fibres of supinator. It then gives branches to the extensors.

How well did you know this?
1
Not at all
2
3
4
5
Perfectly
23
Q

A surgical resection specimen is analysed histologically. The pathologist comments that at the periphery of the resected specimen, oxyphil cells are identified. In which of the structures listed below are these cells typically found?

	Thymus
	Thyroid gland
	Parathyroid gland
	Lymph node
	Adrenal gland
A

Ans: C

Oxyphil cells are typically found in parathyroid glands

How well did you know this?
1
Not at all
2
3
4
5
Perfectly
24
Q

A 34 year old man is injured by farm machinery and sustains a laceration at the superolateral aspect of the popliteal fossa. The medial aspect of biceps femoris is lacerated. Which of the following underlying structures is at greatest risk of injury?

	Gracilis
	Sural nerve
	Nerve to semimembranosus
	Popliteal artery
	Common peroneal nerve
A

Ans: E

The common peroneal nerve lies under the medial aspect of biceps femoris and is therefore at greatest risk of injury. The tibial nerve may also be damaged in such an injury (but is not listed here). The sural nerve branches off more inferiorly.

How well did you know this?
1
Not at all
2
3
4
5
Perfectly
25
Q

What is the lymphatic drainage of the male spongy urethra?

	External iliac nodes
	Internal iliac nodes
	Para aortic nodes
	Deep inguinal nodes
	Meso rectal nodes
A

Ans: D

The lymphatic drainage of the spongy urethra and the glans penis is to the deep inguinal nodes. The prostatic and membranous urethra drains to the internal iliac nodes.

How well did you know this?
1
Not at all
2
3
4
5
Perfectly
26
Q

A 34 year old lady suffers from hyperparathyroidism. The right inferior parathyroid is identified as having an adenoma and is scheduled for resection. From which of the following embryological structures is it derived?

	Second pharyngeal pouch
	Third pharyngeal pouch
	Fourth pharyngeal pouch
	First pharyngeal pouch
	None of the above
A

Ans: B

The inferior parathyroid is a derivative of the third pharyngeal pouch. The superior parathyroid originates from the fourth pharyngeal pouch.

How well did you know this?
1
Not at all
2
3
4
5
Perfectly
27
Q

A 23 year old man falls and slips at a nightclub. A shard of glass penetrates the skin at the level of the medial epicondyle, which of the following sequelae is least likely to occur?

Atrophy of the first dorsal interosseous muscle
Difficulty in abduction of the the 2nd, 3rd, 4th and 5th fingers
Claw like appearance of the hand
Loss of sensation on the anterior aspect of the 5th finger
Partial denervation of flexor digitorum profundus
A

Ans: C

Injury to the ulnar nerve in the mid to distal forearm will typically produce a claw hand. This consists of flexion of the 4th and 5th interphalangeal joints and extension of the metacarpophalangeal joints. The effects are potentiated when flexor digitorum profundus is not affected, and the clawing is more pronounced.More proximally sited ulnar nerve lesions produce a milder clinical picture owing to the simultaneous paralysis of flexor digitorum profundus (ulnar half).

This is the ‘ulnar paradox’, due to the more proximal level of transection the hand will typically not have a claw like appearance that may be seen following a more distal injury. The first dorsal interosseous muscle will be affected as it is supplied by the ulnar nerve.

How well did you know this?
1
Not at all
2
3
4
5
Perfectly
28
Q

A 56 year old man is undergoing a superficial parotidectomy for a pleomorphic adenoma. During the dissection of the parotid, which of the following structures will be encountered lying most superficially?

	Facial nerve
	External carotid artery
	Occipital artery
	Maxillary artery
	Retromandibular vein
A

Ans: A

Most superficial structure on the parotid gland = facial nerve

The facial nerve is the most superficial structure in the parotid gland. Slightly deeper to this lies the retromandibular vein, with the arterial layer lying most deeply.

How well did you know this?
1
Not at all
2
3
4
5
Perfectly
29
Q

A 43 year old man is stabbed outside a nightclub. He suffers a transection of his median nerve just as it leaves the brachial plexus. Which of the following features is least likely to ensue?

	Ulnar deviation of the wrist
	Complete loss of wrist flexion
	Loss of pronation
	Loss of flexion at the thumb joint
	Inability to oppose the thumb
A

Ans: B

Loss of the median nerve will result in loss of function of the flexor muscles. However, flexor carpi ulnaris will still function and produce ulnar deviation and some residual wrist flexion. High median nerve lesions result in complete loss of flexion at the thumb joint.

How well did you know this?
1
Not at all
2
3
4
5
Perfectly
30
Q

A 78 year old man is due to undergo an endarterectomy of the internal carotid artery. Which of the following nervous structures are most at risk during the dissection?

	Recurrent laryngeal nerve
	Sympathetic chain
	Hypoglossal nerve
	Phrenic nerve
	Lingual nerve
A

Ans: C

Nerves at risk during a carotid endarterectomy:
Hypoglossal nerve
Greater auricular nerve
Superior laryngeal nerve

How well did you know this?
1
Not at all
2
3
4
5
Perfectly
31
Q

Which of the structures listed below articulates with the head of the radius superiorly?

	Capitulum
	Trochlea
	Lateral epicondyle
	Ulna
	Medial epicondyle
A

Ans: A

The head of the radius articulates with the capitulum of the humerus.

How well did you know this?
1
Not at all
2
3
4
5
Perfectly
32
Q

Which of the following fascial structures encases the apex of the lungs?

	Waldeyers fascia
	Sibsons fascia
	Pretracheal fascia
	Clavipectoral fascia
	None of the above
A

Ans: B

Sibson’s fascia overlies the apices of both lungs

The suprapleural fascia (Sibson’s fascia) runs from C7 to the first rib and overlies the apex of both lungs.It lies between the parietal pleura and the thoracic cage.

How well did you know this?
1
Not at all
2
3
4
5
Perfectly
33
Q

As regards the internal jugular vein, which of the following statements is untrue?

It lies within the carotid sheath
It is the continuation of the sigmoid sinus
The terminal part of the thoracic duct crosses anterior to it to insert into the right subclavian vein
The hypoglossal nerve is closely related to it as it passes near the atlas
The vagus nerve is closely related to it within the carotid sheath
A

Ans: C

Each jugular vein begins in the jugular foramen, where they are the continuation of the sigmoid sinus. They terminate at the medial end of the clavicle where they unite with the subclavian vein.

The vein lies within the carotid sheath throughout its course. Below the skull the internal carotid artery and last four cranial nerves are anteromedial to the vein. Thereafter it is in contact medially with the internal (then common) carotid artery. The vagus lies posteromedially.

How well did you know this?
1
Not at all
2
3
4
5
Perfectly
34
Q

A 28 year old man requires a urethral catheter to be inserted prior to undergoing a splenectomy. Where is the first site of resistance to be encountered on inserting the catheter?

	Bulbar urethra
	Membranous urethra
	Internal sphincter
	Prostatic urethra
	Bladder neck
A

Ans: B

The membranous urethra is the least distensible portion of the urethra. This is due to the fact that it is surrounded by the external sphincter.

How well did you know this?
1
Not at all
2
3
4
5
Perfectly
35
Q

At the level of the wrist joint, which of the statements below best describes the relationship of the ulnar artery to the ulnar nerve?

	It lies on its radial side
	It lies deep to it
	It lies superficial to it
	It lies on its ulnar side
	None of the above
A

Ans: A

In the middle of the forearm, the artery is overlapped by the flexor carpi ulnaris and on the flexor retinaculum it is covered by a superficial layer from that structure. In its distal two-thirds, flexor digitorum superficialis lies on its radial side, and the ulnar nerve is situated on its ulnar side.

How well did you know this?
1
Not at all
2
3
4
5
Perfectly
36
Q

A 24 year old man falls and sustains a fracture through his scaphoid bone. From which of the following areas does the scaphoid derive the majority of its blood supply?

	From its proximal medial border
	From its proximal lateral border
	From its proximal posterior surface
	From the proximal end
	From the distal end
A

Ans: E

The blood supply to the scaphoid enters from a small non articular surface near its distal end. Transverse fractures through the scaphoid therefore carry a risk of non union.

How well did you know this?
1
Not at all
2
3
4
5
Perfectly
37
Q

A 21 year old man has an inguinal hernia and is undergoing a surgical repair. As the surgeons approach the inguinal canal they expose the superficial inguinal ring. Which of the following forms the lateral edge of this structure?

	Inferior epigastric artery
	Conjoint tendon
	Rectus abdominis muscle
	External oblique aponeurosis
	Transversalis fascia
A

Ans: D

The external oblique aponeurosis forms the anterior wall of the inguinal canal and also the lateral edge of the superficial inguinal ring. The rectus abdominis lies posteromedially and the transversalis posterior to this.

How well did you know this?
1
Not at all
2
3
4
5
Perfectly
38
Q

A patient sustains damage to the median nerve during a carpal tunnel release. Which of the following muscles will be affected?

	Abductor digiti minimi
	Abductor pollicis brevis
	Adductor pollicis
	Palmaris brevis
	Flexor digiti minimi brevis
A

Ans: B

Of the muscles listed, only the abductor pollicis brevis is innervated by the median. In questions like this one, ensure you don’t become confused between adductor and abductor.

How well did you know this?
1
Not at all
2
3
4
5
Perfectly
39
Q

Which of the following cranial venous sinuses is unpaired?

	Transverse sinus
	Superior sagittal sinus
	Cavernous sinus
	Sigmoid sinus
	Inferior petrosal sinus
A

Ans: B

The superior sagittal sinus is unpaired

How well did you know this?
1
Not at all
2
3
4
5
Perfectly
40
Q

Which of the following laryngeal tumours will not typically metastasise to the cervical lymph nodes?

	Glottic
	Supraglottic
	Subglottic
	Transglottic
	Aryepiglottic fold
A

Ans: A

The vocal cords have no lymphatic drainage and therefore this region serves as a lymphatic watershed. The supraglottic part drains to the upper deep cervical nodes through vessels piercing the thyrohyoid membrane. The sub glottic part drains to the pre laryngeal, pre tracheal and inferior deep cervical nodes. The aryepiglottic and vestibular folds have a rich lymphatic drainage and will metastasise early.

How well did you know this?
1
Not at all
2
3
4
5
Perfectly
41
Q

Which of the following forms the medial wall of the femoral canal?

	Pectineal ligament
	Adductor longus
	Sartorius
	Lacunar ligament
	Inguinal ligament
A

Ans: D

Borders of the femoral canal
Laterally	Femoral vein
Medially	Lacunar ligament
Anteriorly	Inguinal ligament
Posteriorly	Pectineal ligament
How well did you know this?
1
Not at all
2
3
4
5
Perfectly
42
Q

Which of the structures listed below accompanies the aorta as it traverses the aortic hiatus?

	Oesophagus
	Thoracic duct
	Vagal trunks
	Right phrenic nerve
	Left phrenic nerve
A

Ans: B

The aorta is accompanied by the thoracic duct as it traverses the aortic hiatus. The vagal trunks accompany the oesophagus which passes through the muscular part of the diaphragm on the right. The right phrenic nerve accompanies the IVC as it passes through the caval opening. The left phrenic nerve passes through the muscular part of the diaphragm anterior to the central tendon on the left.

How well did you know this?
1
Not at all
2
3
4
5
Perfectly
43
Q

A 67 year old man is undergoing a transurethral resection of a bladder tumour using diathermy. Suddenly during the procedure the patient’s thigh begins to twitch. Stimulation of which of the following nerves is the most likely cause?

	Femoral
	Pudendal
	Sciatic
	Obturator
	Gluteal
A

Ans: D

The obturator nerve is most closely related to the bladder

How well did you know this?
1
Not at all
2
3
4
5
Perfectly
44
Q

A 5 year old boy is playing with some small ball bearings. Unfortunately, he inhales one. To which of the following lung regions is the ball most likely to settle?

	Right lower lobe
	Left main bronchus
	Right upper lobe
	Left lower lobe
	None of the above
A

Ans: A

As the most dependent part of the right lung a small object is most likely to lodge here. Most objects will preferentially enter the right lung owing to the angle the right main bronchus takes from the trachea.

How well did you know this?
1
Not at all
2
3
4
5
Perfectly
45
Q

A patient presents with superior vena caval obstruction. How many collateral circulations exist as alternative pathways of venous return?

	None
	One
	Two
	Three
	Four
A

Ans: E

There are 4 collateral venous systems:
Azygos venous system
Internal mammary venous pathway
Long thoracic venous system with connections to the femoral and vertebral veins (2 pathways)

Despite this, venous hypertension still occurs.

How well did you know this?
1
Not at all
2
3
4
5
Perfectly
46
Q

An 18 year old man is cutting some plants when a small piece of vegetable matter enters his eye. His eye becomes watery. Which of the following is responsible for relaying parasympathetic neuronal signals to the lacrimal apparatus?

	Pterygopalatine ganglion
	Otic ganglion
	Submandibular ganglion
	Ciliary ganglion
	None of the above
A

Ans: A

The parasympathetic fibres to the lacrimal apparatus transit via the pterygopalatine ganglion.

How well did you know this?
1
Not at all
2
3
4
5
Perfectly
47
Q

Which of the nerves listed below is directly responsible for the innervation of the lateral aspect of flexor digitorum profundus?

	Ulnar nerve
	Anterior interosseous nerve
	Radial nerve
	Median nerve
	Posterior interosseous nerve
A

Ans: B

The anterior interosseous nerve is a branch of the median nerve and is responsible for innervation of the lateral aspect of the flexor digitorum profundus.

How well did you know this?
1
Not at all
2
3
4
5
Perfectly
48
Q

A 45 year old lady is undergoing a Whipples procedure for carcinoma of the pancreatic head. The bile duct is transected. Which of the following vessels is mainly responsible for the blood supply to the bile duct remnant?

	Cystic artery
	Hepatic artery
	Portal vein
	Left gastric artery
	None of the above
A

Ans: B

The bile duct has an axial blood supply which is derived from the hepatic artery and from retroduodenal branches of the gastroduodenal artery. Unlike the liver there is no contribution by the portal vein to the blood supply of the bile duct. Damage to the hepatic artery during a difficult cholecystectomy is a recognised cause of bile duct strictures. In this scenario the distal vessels have been removed as the patient is undergoing a resection.

How well did you know this?
1
Not at all
2
3
4
5
Perfectly
49
Q

What vessel is the origin of the middle rectal artery?

	Aorta
	Inferior mesenteric artery
	Superior mesenteric artery
	Internal iliac artery
	Internal pudendal artery
A

Ans: D

The rectum is supplied by 3 main vessels
Superior rectal artery from inferior mesenteric artery
Middle rectal artery from the internal iliac artery
Inferior rectal artery from the internal pudendal artery

How well did you know this?
1
Not at all
2
3
4
5
Perfectly
50
Q

Which of the structures listed below are most closely related to the axillary nerve within the quadrangular space?

	Posterior circumflex humeral vessels
	Axillary artery
	Anterior circumflex humeral vessels
	Radial artery
	Acromiothoracic artery
A

Ans: A

The posterior circumflex humeral vessels which are branches of the axillary artery are related to the axillary nerve within the quadrangular space.

How well did you know this?
1
Not at all
2
3
4
5
Perfectly
51
Q

A 43 year old lady is undergoing a total thyroidectomy for an extremely large goitre. The surgeons decide that access may be improved by division of the infra hyoid strap muscles. At which of the following sites should they be divided?

	In their upper half
	In their lower half
	In the middle
	At their origin from the hyoid
	At the point of their insertion
A

Ans: A

Should the strap muscles require division during surgery they should be divided in their upper half. This is because their nerve supply from the ansa cervicalis enters in their lower half.

How well did you know this?
1
Not at all
2
3
4
5
Perfectly
52
Q

A 7 year old boy presents with right iliac fossa pain and there is a clinical suspicion that appendicitis is present. From which of the following embryological structures is the appendix derived?

	Vitello-intestinal duct
	Uranchus
	Foregut
	Hindgut
	Midgut
A

Ans: E

The appendix is derived from the midgut

How well did you know this?
1
Not at all
2
3
4
5
Perfectly
53
Q

A 22 year old women has recently undergone a surgical excision of the submandibular gland. She presents to the follow up clinic with a complaint of tongue weakness on the ipsilateral side to her surgery. Which nerve has been damaged?

	Hypoglossal nerve
	Lingual nerve
	Inferior alveolar nerve
	Facial nerve
	Lesser petrosal nerve
A

Ans: A

Three cranial nerves may be injured during submandibular gland excision.
Marginal mandibular branch of the facial nerve
Lingual nerve
Hypoglossal nerve

Hypoglossal nerve damage may result in paralysis of the ipsilateral aspect of the tongue. The nerve itself lies deep to the capsule surrounding the gland and should not be injured during an intracapsular dissection. The lingual nerve is probably at greater risk of injury. However, the effects of lingual nerve injury are sensory rather than motor.

How well did you know this?
1
Not at all
2
3
4
5
Perfectly
54
Q

You decide to take an arterial blood gas from the femoral artery. Where should the needle be inserted to gain the sample?

1-2 cm inferiorly to the mid point of the inguinal ligament
1-2cm inferiorly to the mid inguinal point
2cm inferomedially to the pubic tubercle
2cm superomedially to the pubic tubercle
3cm inferolaterally to the deep inguinal ring
A

Ans: B

The mid inguinal point is midway between the anterior superior iliac spine and the symphysis pubis

The mid inguinal point in the surface marking for the femoral artery.

How well did you know this?
1
Not at all
2
3
4
5
Perfectly
55
Q

A 67 year old man undergoes a carotid endarterectomy and seems to recover well following surgery. When he is reviewed on the ward post operatively he complains that his voice is hoarse. What is the most likely cause?

	Damage to the accessory nerve
	Damage to the cervical plexus
	Damage to the glossopharyngeal nerve
	Damage to the hypoglossal nerve
	Damage to the vagus
A

Ans: E

Many of these nerves are at risk of injury during carotid surgery. However, only damage to the vagus would account for a hoarse voice.

How well did you know this?
1
Not at all
2
3
4
5
Perfectly
56
Q

A 25 year old man has an inguinal hernia, which of the following structures must be divided (at open surgery) to gain access to the inguinal canal?

	Transversalis fascia
	External oblique aponeurosis
	Conjoint tendon
	Rectus abdominis
	Inferior epigastric artery
A

Ans: B

This question is asking what structure forms the anterior wall of the inguinal canal. The anterior wall is formed by the external oblique aponeurosis. Once this is divided the canal is entered, the cord can be mobilised and a hernia repair performed. The transversalis fascia and conjoint tendons form the posterior wall and would not routinely be divided to gain access to the inguinal canal itself.

How well did you know this?
1
Not at all
2
3
4
5
Perfectly
57
Q

Which muscle initiates abduction of the shoulder?

	Infraspinatus
	Latissimus dorsi
	Supraspinatus
	Deltoid
	Teres major
A

Ans: C

The intermediate portion of the deltoid muscle is the chief abductor of the humerus. However, it can only do this after the movement has been initiated by supraspinatus. Damage to the tendon of supraspinatus is a common form of rotator cuff disease.

How well did you know this?
1
Not at all
2
3
4
5
Perfectly
58
Q

Which of the following nerves is most commonly damaged during a superficial parotidectomy?

	Greater auricular
	Facial
	Greater occipital
	Accessory
	Opthalmic branch of the trigeminal
A

Ans: A

The greater auricular nerve and in particular its lobular branch is commonly injured in parotid surgery and consent usually makes particular reference to this. In a superficial parotidectomy, the facial nerve should not be injured and this is less common than a greater auricular nerve injury.

How well did you know this?
1
Not at all
2
3
4
5
Perfectly
59
Q

A 34 year old man is shot in the postero- inferior aspect of his thigh. Which of the following lies at the most lateral aspect of the popliteal fossa?

	Popliteal artery
	Popliteal vein
	Common peroneal nerve
	Tibial nerve
	Small saphenous vein
A

Ans: C

The contents of the popliteal fossa are (from medial to lateral):
Popliteal artery
Popliteal vein
Tibial nerve
Common peroneal nerve

The sural nerve is a branch of the tibial nerve and usually arises at the inferior aspect of the popliteal fossa. However, its anatomy is variable.

How well did you know this?
1
Not at all
2
3
4
5
Perfectly
60
Q

A 67 year old man has an abdominal aortic aneurysm which displaces the left renal vein. Which branch of the aorta is most likely to affected at this level?

	Inferior mesenteric artery
	Superior mesenteric artery
	Coeliac axis
	Testicular artery
	None of the above
A

Ans: B

The left renal vein lies behind of the SMA as it branches off the aorta. Whilst juxtarenal AAA may sometimes require the division of the left renal vein, direct involvement of the SMA may require a hybrid surgical bypass and subsequent endovascular occlusion.

How well did you know this?
1
Not at all
2
3
4
5
Perfectly
61
Q

An 21 year old man undergoes an uncomplicated tonsillectomy for recurrent attacks of tonsillitis. Post operatively he complains of otalgia. Which nerve is responsible?

	Trigeminal
	Hypoglossal
	Glossopharyngeal
	Facial
	Vagus
A

Ans: C

The glossopharyngeal nerve supplies this area and the ear and otalgia may be the result of referred pain.

How well did you know this?
1
Not at all
2
3
4
5
Perfectly
62
Q

A 12 year old boy undergoes surgery for recurrent mastoid infections. Post operatively he complains of an altered taste sensation. Which of the following nerves has been injured?

	Glossopharyngeal
	Greater petrosal
	Olfactory
	Trigeminal
	Chorda tympani
A

Ans: E

The chorda tympani branch of the facial nerve passes forwards through itrs canaliculus into the middle ear, and crosses the medial aspect of the tympanic membrane. It then passes antero-inferiorly in the infratemporal fossa. It distributes taste fibres to the anterior two thirds of the tongue.

How well did you know this?
1
Not at all
2
3
4
5
Perfectly
63
Q

What is the origin of the superior gluteal artery?

	Internal iliac artery
	External iliac artery
	Femoral artery
	Common iliac artery
	Circumflex femoral artery
A

Ans: A

The inferior gluteal artery arises from the anterior trunk of the internal iliac artery
The superior gluteal artery arises from the posterior trunk of the internal iliac artery

How well did you know this?
1
Not at all
2
3
4
5
Perfectly
64
Q

The first root of the brachial plexus commonly arises at which of the following levels?

	C6
	C5
	C3
	C2
	C8
A

Ans: B

It begins at C5 and has 5 roots. It ends with a total of 15 nerves of these 5 are the main nerves to the upper limb (axillary, radial, ulnar, musculocutaneous and median)

How well did you know this?
1
Not at all
2
3
4
5
Perfectly
65
Q

What is the anatomical level of the transpyloric plane?

	T11
	T12
	L1
	L4
	T10
A

Ans: C

How well did you know this?
1
Not at all
2
3
4
5
Perfectly
66
Q

Which of the nerves listed below is responsible for the innervation of gluteus maximus?

	Inferior gluteal nerve
	Superior gluteal nerve
	Posterior femoral cutaneous nerve
	Sciatic nerve
	Perineal nerve
A

Ans: A

Superior gluteal nerve
Arises from dorsal surface of the sacral plexus (L4, 5, S1)
Passes into gluteal region together with superior gluteal vessels
Supplies gluteus medius and minimus

Inferior gluteal nerve
Arises from dorsal surface of sacral plexus (L5, S1 and S2)
Runs medial to the posterior femoral cutaneous nerve
Enters gluteal region at inferior border of piriformis
Supplies gluteus maximus

How well did you know this?
1
Not at all
2
3
4
5
Perfectly
67
Q

When the brachial plexus is injured in the axilla as a result of a crutch palsy, which of the nerves listed is most commonly affected?

	Thoracodorsal nerve
	Suprascapular nerve
	Radial nerve
	Ulnar nerve
	Long thoracic nerve
A

Ans: C

The radial nerve is most commonly injured and results in a wrist drop. The ulnar nerve arises from the medial cord and is rarely affected as a result of this injury mechanism.

How well did you know this?
1
Not at all
2
3
4
5
Perfectly
68
Q

A 35 year old man falls and sustains a fracture to the medial third of his clavicle. Which vessel is at greatest risk of injury?

	Subclavian vein
	Subclavian artery
	External carotid artery
	Internal carotid artery
	Vertebral artery
A

Ans: A

The subclavian vein lies behind subclavius and the medial part of the clavicle. It rests on the first rib, below and in front of the third part of the subclavian artery, and then on scalenus anterior which separates it from the second part of the artery (posteriorly).

How well did you know this?
1
Not at all
2
3
4
5
Perfectly
69
Q

The pudendal canal is a fascial canal located on the lateral wall of the ischioanal fossa. In this location, it lies on the inferior border of which of the following muscles?

	Coccygeus
	Obturator internus
	Pubococcygeus
	Iliococcygeus
	Piriformis
A

Ans: B

The coccygeus, pubococcygeus and iliococcygeus form part of the pelvic diaphragm and are not related to it. The piriformis exits the pelvis via the greater sciatic foramen and is not associated with the canal in the ischiorectal fossa.

How well did you know this?
1
Not at all
2
3
4
5
Perfectly
70
Q

Where are the greatest proportion of musculi pectinati found?

	Right ventricle
	Left ventricle
	Right atrium
	Pulmonary valve
	Aortic valve
A

Ans: C

The musculi pectinati are found in the atria, hence the reason that the atrial walls in the right atrium are irregular anteriorly.
The musculi pectinati of the atria are internal muscular ridges on the anterolateral surface of the chambers and they are only present in the area derived from the embryological true atrium.

How well did you know this?
1
Not at all
2
3
4
5
Perfectly
71
Q

What is the lymphatic drainage of the upper ureter?

	Common iliac nodes
	Para aortic nodes
	External iliac nodes
	Internal iliac nodes
	Meso colic nodes
A

Ans: B

The upper ureter drains to the para-aortic nodes, the lower ureter drains to the common iliac nodes.

How well did you know this?
1
Not at all
2
3
4
5
Perfectly
72
Q

Which of the nerves listed below provides sensory innervation to the skin overlying the lateral aspect of the nose?

	Infratrochlear nerve
	Zygomatic nerve
	Nasopalatine nerve
	Lateral nasal branches of the ethmoidal nerve
	Frontal nerve
A

Ans: D

The lateral aspect of the external nose is innervated by lateral nasal branches of the anterior ethmoidal nerve. The ethmoidal nerve is a branch of the nasociliary nerve which is one of the divisions of the trigeminal.

How well did you know this?
1
Not at all
2
3
4
5
Perfectly
73
Q

Which of the following statements relating to the right phrenic nerve is false?

It lies deep to the prevertebral layer of deep cervical fascia
Crosses posterior to the 2nd part of the subclavian artery
It runs on the anterior surface of the scalene muscle
On the right side it leaves the mediastinum via the vena cava hiatus at a level of T8
The right phrenic nerve passes over the right atrium
A

Ans: B

Path
The phrenic nerve passes with the internal jugular vein across scalenus anterior. It passes deep to prevertebral fascia of deep cervical fascia.
Left: crosses anterior to the 1st part of the subclavian artery.
Right: Anterior to scalenus anterior and crosses anterior to the 2nd part of the subclavian artery.
On both sides, the phrenic nerve runs posterior to the subclavian vein and posterior to the internal thoracic artery as it enters the thorax.

Right phrenic nerve
In the superior mediastinum: anterior to right vagus and laterally to superior vena cava
Middle mediastinum: right of pericardium
It passes over the right atrium to exit the diaphragm at T8

Left phrenic nerve
Passes lateral to the left subclavian artery, aortic arch and left ventricle
Passes anterior to the root of the lung
Pierces the diaphragm alone

How well did you know this?
1
Not at all
2
3
4
5
Perfectly
74
Q

Which of the following structures separates the subclavian artery and vein?

	Digastric muscle
	Prevertebral fascia
	Anterior scalene muscle
	Middle scalene muscle
	Omohyoid
A

Ans: C

The anterior scalene muscle is an important anatomical landmark and separates the subclavian vein (anterior) from the subclavian artery (posterior).

How well did you know this?
1
Not at all
2
3
4
5
Perfectly
75
Q

A 33 year old man is stabbed in the right chest and undergoes a thoracotomy. The right lung is mobilised and the pleural reflection at the lung hilum is opened. Which of the structures listed below does not lie within this region?

	Pulmonary artery
	Azygos vein
	Pulmonary vein
	Bronchus
	None of the above
A

Ans: B

The pleural reflections encase the hilum of the lung and continue inferiorly as the pulmonary ligament. It encases the pulmonary vessels and bronchus. The azygos vein is not contained within it.

How well did you know this?
1
Not at all
2
3
4
5
Perfectly
76
Q

A 56 year old man requires long term parenteral nutrition and the decision is made to insert a PICC line for long term venous access. This is inserted into the basilic vein at the region of the elbow. As the catheter is advanced, into which venous structure is the tip of the catheter most likely to pass from the basilic vein?

	Subclavian vein
	Axillary vein
	Posterior circumflex humeral vein
	Cephalic vein
	Superior vena cava
A

Ans: B

The basilic vein drains into the axillary vein and although PICC lines may end up in a variety of fascinating locations the axillary vein is usually the commonest site following from the basilic. The posterior circumflex humeral vein is encountered prior to the axillary vein. However, a PICC line is unlikely to enter this structure because of its angle of entry into the basilic vein.

How well did you know this?
1
Not at all
2
3
4
5
Perfectly
77
Q

An individual is noted to have a left sided superior vena cava. By which pathway is blood from this system most likely to enter the heart?

Via the coronary sinus
Via the azygos venous system and into the superior vena cava
Via anomalies in the pumonary vascular bed
Via the left atrium and persistent foramen ovale
Directly into the roof of the right atrium
A

Ans: A

Persistent left superior vena cava is the most common anomaly of the thoracic venous system. It is prevalent in 0.3% of the population and is a benign entity of failed involution during embryogenesis.

How well did you know this?
1
Not at all
2
3
4
5
Perfectly
78
Q

An 8 year old boy falls onto an outstretched hand and sustains a supracondylar fracture. In addition to a weak radial pulse the child is noted to have loss of pronation of the affected hand. Which nerve is compromised?

	Median
	Radial
	Ulnar
	Musculocutaneous
	Axillary
A

Ans: A

This is a common injury in children. In this case the angulation and displacement have resulted in median nerve injury.

How well did you know this?
1
Not at all
2
3
4
5
Perfectly
79
Q

A 40 year old lady trips and falls through a glass door and sustains a severe laceration to her left arm. Amongst her injuries it is noticed that she has lost the ability to adduct the fingers of her left hand. Injury to which of the following nerves is most likely to account for her examination findings?

	Ulnar
	Median
	Radial
	Musculocutaneous
	Axillary
A

Ans: A

The interossei are supplied by the ulnar nerve.

How well did you know this?
1
Not at all
2
3
4
5
Perfectly
80
Q

A 53 year old man is undergoing a radical gastrectomy for carcinoma of the stomach. Which of the following structures will need to be divided to gain access to the coeliac axis?

	Lesser omentum
	Greater omentum
	Falciform ligament
	Median arcuate ligament
	Gastrosplenic ligament
A

Ans: A

The lesser omentum will need to be divided. During a radical gastrectomy this forms one of the nodal stations that will need to be taken.

How well did you know this?
1
Not at all
2
3
4
5
Perfectly
81
Q

A 76 year old man complains of symptoms of claudication. The decision is made to measure his ankle brachial pressure index. The signal from the dorsalis pedis artery is auscultated with a hand held doppler device. This vessel is the continuation of which of the following?

	Posterior tibial artery
	Anterior tibial artery
	Peroneal artery
	Popliteal artery
	None of the above
A

Ams: B

The dorsalis pedis is a continuation of the anterior tibial artery.

How well did you know this?
1
Not at all
2
3
4
5
Perfectly
82
Q

A 67 year old man is due to undergo a revisional total hip replacement using a posterior approach. After dividing gluteus maximus in the line of its fibres there is brisk arterial bleeding. Which of the following vessels is likely to be responsible?

	Profunda femoris artery
	External iliac artery
	Internal iliac artery
	Obturator artery
	Inferior gluteal artery
A

Ans: E)

The inferior gluteal artery runs on the deep surface of the gluteus maximus muscle. It is a branch of the internal iliac artery. It is commonly divided during the posterior approach to the hip joint.

How well did you know this?
1
Not at all
2
3
4
5
Perfectly
83
Q

A 17 year old lady presents with right iliac fossa pain and diagnosed as having acute appendicitis. You take her to theatre to perform a laparoscopic appendicectomy. During the procedure the scrub nurse distracts you and you inadvertently avulse the appendicular artery. The ensuing haemorrhage is likely to be supplied directly from which vessel?

	Inferior mesenteric artery
	Superior mesenteric artery
	Ileo-colic artery
	Internal iliac artery
	None of the above
A

Ans: C

The appendicular artery is a branch of the ileocolic artery.

How well did you know this?
1
Not at all
2
3
4
5
Perfectly
84
Q

Which of the vessels listed below is the most inferiorly sited single aortic branch?

	Common iliac artery
	Inferior mesenteric artery
	Superior mesenteric artery
	Gonadal artery
	Median sacral artery
A

Ans: E

The median sacral artery leaves the aorta a little above its bifurcation. It descends in the midline anterior to L4 and L5.

How well did you know this?
1
Not at all
2
3
4
5
Perfectly
85
Q

A 63 year old man who smokes heavily presents with dyspepsia. He is tested and found to be positive for helicobacter pylori infection. One evening he has an episode of haematemesis and collapses. What is the most likely vessel to be responsible?

	Portal vein
	Short gastric arteries
	Superior mesenteric artery
	Gastroduodenal artery
	None of the above
A

Ans: D

He is most likely to have a posteriorly sited duodenal ulcer. These can invade the gastroduodenal artery and present with major bleeding. Although gastric ulcers may invade vessels they do not tend to produce major bleeding of this nature.

How well did you know this?
1
Not at all
2
3
4
5
Perfectly
86
Q

Which of the structures listed below are not located within the mediastinum?

	Thymus
	Heart
	Great vessels
	Arch of azygos vein
	Vertebral bodies
A

Ans: E

The vertebral bodies lie outside of the mediastinum, as do the lungs.

How well did you know this?
1
Not at all
2
3
4
5
Perfectly
87
Q

A 22 year old man is stabbed in the chest at the level of the junction between the sternum and manubrium. Which structure is at greatest risk?

	Left atrium
	Oesophagus
	Thyroid gland
	Inferior vena cava
	Aortic arch
A

Ans: E

At the level of the Angle of Louis (Manubriosternal angle), is the surface marking for the aortic arch. The oesophagus is posteriorly located and at less risk.

How well did you know this?
1
Not at all
2
3
4
5
Perfectly
88
Q

Which of these nerves passes through the greater and lesser sciatic foramina?

	Pudendal nerve
	Sciatic nerve
	Superior gluteal nerve
	Inferior gluteal nerve
	Posterior cutaneous nerve of the thigh
A

Ans: A

Structures passing through the lesser and greater sciatic foramina (medial to lateral): PIN
Pudendal nerve
Internal pudendal artery
Nerve to obturator internus

How well did you know this?
1
Not at all
2
3
4
5
Perfectly
89
Q

Which of the following is not found within the deep perineal pouch in an adult male?

	Pudendal nerve
	Dorsal nerve of the penis
	Sphinter urethrae
	Urethral artery
	Obturator nerve
A

Ans: E

Contents of the deep perineal pouch
Urethral sphincter
Transversus perinei
Dorsal nerve of penis, muscular branches of the perineal nerve
Deep and dorsal arteries of penis, stem of origin of artery to the bulb of penis, urethral artery.

How well did you know this?
1
Not at all
2
3
4
5
Perfectly
90
Q

A cervical rib is due to which of the following?

Hyperplasia of the annulus fibrosus
Proliferation of the nucleus pulposus
Fusion of the transverse processes of the 6th and 7th cervical vertebrae
An accessory cervical vertebra
Elongation of the transverse processes of the 7th cervical vertebra
A

Ans: E

Cervical ribs occur as a result of the elongation of the transverse process of the 7th cervical vertebra. It is usually a fibrous band that attaches to the first thoracic rib.

How well did you know this?
1
Not at all
2
3
4
5
Perfectly
91
Q

Which of the structures listed below is not a content of the carotid sheath?

	Internal jugular vein
	Internal carotid artery
	Vagus nerve
	Recurrent laryngeal nerve
	Common carotid artery
A

Ans: D

Contents of carotid sheath:
Common carotid artery
Internal carotid artery
Internal jugular vein
Vagus nerve
How well did you know this?
1
Not at all
2
3
4
5
Perfectly
92
Q

A 28 year old rugby player injures his right humerus and on examination is noted to have a minor sensory deficit overlying the point of deltoid insertion into the humerus. Which of the nerves listed below is most likely to have been affected?

	Radial
	Axillary
	Musculocutaneous
	Median
	Subscapular
A

Ans: B

This patch of skin is supplied by the axillary nerve

How well did you know this?
1
Not at all
2
3
4
5
Perfectly
93
Q

A 22 year old man is undergoing a wedge excision of his great toenail. As the surgeon passes a needle into the area to administer local anaesthetic, the patient notices a sharp pain. By which pathway will this sensation be conveyed to the central nervous system?

	Anterior corticospinal tract
	Posterior spinocerebellar tract
	Cuneate fasciculus
	Vestibulospinal tract
	Spinothalamic tract
A

Ans: E

Spinothalamic tract- Pain and temperature
Vestibulospinal tract- Motor neuronal signals relating to posture
Cuneate fasciculus- Fine touch, pressure and proprioception
Posterior spinocerebellar tract- Proprioceptive signals to cerebellum
Anterior corticospinal tract- Conveys motor signals from precentral gyrus to motor cells within the cord

How well did you know this?
1
Not at all
2
3
4
5
Perfectly
94
Q

A 73 year old lady is admitted with brisk rectal bleeding. Despite attempts at resuscitation the bleeding proceeds to cause haemodynamic compromise. An upper GI endoscopy is normal. A mesenteric angiogram is performed and a contrast blush is seen in the region of the sigmoid colon. The radiologist decides to embolise the vessel supplying this area. At what spinal level does it leave the aorta?

	L2
	L1
	L4
	L3
	T10
A

Ans: D

The inferior mesenteric artery leaves the aorta at L3. It supplies the left colon and sigmoid. Its proximal continuation to communicate with the middle colic artery is via the marginal artery.

How well did you know this?
1
Not at all
2
3
4
5
Perfectly
95
Q

Inspection of the left ventricle reveals all except which of the following?

	Papillary muscles
	Trabeculae carnae
	Chordae tendinae
	Conus arteriosus
	Openings of the venae cordis minimae
A

Ans: D

The conus arteriosus (infundibulum) is the smooth walled outflow tract of the right ventricle leading to the pulmonary trunk.

How well did you know this?
1
Not at all
2
3
4
5
Perfectly
96
Q

A 23 year old lady with troublesome axillary hyperhidrosis is undergoing a thorascopic sympathectomy to treat the condition. Which of the following structures will need to be divided to access the sympathetic trunk?

	Intercostal vein
	Intercostal artery
	Parietal pleura
	Visceral pleura
	None of the above
A

Ans: C

The sympathetic chain lies posterior to the parietal pleura. During a thorascopic sympathetomy this structure will need to be divided. The intercostal vessels lie posteriorly. They may be damaged with troublesome bleeding but otherwise are best left alone as deliberate division will not improve surgical access.

How well did you know this?
1
Not at all
2
3
4
5
Perfectly
97
Q

In which space is a lumbar puncture performed?

	Subdural space
	Epidural space
	Subarachnoid space
	Extradural space
	Intraventricular space
A

Ans: C

Samples of CSF are normally obtained by inserting a needle between the third and fourth lumbar vertebrae. The tip of the needle lies in the sub arachnoid space, the spinal cord terminates at L1 and is not at risk of injury. Clinical evidence of raised intracranial pressure is a contraindication to lumbar puncture.

How well did you know this?
1
Not at all
2
3
4
5
Perfectly
98
Q

A 21 year old man is stabbed in the antecubital fossa. A decision is made to surgically explore the wound. At operation the surgeon dissects down onto the brachial artery. A nerve is identified medially, which nerve is it likely to be?

	Radial
	Recurrent branch of median
	Anterior interosseous
	Ulnar
	Median
A

Ans: E

How well did you know this?
1
Not at all
2
3
4
5
Perfectly
99
Q

A 65 year old man with long standing atrial fibrillation develops an embolus to the lower leg. The decision is made to perform an embolectomy, utilising a trans popliteal approach. After incising the deep fascia, which of the following structures will the surgeons encounter first on exploring the central region of the popliteal fossa?

	Popliteal vein
	Common peroneal nerve
	Popliteal artery
	Tibial nerve
	None of the above
A

Ans: D

The tibial nerve lies superior to the vessels in the inferior aspect of the popliteal fossa. In the upper part of the fossa the tibial nerve lies lateral to the vessels, it then passes superficial to them to lie medially. The popliteal artery is the deepest structure in the popliteal fossa.

How well did you know this?
1
Not at all
2
3
4
5
Perfectly
100
Q

A 53 year old man is undergoing a distal pancreatectomy for trauma. Which of the following vessels is responsible for the arterial supply to the tail of the pancreas?

	Splenic artery
	Pancreaticoduodenal artery
	Gastric artery
	Hepatic artery
	Superior mesenteric artery
A

Ans: A

Pancreatic head is supplied by the pancreaticoduodenal artery
Pancreatic tail is supplied by branches of the splenic artery

How well did you know this?
1
Not at all
2
3
4
5
Perfectly
101
Q

A 43 year old lady presents with varicose veins and undergoes a saphenofemoral disconnection, long saphenous vein stripping to the ankle and isolated hook phlebectomies. Post operatively she notices an area of numbness superior to her ankle. What is the most likely cause for this?

	Sural nerve injury
	Femoral nerve injury
	Saphenous nerve injury
	Common peroneal nerve injury
	Superficial peroneal nerve injury
A

Ans: C

The sural nerve is related to the short saphenous vein. The saphenous nerve is related to the long saphenous vein below the knee and for this reason full length stripping of the vein is no longer advocated.

How well did you know this?
1
Not at all
2
3
4
5
Perfectly
102
Q

Which of the following muscles does not attach to the radius?

	Pronator quadratus
	Biceps
	Brachioradialis
	Supinator
	Brachialis
A

Ans: E

The brachialis muscle inserts into the ulna. The other muscles are all inserted onto the radius.

How well did you know this?
1
Not at all
2
3
4
5
Perfectly
103
Q

A 25 year old man is stabbed in the upper arm. The brachial artery is lacerated at the level of the proximal humerus, and is being repaired. A nerve lying immediately lateral to the brachial artery is also lacerated. Which of the following is the nerve most likely to be?

	Ulnar nerve
	Median nerve
	Radial nerve
	Intercostobrachial nerve
	Axillary nerve
A

Ans: B
The brachial artery begins at the lower border of teres major and terminates in the cubital fossa by branching into the radial and ulnar arteries. In the upper arm the median nerve lies closest to it in the lateral position. In the cubital fossa it lies medial to it.

How well did you know this?
1
Not at all
2
3
4
5
Perfectly
104
Q

What is the course of the median nerve relative to the brachial artery in the upper arm?

	Medial to anterior to lateral
	Lateral to posterior to medial
	Medial to posterior to lateral
	Medial to anterior to medial
	Lateral to anterior to medial
A

Ans: E

Relations of median nerve to the brachial artery:
Lateral -> Anterior -> Medial

How well did you know this?
1
Not at all
2
3
4
5
Perfectly
105
Q

Which of the following is not a content of the cavernous sinus?

	Oculomotor nerve
	Internal carotid artery
	Opthalmic nerve
	Abducens nerve
	Optic nerve
A

Ans: E

Mnemonic for contents of cavernous sinus:
O TOM CAT

Occulomotor nerve (III)
Trochlear nerve (IV)
Ophthalmic nerve (V1)
Maxillary nerve (V2)
Carotid artery
Abducent nerve (VI)
T

OTOM=lateral wall components
CA= components within sinus

Relations
Medial Lateral
Pituitary fossa
Sphenoid sinus Temporal lobe

Contents
Lateral wall components (from top to bottom:)
Oculomotor nerve
Trochlear nerve
Ophthalmic nerve
Maxillary nerve
Contents of the sinus (from medial to lateral:)
Internal carotid artery (and sympathetic plexus)
Abducens nerve

How well did you know this?
1
Not at all
2
3
4
5
Perfectly
106
Q

Surgical occlusion of which of these structures, will result in the greatest reduction in hepatic blood flow?

	Portal vein
	Common hepatic artery
	Right hepatic artery
	Coeliac axis
	Left hepatic artery
A

Ans: A

The portal vein transports 70% of the blood supply to the liver, while the hepatic artery provides 30%. The portal vein contains the products of digestion. The arterial and venous blood is dispersed by sinusoids to the central veins of the liver lobules; these drain into the hepatic veins and then into the IVC. The caudate lobe drains directly into the IVC rather than into other hepatic veins.

How well did you know this?
1
Not at all
2
3
4
5
Perfectly
107
Q

A 43 year old man is due to undergo an excision of the sub mandibular gland. Which of the following incisions is the most appropriate for this procedure?

A transversely orientated incision 3cm below the mandible
A transversely orientated incision immediately inferior to the mandible
A vertical incision 3 cm anterior to the angle of the mandible and extending inferiorly
A transversely orientated incision 2cm above the mandible
A transversely orientated incision 12cm below the mandible
A

Ans: A

To access the sub mandibular gland a transverse incision 3cm below the mandible should be made. Incisions located higher than this may damage the marginal mandibular branch of the facial nerve.

How well did you know this?
1
Not at all
2
3
4
5
Perfectly
108
Q

A 5 year old boy presents with recurrent headaches. As part of his assessment he undergoes an MRI scan of his brain. This demonstrates enlargement of the lateral and third ventricles. Where is the most likely site of obstruction?

	Foramen of Luschka
	Foramen of Magendie
	Foramen of Munro
	Aqueduct of Sylvius
	None of the above
A

Ans: D

The CSF fills the space between the arachnoid mater and pia mater (covering surface of the brain). The total volume of CSF in the brain is approximately 150ml. Approximately 500 ml is produced by the ependymal cells in the choroid plexus (70%), or blood vessels (30%). It is reabsorbed via the arachnoid granulations which project into the venous sinuses.

Circulation

  1. Lateral ventricles (via foramen of Munro)
  2. 3rd ventricle
  3. Cerebral aqueduct (aqueduct of Sylvius)
  4. 4th ventricle (via foramina of Magendie and Luschka)
  5. Subarachnoid space
  6. Reabsorbed into the venous system via arachnoid granulations into superior sagittal sinus
Composition
Glucose: 50-80mg/dl
Protein: 15-40 mg/dl
Red blood cells: Nil
White blood cells: 0-3 cells/ mm3
How well did you know this?
1
Not at all
2
3
4
5
Perfectly
109
Q

A 23 year old man presents with appendicitis. A decision is made to perform an appendicectomy. The operation commences with a 5cm incision centered on McBurneys point. Which of the following structures will be encountered first during the dissection?

	External oblique aponeurosis
	Internal oblique muscle
	Transversalis fascia
	Rectus sheath
	Peritoneum
A

Ans: A

The external oblique will be encountered first in this location. The rectus sheath lies more medially.
The external oblique muscle is the most superficial of the abdominal wall muscles. It originates from the 5th to 12th ribs and passes inferomedially to insert into the linea alba, pubic tubercle and anterior half of the iliac crest. It is innervated by the thoracoabdominal nerves (T7-T11) and sub costal nerves.

How well did you know this?
1
Not at all
2
3
4
5
Perfectly
110
Q

A 23 year old man is undergoing an inguinal hernia repair. The surgeons mobilise the spermatic cord and place it in a hernia ring. A small slender nerve is identified superior to the cord. Which nerve is it most likely to be?

	Iliohypogastric nerve
	Pudendal nerve
	Femoral branch of the genitofemoral nerve
	Ilioinguinal nerve
	Obturator nerve
A

Ans: D

The ilioinguinal nerve passes through the inguinal canal and is the nerve most commonly identified during hernia surgery. The genitofemoral nerve splits into two branches, the genital branch passes through the inguinal canal within the cord structures. The femoral branch of the genitofemoral nerve enters the thigh posterior to the inguinal ligament, lateral to the femoral artery. The iliohypogastric nerve pierces the external oblique aponeurosis above the superficial inguinal ring.

How well did you know this?
1
Not at all
2
3
4
5
Perfectly
111
Q

A 34 year old man undergoes excision of a sarcoma from the right buttock. During the procedure the sciatic nerve is sacrificed. Which of the following will not occur as a result of this process?

Loss of extension at the knee joint
Foot drop
Inability to extend extensor hallucis longus
Unchanged sensation to the posterior aspect of the thigh
Loss of sensation to the posterior aspect of the lower leg
A

Ans: A

Extension of the knee joint is caused by the obturator and femoral nerves. Sensation to the posterior aspect of the thigh is via the posterior cutaneous nerve of the thigh and this is a direct branch from the plexus itself.

How well did you know this?
1
Not at all
2
3
4
5
Perfectly
112
Q

Where does the spinal cord terminate in neonates?

	L1
	L2
	L3
	L4
	L5
A

Ans: C

At the 3rd month the foetus’s spinal cord occupies the entire length of the vertebral canal. The vertebral column then grows longer exceeding the growth rate of the spinal cord. This results with the cord being at L3 at birth and L1-2 by adulthood.

How well did you know this?
1
Not at all
2
3
4
5
Perfectly
113
Q

A 45 year old man is undergoing a low anterior resection for a carcinoma of the rectum. Which of the following fascial structures will need to be divided to mobilise the mesorectum from the sacrum and coccyx?

	Denonvilliers fascia
	Colles fascia
	Sibsons fascia
	Waldeyers fascia
	None of the above
A

Ans: D

Fascial layers surrounding the rectum:
Anteriorly lies the fascia of Denonvilliers
Posteriorly lies Waldeyers fascia

Waldeyers fascia separates the mesorectum from the sacrum and will need to be divided.

How well did you know this?
1
Not at all
2
3
4
5
Perfectly
114
Q

A 10 year old child has a grommet inserted for a glue ear. What type of epithelium is present on the external aspect of the tympanic membrane?

	Stratified squamous
	Ciliated columnar
	Non ciliated columnar
	Non stratified squamous
	None of the above
A

Ans: A

The external aspect of the tympanic membrane is lined by stratified squamous epithelium. This is significant clinically in the development of middle ear infections when this type of epithelium may migrate inside the middle ear.

How well did you know this?
1
Not at all
2
3
4
5
Perfectly
115
Q

A 73 year old lady is admitted with acute mesenteric ischaemia. A CT angiogram is performed and a stenotic lesion is noted at the origin of the superior mesenteric artery. At which of the following levels does this branch from the aorta?

	L1
	L2
	L3
	L4
	L5
A

Ans: A

The SMA leaves the aorta at L1. It passes under the neck of the pancreas prior to giving its first branch the inferior pancreatico-duodenal artery.

How well did you know this?
1
Not at all
2
3
4
5
Perfectly
116
Q

The following statements relating to the musculocutaneous nerve are true except?

It arises from the lateral cord of the brachial plexus
It provides cutaneous innervation to the lateral side of the forearm
If damaged, then extension of the elbow joint will be impaired
It supplies the biceps muscle
It runs beneath biceps
A

Ans: C

supplies biceps, brachialis and coracobrachialis. If damaged then elbow flexion will be impaired.

How well did you know this?
1
Not at all
2
3
4
5
Perfectly
117
Q

Which of the following structures does not pass through the foramen ovale?

	Lesser petrosal nerve
	Accessory meningeal artery
	Maxillary nerve
	Emissary veins
	Otic ganglion
A

Ans: C

Mnemonic: OVALE

O tic ganglion
V3 (Mandibular nerve:3rd branch of trigeminal)
A ccessory meningeal artery
L esser petrosal nerve
E missary veins
How well did you know this?
1
Not at all
2
3
4
5
Perfectly
118
Q

Which of the cranial nerves listed below is least likely to carry parasympathetic fibres?

	III
	VII
	IX
	X
	II
A

Ans: E

Cranial nerves carrying parasympathetic fibres
X IX VII III (1973)

How well did you know this?
1
Not at all
2
3
4
5
Perfectly
119
Q

A 72 year old man is undergoing an open abdominal aortic aneurysm repair. The aneurysm is located in a juxtarenal location and surgical access to the neck of aneurysm is difficult. Which of the following structures may be divided to improve access?

	Cisterna chyli
	Transverse colon
	Left renal vein
	Superior mesenteric artery
	Coeliac axis
A

Ans: C

The left renal vein will be stretched over the neck of the anuerysm in this location and is not infrequently divided. This adds to the nephrotoxic insult of juxtarenal aortic surgery as a supra renal clamp is also often applied. Deliberate division of the Cisterna Chyli will not improve access and will result in a chyle leak. Division of the transverse colon will not help at all and would result in a high risk of graft infection. Division of the SMA is pointless for a juxtarenal procedure.

How well did you know this?
1
Not at all
2
3
4
5
Perfectly
120
Q

At which of the following levels does the inferior thyroid artery enter the thyroid gland?

	C6
	C2
	C4
	C3
	C5
A

Ans: A

It enters the gland at C6.

How well did you know this?
1
Not at all
2
3
4
5
Perfectly
121
Q

An occlusion of the anterior cerebral artery may compromise the blood supply to the following structures except:

	Medial inferior surface of the frontal lobe
	Corpus callosum
	Medial surface of the frontal lobe
	Olfactory bulb
	Brocas area
A

Ans: E

Brocas area is usually supplied by branches from the middle cerebral artery.

How well did you know this?
1
Not at all
2
3
4
5
Perfectly
122
Q

What is the longest part of the male urethra?

	Membranous urethra
	Spongy urethra
	Prostatic urethra
	Urethra within the internal urethra orifice
	Urethra within the urethral crest
A

Ans: B

The spongy urethra is around 15cm long and is the longest part of the male urethra.

How well did you know this?
1
Not at all
2
3
4
5
Perfectly
123
Q

Parasympathetic fibres innervating the parotid gland originate from which of the following?

	Submandibular ganglion
	Otic ganglion
	Ciliary ganglion
	Pterygopalatine ganglion
	None of the above
A

Ans: B

Secretion of saliva by the parotid gland is controlled by nerve fibres originating in the inferior salivatory nucleus; these leave the brain via the tympanic nerve (branch of glossopharyngeal nerve (CN IX), travel through the tympanic plexus (located in the middle ear), and then form the lesser petrosal nerve until reaching the otic ganglion. After synapsing in the Otic ganglion, the postganglionic (postsynaptic) fibres travel as part of the auriculotemporal nerve (a branch of the mandibular nerve (V3) to reach the parotid gland.

How well did you know this?
1
Not at all
2
3
4
5
Perfectly
124
Q

Following an oesophagogastrectomy the surgeons will anastomose the oesophageal remnant to the stomach, which of the following is not part of the layers that comprise the oesophageal wall?

	Serosa
	Adventitia
	Muscularis propria
	Submucosa
	Mucosa
A

Ans: A

The oesophageal wall lacks the serosa layer

The wall lacks a serosa which can make the wall hold sutures less securely.

How well did you know this?
1
Not at all
2
3
4
5
Perfectly
125
Q

Which of the following structures suspends the spinal cord in the dural sheath?

	Filum terminale
	Conus medullaris
	Ligamentum flavum
	Denticulate ligaments
	Anterior longitudinal ligament
A

Ans: D

The spinal cord is approximately 45cm in men and 43cm in women. The denticulate ligament is a continuation of the pia mater (innermost covering of the spinal cord) which has intermittent lateral projections attaching the spinal cord to the dura mater.

How well did you know this?
1
Not at all
2
3
4
5
Perfectly
126
Q

Where is the ‘safe triangle’ for chest drain insertion located?

4th intercostal space, mid axillary line
5th intercostal space, mid axillary line
4th intercostal space, mid scapular line
5th intercostal space, mid scapular line
4th intercostal space, mid clavicular line
A

Ans: B

‘Safe Triangle’ for chest drain insertion:

5th intercostal space, mid axillary line

How well did you know this?
1
Not at all
2
3
4
5
Perfectly
127
Q

A 32 year old rugby player is hit hard on the shoulder during a rough tackle. Clinically, his arm is hanging loose on the side. It is pronated and medially rotated. What structure is most likely to have been compromised?

	Brachial trunks C5-6
	Brachial trunks C6-7
	Brachial trunks C8 - T1
	Anterior interosseous nerve
	Posterior interosseous nerve
A

Ans: A

The patient has an Erb’s palsy involving brachial trunks C5-6.

How well did you know this?
1
Not at all
2
3
4
5
Perfectly
128
Q

Your consultant decides to perform an open inguinal hernia repair under local anaesthesia. Which of the following dermatomal levels will require blockade?

	T10
	T12
	T11
	S1
	S2
A

Ans: B

See the figure

How well did you know this?
1
Not at all
2
3
4
5
Perfectly
129
Q

A 73 year old man undergoes an excision biopsy of a lymph node that is closely applied to sternocleidomastoid. This muscle is mobilized and a nerve that is present is damaged. Which muscle below is most likely to be affected?

	Trapezius
	Rhomboid major
	Deltoid
	Supraspinatus
	Rhomboid minor
A

Ans: A

The accessory nerve has a number of lymph nodes applied to it near the sternocleidomastoid muscle. It is particularly at risk if SCM is mobilized. If injured, the trapezius muscle and SCM will be paralysed.

How well did you know this?
1
Not at all
2
3
4
5
Perfectly
130
Q

A 35 year tennis player attends reporting tingling down his arm. He says that his ‘funny bone’ was hit very hard by a tennis ball. There is weakness of abduction and adduction of his extended fingers. Which nerve has been affected?

	Ulnar
	Anterior interosseous
	Posterior interosseous
	Median
	Musculocutaneous
A

Ans: A

The ulnar nerve arises from the medial cord of the brachial plexus (C8, T1 and contribution from C7). The nerve descends between the axillary artery and vein, posterior to the cutaneous nerve of the forearm and then lies anterior to triceps on the medial side of the brachial artery. In the distal half of the arm it passes through the medial intermuscular septum, and continues between this structure and the medial head of triceps to enter the forearm between the medial epicondyle of the humerus and the olecranon. It may be injured at this site in this scenario.

How well did you know this?
1
Not at all
2
3
4
5
Perfectly
131
Q

A 44 year old man is undergoing a parotidectomy and the surgeon is carefully preserving the facial nerve. Unfortunately his trainee then proceeds to divide it. Which of the following will not be affected as a result?

	Taste sensation from anterior two thirds of the tongue
	Closing the ipsilateral eyelid
	Raising the ipsilateral side of the lip
	Ipsilateral corneal reflex
	None of the above
A

Ans: A

The chorda tympani branches inside the facial canal and will therefore be unaffected by this most unfortunate event! The corneal reflex is mediated by the opthalmic branch of the trigeminal nerve sensing the stimulus on the cornea, lid or conjunctiva; the facial nerve initiates the motor response of the reflex.

How well did you know this?
1
Not at all
2
3
4
5
Perfectly
132
Q

Which cranial nerve supplies general sensation to the posterior third of the tongue?

	Facial
	Trigeminal
	Vagus
	Hypoglossal
	Glossopharyngeal
A

Ans: E

The glossopharyngeal nerve supplies general sensation to the posterior third of the tongue and contributes to the gag reflex.

How well did you know this?
1
Not at all
2
3
4
5
Perfectly
133
Q

A 45 year old lady develops severe back pain and on examination is found to have clinical evidence of an L5/ S1 radiculopathy. Her symptoms deteriorate and eventually a laminectomy is performed. During a posterior surgical approach the surgeons encounter a tough ligamentous structure lying anterior to the spinous processes. This structure is most likely to be the

	Transverse spinal ligament
	Supraspinal ligament
	Anterior longitudinal ligament
	Ligamentum flavum
	Posterior longitudinal ligament
A

Ans: D

How well did you know this?
1
Not at all
2
3
4
5
Perfectly
134
Q

Which of the following does not pass through the superior orbital fissure?

	Lacrimal nerve
	Abducens nerve
	Opthalmic artery
	Trochlear nerve
	Superior opthalmic vein
A

Ans: C

Mnemonic for the nerves passing through the supraorbital fissure:

Live Frankly To See Absolutely No Insult

Lacrimal
Frontal
Trochlear
Superior Division of Oculomotor
Abducens
Nasociliary
Inferior Division of Oculomotor nerve
How well did you know this?
1
Not at all
2
3
4
5
Perfectly
135
Q

An 18 year old man undergoes a tonsillectomy for attacks of recurrent acute tonsillitis. Whilst in recovery he develops a post operative haemorrhage. Which of the following vessels is the most likely culprit?

	Facial vein
	External palatine vein
	External carotid artery
	Internal jugular vein
	None of the above
A

Ans: B

The external palatine vein lies immediately lateral to the tonsil and if damaged may be a cause of reactionary haemorrhage following tonsillectomy.

How well did you know this?
1
Not at all
2
3
4
5
Perfectly
136
Q

Which of the nerves listed below is responsible for providing innervation to the lower molar teeth?

	Greater palatine nerve
	Nasopalatine nerve
	Inferior alveolar nerve
	Zygomatic nerve
	Mandibular nerve
A

Ans: C

The branches of the lower molar and premolar teeth are supplied by branches of the inferior alveolar nerve. Those of the canine and incisors by the incisive branch of the same nerve. The gingiva and supporting structures are innervated by the lingual nerve.

How well did you know this?
1
Not at all
2
3
4
5
Perfectly
137
Q

A patient is found to have an ischaemic left colon. Which artery arising from the aorta at around the level of L3 is most likely to account for this situation?

	Superior mesenteric artery
	Inferior mesenteric artery
	Superior rectal artery
	Ileocolic artery
	Middle colic artery
A

Ans: B

How well did you know this?
1
Not at all
2
3
4
5
Perfectly
138
Q

At which level does the aorta traverse the diaphragm?

	T10
	T9
	T8
	T11
	T12
A

A: E

Memory aid:
T8 (8 letters) = vena cava
T10 (10 letters) = oesophagus
T12 (12 letters) = aortic hiatus

How well did you know this?
1
Not at all
2
3
4
5
Perfectly
139
Q

What is the arterial blood supply to the lacrimal apparatus?

	Nasociliary artery
	Supra orbital artery
	Internal carotid artery
	Ophthalmic artery
	Supra trochlear artery
A

A: D

The ophthalmic artery supplies the gland

How well did you know this?
1
Not at all
2
3
4
5
Perfectly
140
Q

A 24 year old lady is stabbed in the buttock. Following the injury the wound is sutured in the emergency department. Eight weeks later she attends the clinic, as she walks into the clinic room she has a waddling gait and difficulty with thigh abduction. On examination she has buttock muscle wasting. Which nerve has been injured?

	Superior gluteal nerve
	Obturator nerve
	Sciatic nerve
	Femoral nerve
	Inferior gluteal nerve
A

Ans: A

Damage to the superior gluteal nerve will result in a Trendelenburg gait.

How well did you know this?
1
Not at all
2
3
4
5
Perfectly
141
Q

At which level is the hilum of the left kidney located?

	L1
	L2
	T12
	T11
	L3
A

A: A

Remember L1 (‘left one’) is the level of the hilum of the left kidney

How well did you know this?
1
Not at all
2
3
4
5
Perfectly
142
Q

During a radical neck dissection, division of which of the following fascial layers will expose the ansa cervicalis?

	Pretracheal fascia
	Carotid sheath
	Prevertebral fascia
	Investing layer of fascia
	Sibsons fascia
A

Ans: A

The ansa cervicalis lies anterior to the carotid sheath. It may be exposed by division of the pretracheal fascia at the posterolateral aspect of the thyroid gland. The pre vertebral fascia lies more posteriorly and division of the investing layer of fascia will not expose this nerve.

How well did you know this?
1
Not at all
2
3
4
5
Perfectly
143
Q

A 73 year old lady presents with symptoms of faecal incontinence. On examination she has weak anal sphincter muscles. What are the main nerve root values of the nerves supplying the external anal sphincter?

	S2,3
	L5, S1
	S4,5
	S5
	S2,3,4
A

Ans: E

S2, 3, 4 Keeps the poo off the floor

The external anal sphincter is innervated by the inferior rectal branch of the pudendal nerve, this has root values of S2, 3 and the perineal branch of S4.

How well did you know this?
1
Not at all
2
3
4
5
Perfectly
144
Q

A 22 year old falls over and lands on a shard of glass. It penetrates the palmar aspect of his hand, immediately lateral to the pisiform bone. Which of the following structures is most likely to be injured?

Palmar cutaneous branch of the median nerve
Lateral tendons of flexor digitorum superficialis
Ulnar artery
Flexor carpi radialis tendons
Lateral tendons of flexor digitorum profundus
A

Ans: C

The ulnar nerve and artery are at most immediate risk in this injury.

How well did you know this?
1
Not at all
2
3
4
5
Perfectly
145
Q

A 72 year old man has a fall. He is found to have a fractured neck of femur and goes on to have a left hip hemiarthroplasty. Two months post operatively he is found to have an odd gait. When standing on his left leg his pelvis dips on the right side. There is no foot drop. What is the cause?

	Sciatic nerve damage
	L5 radiculopathy
	Inferior gluteal nerve damage
	Previous poliomyelitis
	Superior gluteal nerve damage
A

Ans: E

This patient has a trendelenburg gait caused by damage to the superior gluteal nerve causing weakness of the abductor muscles. Classically a patient is asked to stand on one leg and the pelvis dips on the opposite side. The absence of a foot drop excludes the possibility of polio or L5 radiculopathy.

How well did you know this?
1
Not at all
2
3
4
5
Perfectly
146
Q

Which of the following structures lies posterior to the femoral nerve in the femoral triangle?

	Adductor longus
	Pectineus
	Psoas major
	Iliacus
	None of the above
A

Ans: D

The iliacus lies posterior to the femoral nerve in the femoral triangle. The femoral sheath lies anterior to the iliacus and pectineus muscles.

Mnemonic for femoral nerve supply

(don’t) M I S V Q Scan for PE
M edial cutaneous nerve of the thigh
I ntermediate cutaneous nerve of the thigh
S aphenous nerve

V astus
Q uadriceps femoris
S artorius

PE ectineus

How well did you know this?
1
Not at all
2
3
4
5
Perfectly
147
Q

You are assisting in an open right adrenalectomy for a large adrenal adenoma. The consultant is distracted and you helpfully pull the adrenal into the wound to improve the view. Unfortunately this is followed by brisk bleeding. The vessel responsible for this is most likely to be:

	Portal vein
	Phrenic vein
	Right renal vein
	Superior mesenteric vein
	Inferior vena cava
A

Ans: E

How well did you know this?
1
Not at all
2
3
4
5
Perfectly
148
Q

A 28 year old lady requires an episiotomy for a ventouse vaginal delivery. Which of the nerves listed below will usually be anaesthetised to allow the episiotomy?

	Femoral
	Ilioinguinal
	Pudendal
	Genitofemoral
	Sacral plexus
A

Ans: C

The pudendal nerve innervates the posterior vulval area and is routinely blocked in procedures such as episiotomy.

How well did you know this?
1
Not at all
2
3
4
5
Perfectly
149
Q

A motorcyclist is involved in a road traffic accident. He suffers a complex humeral shaft fracture which is plated. Post operatively he complains of an inability to extend his fingers. Which of the following structures is most likely to have been injured?

	Ulnar nerve
	Radial nerve
	Median nerve
	Axillary nerve
	None of the above
A

Ans: B

Mnemonic for radial nerve muscles: BEST

B rachioradialis
E xtensors
S upinator
T riceps

How well did you know this?
1
Not at all
2
3
4
5
Perfectly
150
Q

An enthusiastic surgical registrar undertakes his first solo splenectomy. The operation is far more difficult than anticipated and the registrar leaves a tube drain to the splenic bed at the end of the procedure. Over the following 24 hours approximately 500ml of clear fluid has entered the drain. Biochemical testing of the fluid is most likely to reveal:

	Elevated creatinine
	Elevated triglycerides
	Elevated glucagon
	Elevated amylase
	None of the above
A

Ans: D

During splenectomy the tail of the pancreas may be damaged. The pancreatic duct will then drain into the splenic bed, amylase is the most likely biochemical finding. Glucagon is not secreted into the pancreatic duct.

How well did you know this?
1
Not at all
2
3
4
5
Perfectly
151
Q

A 48 year old lady is undergoing an axillary node clearance for breast cancer. Which of the structures listed below are most likely to be encountered during the axillary dissection?

	Cords of the brachial plexus
	Thoracodorsal trunk
	Internal mammary artery
	Thoracoacromial artery
	None of the above
A

Ans: B

Beware of damaging the thoracodorsal trunk if a latissimus dorsi flap reconstruction is planned.

The thoracodorsal trunk runs through the nodes in the axilla. If injured it may compromise the function and blood supply to latissimus dorsi, which is significant if it is to be used as a flap for a reconstructive procedure.

How well did you know this?
1
Not at all
2
3
4
5
Perfectly
152
Q

A 56 year old lady is referred to the colorectal clinic with symptoms of pruritus ani. On examination a polypoidal mass is identified inferior to the dentate line. A biopsy confirms squamous cell carcinoma. To which of the following lymph node groups will the lesion potentially metastasise?

	Internal iliac
	External iliac
	Mesorectal
	Inguinal
	None of the above
A

Ans: D

Lesions distal to the dentate line drain to the inguinal nodes. Occasionally this will result in the need for a block dissection of the groin.

Mesorectal lymph nodes (superior to dentate line)
Inguinal nodes (inferior to dentate line)
How well did you know this?
1
Not at all
2
3
4
5
Perfectly
153
Q

Which of the following ligaments contains the artery supplying the head of femur in children?

	Transverse ligament
	Ligamentum teres
	Iliofemoral ligament
	Ischiofemoral ligament
	Pubofemoral ligament
A

Ans: B

How well did you know this?
1
Not at all
2
3
4
5
Perfectly
154
Q

A 72 year old man develops a hydrocele which is being surgically managed. As part of the procedure the surgeons divide the tunica vaginalis. From which of the following is this structure derived?

	Peritoneum
	External oblique aponeurosis
	Internal oblique aponeurosis
	Transversalis fascia
	Rectus sheath
A

A: A

The tunica vaginalis is derived from peritoneum, it secretes the fluid that fills the hydrocele cavity.

How well did you know this?
1
Not at all
2
3
4
5
Perfectly
155
Q

A 43 year old lady is donating her left kidney to her sister and the surgeons are harvesting the left kidney. Which of the following structures will lie most anteriorly at the hilum of the left kidney?

	Left renal artery
	Left renal vein
	Left ureter
	Left ovarian vein
	Left ovarian artery
A

Ans: B

The renal veins lie most anteriorly, then artery and ureter lies posteriorly.

How well did you know this?
1
Not at all
2
3
4
5
Perfectly
156
Q

What is the sensory nerve supply to the angle of the jaw?

Maxillary branch of the trigeminal nerve
Mandibular branch of the trigeminal nerve
C3-C4
Greater auricular nerve (C2-C3)
Buccal branch of the facial nerve
A

Ans: D

The trigeminal nerve is the major sensory nerve to the face except over the angle of the jaw. The angle of the jaw is innervated by the greater auricular nerve.

How well did you know this?
1
Not at all
2
3
4
5
Perfectly
157
Q

A 63 year old man is undergoing a coronary artery bypass procedure. During the median sternotomy which structure would routinely require division?

	Parietal pleura
	Interclavicular ligament
	Internal mammary artery
	Brachiocephalic vein
	Left vagus nerve
A

Ans: B

The interclavicular ligament lies at the upper end of a median sternotomy and is routinely divided to provide access.

How well did you know this?
1
Not at all
2
3
4
5
Perfectly
158
Q

A 42 year old woman complains of a burning pain of her anterior thigh which worsens on walking. There is a positive tinel sign over the inguinal ligament. Which nerve is affected?

	Ilioinguinal nerve
	Genitofemoral nerve
	Lateral cutaneous nerve of the thigh
	Femoral nerve
	Saphenous nerve
A

Ans: C

The lateral cutaneous nerve supplies sensation to the anterior and lateral aspect of the thigh. Entrapment is commonly due to intra and extra pelvic causes. Treatment involves local anaesthetic injections.

How well did you know this?
1
Not at all
2
3
4
5
Perfectly
159
Q

Which of the following structures separates the subclavian artery from the subclavian vein?

	Scalenus anterior
	Scalenus medius
	Sternocleidomastoid
	Pectoralis major
	Pectoralis minor
A

Ans: A

The artery and vein are separated by scalenus anterior. This muscle runs from the transverse processes of C3,4,5 and 6 to insert onto the scalene tubercle of the first rib.

How well did you know this?
1
Not at all
2
3
4
5
Perfectly
160
Q

A 56 year old lady is due to undergo a left hemicolectomy for carcinoma of the splenic flexure. The surgeons decide to perform a high ligation of the inferior mesenteric vein. Into which of the following does this structure usually drain?

	Portal vein
	Inferior vena cava
	Left renal vein
	Left iliac vein
	Splenic vein
A

Ans: E

The inferior mesenteric vein drains into the splenic vein, this point of union lies close to the duodenum and this surgical maneouvre is a recognised cause of ileus.

How well did you know this?
1
Not at all
2
3
4
5
Perfectly
161
Q

A 43 year old lady is due to undergo an axillary node clearance as part of treatment for carcinoma of the breast. Which of the following fascial layers will be divided during the surgical approach to the axilla?

	Sibsons fascia
	Pre tracheal fascia
	Waldayers fascia
	Clavipectoral fascia
	None of the above
A

Ans: D

The clavipectoral fascia is situated under the clavicular portion of pectoralis major.

How well did you know this?
1
Not at all
2
3
4
5
Perfectly
162
Q

What are the boundaries of the ‘safe triangle’ for chest drain insertion?

Bounded by trapezius, latissimus dorsi, and laterally by the vertebral border of the scapula
Bounded by latissimus dorsi, pectoralis major, line superior to the nipple and apex at the axilla
Bounded by latissimus dorsi, serratus anterior, line superior to the nipple and apex at the axilla
Bounded by trapezius, deltoid, rhomboid major and teres minor
Bounded by trapezius, deltoid and latissimus dorsi
A

Ans: B

How well did you know this?
1
Not at all
2
3
4
5
Perfectly
163
Q

Where is a gomphoses type of fibrous joint typically found?

	Teeth
	Skull
	Manubriosternum
	Ribs
	Femur
A

Ans: A

How well did you know this?
1
Not at all
2
3
4
5
Perfectly
164
Q

The vertebral artery traverses all of the following except?

	Transverse process of C6
	Transverse process of the axis
	Vertebral canal
	Foramen magnum
	Intervertebral foramen
A

Ans: E

The vertebral artery passes through the foramina which are located in the transverse processes of the cervical vertebra, it does not traverse the intervertebral foramen.

How well did you know this?
1
Not at all
2
3
4
5
Perfectly
165
Q

A 60 year old female attends the preoperative hernia clinic. She reports some visual difficulty. On examination she is noted to have a homonymous hemianopia. Where is the lesion most likely to be?

	Frontal lobe
	Pituitary gland
	Parietal lobe
	Optic chiasm
	Optic tract
A

Ans: E

Lesions before optic chiasm:
Monocular vision loss = Optic nerve lesion
Bitemporal hemianopia = Optic chiasm lesion

Lesions after the optic chiasm:
Homonymous hemianopia = Optic tract lesion
Upper quadranopia = Temporal lobe lesion
Lower quadranopia = Parietal lobe lesion

How well did you know this?
1
Not at all
2
3
4
5
Perfectly
166
Q

A 34 year old male is being examined in the pre-operative assessment clinic. A murmur is identified in the 4th intercostal space just next to the left side of the sternum. From where is it most likely to have originated?

	Mitral valve
	Aortic valve
	Pulmonary valve
	Right ventricular aneurysm
	Tricuspid valve
A

Ans: E

The tricuspid valve is generally referred to being best auscultated adjacent to the sternum. The plane of projected sound from the mitral area is best heard in the region of the cadiac apex.

Pulmonary valve
Left second intercostal space, at the upper sternal border

Aortic valve
Right second intercostal space, at the upper sternal border

Mitral valve
Left fifth intercostal space, just medial to mid clavicular line

Tricuspid valve
Left fourth intercostal space, at the lower left sternal border

How well did you know this?
1
Not at all
2
3
4
5
Perfectly
167
Q

Which of the following statements relating to quadratus lumborum is false?

	Causes flexion of the thoracic spine
	Causes the rib cage to be pulled down
	Innervated by anterior primary rami of T12 and L1-3
	Attached to the iliac crest
	Inserts into the 12th rib
A

Ans: A

Quadratus lumborum
Origin: Medial aspect of iliac crest and iliolumbar ligament
Insertion: 12th rib
Action: Pulls the rib cage inferiorly. Lateral flexion.
Nerve supply: Anterior primary rami of T12 and L1-3

The rectus abdominis causes flexion of the thoracic spine and therefore the statement suggesting that quaratus lumborum does so is incorrect.

How well did you know this?
1
Not at all
2
3
4
5
Perfectly
168
Q

A 23 year old climber falls and fractures his humerus. The surgeons decide upon a posterior approach to the middle third of the bone. Which of the following nerves is at greatest risk in this approach?

	Ulnar
	Antebrachial
	Musculocutaneous
	Radial
	Intercostobrachial
A

Ans: D

The radial nerve wraps around the humerus and may be injured during a posterior approach. An IM nail may be preferred as it avoids the complex dissection needed for direct bone exposure.

How well did you know this?
1
Not at all
2
3
4
5
Perfectly
169
Q

A 67 year old man is undergoing an angiogram for gastro intestinal bleeding. The radiologist advances the catheter into the coeliac axis. At what spinal level does this vessel typically arise from the aorta?

	T10
	L3
	L4
	T12
	None of the above
A

Ans: D

How well did you know this?
1
Not at all
2
3
4
5
Perfectly
170
Q

Which muscle does not insert on the medial surface of the greater trochanter?

	Gemelli
	Obturator internus
	Piriformis
	Quadratus femoris
	Obturator externus
A

Ans: D

Mnemonic for muscle attachment on greater trochanter is POGO:
Piriformis
Obturator internus
Gemelli
Obturator externus
How well did you know this?
1
Not at all
2
3
4
5
Perfectly
171
Q

What is the largest branch of the brachial artery?

	Radial artery
	Ulnar artery
	Profunda brachii artery
	Humeral nutrient artery
	Ulnar collateral artery
A

Ans: C

The profunda brachii artery is the largest branch and then continues in the radial groove of the humerus.

How well did you know this?
1
Not at all
2
3
4
5
Perfectly
172
Q

During a radical gastrectomy for carcinoma of the stomach the surgeons remove the omentum. What is the main source of its blood supply?

	Ileocolic artery
	Superior mesenteric artery
	Gastroepiploic artery
	Middle colic artery
	Inferior mesenteric artery
A

Ans: C

How well did you know this?
1
Not at all
2
3
4
5
Perfectly
173
Q

A 38 year old lady is due to undergo a parathyroidectomy for hyperparathyroidism. At operation the inferior parathyroid gland is identified as being enlarged. A vessel is located adjacent to the gland laterally. This vessel is most likely to be the:

	External carotid artery
	Common carotid artery
	Internal carotid artery
	External jugular vein
	None of the above
A

Ans: B

The common carotid artery is a lateral relation of the inferior parathyroid.

Relations
Laterally Common carotid

Medially Recurrent laryngeal nerve, trachea

Anterior Thyroid

Posterior Pretracheal fascia

How well did you know this?
1
Not at all
2
3
4
5
Perfectly
174
Q

A 45 year old man has a long femoral line inserted to provide CVP measurements. The catheter passes from the common iliac vein into the inferior vena cava. At which of the following vertebral levels will this occur?

	L5
	L4
	S1
	L3
	L2
A

Ans: A

The common iliac veins fuse with the IVC at L5.

How well did you know this?
1
Not at all
2
3
4
5
Perfectly
175
Q

At which of the following levels does the inferior vena cava exit the abdominal cavity?

	T6
	T7
	T10
	T8
	T12
A

Ans: D

How well did you know this?
1
Not at all
2
3
4
5
Perfectly
176
Q

Which of the following structures lies deepest in the popliteal fossa?

	Popliteal artery
	Popliteal vein
	Tibial nerve
	Common peroneal nerve
	Popliteal lymph nodes
A

Ans: A

The common peroneal nerve exits the popliteal fossa along the medial border of the biceps tendon. Then the tibial nerve lies lateral to the popliteal vessels to pass posteriorly and then medially to them. The popliteal vein lies superficial to the popliteal artery, which is the deepest structure in the fossa.

How well did you know this?
1
Not at all
2
3
4
5
Perfectly
177
Q

How to remember the nerve roots of reflexes?

A

To remember nerve roots and their reflexes:

1-2 Ankle (S1-S2)
3-4 Knee (L3-L4)
5-6 Biceps (C5-C6)
7-8 Triceps (C7-C8)

How well did you know this?
1
Not at all
2
3
4
5
Perfectly
178
Q

A 23 year old man complains of severe groin pain several weeks after a difficult inguinal hernia repair. Which nerve is most likely to have been involved?

	Genitofemoral
	Ilioinguinal
	Femoral
	Iliohypogastric
	Pudendal
A

Ans: B

The ilioinguinal nerve may have been entrapped in the mesh causing a neuroma.

179
Q

Which of the positions listed below best describes the location of the coeliac autonomic plexus?

	Anterolateral to the aorta
	Posterolateral to the aorta
	Anterolateral to the sympathetic chain
	Anteromedial to the sympathetic chain
	Posterior to L1
A

Ans: A

180
Q

An intravenous drug user develops a false aneurysm and requires emergency surgery. The procedure is difficult and the femoral nerve is inadvertently transected. Which of the following muscles is least likely to be affected as a result?

	Sartorius
	Vastus medialis
	Pectineus
	Quadriceps femoris
	Adductor magnus
A

Ans: E

Mnemonic for femoral nerve supply

(don’t) M I S V Q Scan for PE
M edial cutaneous nerve of the thigh
I ntermediate cutaneous nerve of the thigh
S aphenous nerve

V astus
Q uadriceps femoris
S artorius

PE ectineus

181
Q

What is the nerve root value of the external urethral sphincter?

	S4
	S1, S2, S3
	S2, S3, S4
	L3, L4, L5
	L5, S1, S2
A

Ans: C

The external urethral sphincter is innervated by branches of the pudendal nerve, therefore the root values are S2, S3, S4.

182
Q

A 45 year old man is stabbed in the abdomen and the inferior vena cava is injured. How many functional valves does this vessel usually have?

	0
	1
	3
	2
	4
A

Ans: A

Mnemonic for the Inferior vena cava tributaries: I Like To Rise So High:

Iliacs
Lumbar
Testicular
Renal
Suprarenal
Hepatic vein
183
Q

Which of the following structures does not pass posteriorly to the medial malleolus?

	Posterior tibial artery
	Tibial nerve
	Tibialis anterior tendon
	Tendon of flexor digitorum longus
	Tendon of flexor hallucis longus
A

Ans: C

Mnemonic for structures posterior to the medial malleolus:

Tom Dick And Nervous Harry

T ibialis posterior tendon
flexor Digitorum longus
A rtery
N erve
H allucis longus
184
Q

Which of the following statements relating to the root of the neck is false?

The lung projects into the neck beyond the first rib and is constrained by Sibson's fascia
The subclavian artery arches over the first rib anterior to scalenus anterior
The trunks of the brachial plexus lie posterior to the subclavian artery on the first rib
The roots and trunks of the Brachial plexus lie between scalenus anterior and scalenus medius muscles
The thyrocervical trunk is a branch of the subclavian artery
A

Ans: B

The subclavian artery lies posterior to scalenus anterior, the vein lies in front. Sibson’s fascia is another name for the suprapleural membrane.

185
Q

A patient presents to the clinic following a surgical procedure. She complains that she is unable to shrug her shoulder. What is the most likely underlying nerve injury?

	Accessory nerve
	Cervical plexus
	Ansa cervicalis
	Long thoracic nerve
	Axillary nerve
A

Ans: A

The accessory nerve may be injured in operations in the posterior triangle. Injury will affect trapezius.

186
Q

A 53 year old man is to undergo a thyroidectomy. Which nerve is at greatest risk?

	Hypoglossal
	Recurrent laryngeal
	Ansa cervicalis
	Accessory
	Marginal mandibular
A

Ans: B

Recurrent laryngeal nerve injury may complicate thyroid surgery in up to 1- 2% of cases.

187
Q

The cords of the brachial plexus are most closely related to which of the following vessels?

	Subclavian artery
	Axillary artery
	Axillary vein
	Subclavian vein
	Brachial artery
A

Ans: B

The trunks are related to the subclavian artery superiorly. The cords of the plexus surround the axillary artery, they are named according to their positions relative to this structure.

188
Q

Which of the following are not generally supplied by the right coronary artery?

	The sino atrial node
	The circumflex artery
	The atrioventricular node
	Most of the right ventricle
	The right atrium
A

Ans: B

The circumflex artery is generally a branch of the left coronary artery.

Right coronary artery
The RCA supplies:
Right atrium
Diaphragmatic part of the right ventricle
Usually the posterior third of the interventricular septum
The sino atrial node (60% cases)
The atrio ventricular node (80% cases)

Left coronary artery
The LCA supplies:
Left atrium
Most of left ventricle
Part of the right ventricle
Anterior two thirds of the inter ventricular septum
The sino atrial node (remaining 40% cases)

Innervation of the heart
Autonomic nerve fibres from the superficial and deep cardiac plexus. These lie anterior to the bifurcation of the trachea, posterior to the ascending aorta and superior to the bifurcation of the pulmonary trunk. The parasympathetic supply to the heart is from presynaptic fibres of the vagus nerves.

189
Q

A 44 year old man has a malignant melanoma and is undergoing a block dissection of the groin. The femoral triangle is being explored for intra operative bleeding. Which of the following forms the medial border of the femoral triangle?

	Femoral artery
	Biceps femoris
	Adductor longus
	Sartorius
	Adductor magnus
A

Ans: C

Femoral triangle anatomy

Boundaries
Superiorly Inguinal ligament

Laterally Sartorius

Medially Adductor longus

Floor Iliopsoas, adductor longus and pectineus

Roof
Fascia lata and Superficial fascia
Superficial inguinal lymph nodes (palpable below the inguinal ligament)
Long saphenous vein

190
Q

The foramen marking the termination of the adductor canal is located in which of the following?

	Adductor longus
	Adductor magnus
	Adductor brevis
	Sartorius
	Semimembranosus
A

Ans: B

The foramen marking the distal limit of the adductor canal is contained within adductor magnus. The vessel passes through this region to enter the popliteal fossa.

191
Q

Which of the following is the first vessel to branch from the external carotid artery?

	Superior thyroid artery
	Inferior thyroid artery
	Lingual artery
	Facial artery
	Occipital artery
A

Ans: A

Mnemonic
(Order in which they branch off)Some (sup thyroid)Attendings (Ascending Pharyngeal)Like (Lingual)Freaking (Facial)Out (Occipital)Potential (Post auricular)Medical (Maxillary)Students (Sup temporal)

The first branch of the external carotid artery is the superior thyroid artery. The inferior thyroid artery is derived from the thyrocervical trunk. The other branches are illustrated below.

192
Q

A motorcyclist is injured in a road traffic accident and is not wearing a helmet. He suffers a severe closed head injury and develops raised intracranial pressure. The first cranial nerve to be affected by this process is likely to be:

	Oculomotor
	Hypoglossal
	Motor branch of the trigeminal
	Abducens
	Sensory branch of the trigeminal
A

Ans: D

The abducens nerve (CN VI) has a long intra cranial course and is thus susceptible to raised intra cranial pressure. It also passes over the petrous temporal bone and 6th nerve palsies are also seen in mastoiditis.

193
Q

Which structure separates the cephalic vein and the brachial artery in the antecubital fossa?

Brachioradialis muscle
Biceps muscle
Origin of flexor digitorum profundus muscle
Pronator quadratus muscle
Origin of flexor digitorum superficialis muscle
A

Ans: B

194
Q

A 23 year old rugby player sustains a Smiths Fracture. On examination, opposition of the thumb is markedly weakened. Which of the following nerves is least likely to be working normally?

	Ulnar
	Median
	Radial
	Musculocutaneous
	Palmar cutaneous
A

Ans: B

This high velocity injury can often produce significant angulation and displacement. Both of these may impair the function of the median nerve with loss of function of the muscles of the thenar eminence.

195
Q

A 24 year old motor cyclist is involved in a road traffic accident. He suffers a tibial fracture which is treated with an intra medullary nail. Post operatively he develops a compartment syndrome. Surgical decompression of the anterior compartment will relieve pressure on all of the following muscles except?

	Peroneus brevis
	Peroneus tertius
	Extensor digitorum longus
	Tibialis anterior
	None of the above
A

Ans: A

196
Q

A 43 year old lady underwent an attempted placement of a central line into the internal jugular vein. Unfortunately, the doctor damaged the carotid artery and this necessitated surgical exploration. As the surgeons incise the carotid sheath a nerve is identified lying between the internal jugular vein and the carotid artery. Which of the following is this nerve most likely to be?

	Glossopharyngeal nerve
	Hypoglossal nerve
	Superior laryngeal nerve
	Recurrent laryngeal nerve
	Vagus
A

Ans: E

The vagus lies in the carotid sheath. The hypoglossal nerve crosses the sheath, but does not lie within it.

197
Q

A patient has a chest drain insertion. There is fresh blood at the chest drain insertion area. Which vessel has been damaged?

	Pericardiophrenic artery
	Intercostal vein
	Right ventricle
	Vagus artery
	Intercostal artery
A

Ans: E

The intercostal vein is more superior than the artery and is thus slightly less susceptible to injury.

198
Q

Two teenagers are playing with an airgun when one accidentally shoots his friend in the abdomen. He is brought to the emergency department. On examination there is a bullet entry point immediately to the right of the rectus sheath at the level of the 1st lumbar vertebra. Which of the following structures is most likely to be injured by the bullet?

	Head of pancreas
	Right ureter
	Right adrenal gland
	Fundus of the gallbladder
	Gastric antrum
A

A: D

The fundus of the gallbladder lies at this level and is the most superficially located structure.

199
Q

Which of the following muscles inserts onto the lesser tuberostiy of the the humerus?

	Subscapularis
	Deltoid
	Supraspinatus
	Teres minor
	Infraspinatus
A

Ans: A

With the exception of subscapularis which inserts into the lesser tuberosity, the muscles of the rotator cuff insert into the greater tuberosity.

200
Q

Which of the following nerves is not contained within the posterior triangle of the neck?

	Accessory nerve
	Phrenic nerve
	Greater auricular nerve
	Ansa cervicalis
	Lesser occiptal nerve
A

Ans: D

Ansa cervicalis is a content of the anterior triangle of the neck.

201
Q

A 42 year old lady is reviewed in the outpatient clinic following a routine surgical procedure. She complains of diminished sensation at the dorso-lateral aspect of her foot. Which of the following nerves is most likely to be affected?

	Sural
	Superficial peroneal
	Deep peroneal
	Medial plantar
	Lateral plantar
A

Ans: A

The sural nerve supplies the lateral aspect of the foot. It runs alongside the short saphenous vein and may be injured in short saphenous vein surgery.

202
Q

Which of the following anatomical planes separates the prostate from the rectum?

	Sibsons fascia
	Denonvilliers fascia
	Levator ani muscle
	Waldeyers fascia
	None of the above
A

Ans: B

The Denonvilliers fascia separates the rectum from the prostate. Waldeyers fascia separates the rectum from the sacrum

203
Q

A 56 year old lady is undergoing an adrenalectomy for Conns syndrome. During the operation the surgeon damages the middle adrenal artery and haemorrhage ensues. From which of the following structures does this vessel originate?

	Aorta
	Renal artery
	Splenic artery
	Coeliac axis
	Superior mesenteric artery
A

Ans: A

The middle adrenal artery is usually a branch of the aorta, the lower adrenal artery typically arises from the renal vessels.

204
Q

A 73 year old lady suffers a fracture at the surgical neck of the humerus. The decision is made to operate. There are difficulties in reducing the fracture and a vessel lying posterior to the surgical neck is injured. Which of the following is this vessel most likely to be?

	Axillary artery
	Brachial artery
	Thoracoacromial artery
	Transverse scapular artery
	Posterior circumflex humeral artery
A

Ans: E

205
Q

Which of the structures listed below lies posterior to the carotid sheath at the level of the 6th cervical vertebra?

	Hypoglossal nerve
	Vagus nerve
	Cervical sympathetic chain
	Ansa cervicalis
	Glossopharyngeal nerve
A

Ans: C The carotid sheath is crossed anteriorly by the hypoglossal nerves and the ansa cervicalis. The vagus lies within it. The cervical sympathetic chain lies posteriorly between the sheath and the prevertebral fascia.

206
Q

A sprinter attends A&E with severe leg pain. He had forgotten to warm up and ran a 100m sprint race. Towards the end of the race he experienced pain in the posterior aspect of his thigh. The pain worsens, localising to the lateral aspect of the knee. The sprinter is unable to flex the knee. What structure has been injured?

	Anterior cruciate ligament
	Posterior cruciate ligament
	Semimembranosus tendon
	Semitendinosus tendon
	Biceps femoris tendon
A

E)

The biceps femoris is commonly injured in sports that require explosive bending of the knee as seen in sprinting, especially if the athlete has not warmed up first. Avulsion most commonly occurs where the long head attaches to the ischial tuberosity. Injuries to biceps femoris are more common than to the other hamstrings.

207
Q

A 64 year old man has a suspected lymphoma and lymph node biopsy from the posterolateral aspect of the right neck is planned. Which of the nerves listed is at greatest risk?

	Accessory
	Long thoracic
	External laryngeal
	Facial
	Vagus
A

Ans: A

The accessory nerve has a superficial course and is easily injured. It lies under platysma and may be divided during the early part of the procedure.

208
Q

A 24 year old man falls and lands astride a manhole cover. He suffers from an injury to the anterior bulbar urethra. Where will the extravasated urine tend to collect?

	Lesser pelvis
	Connective tissue of the scrotum
	Deep perineal space
	Ischiorectal fossa
	Posterior abdominal wall
A

Ans: B

This portion of the urethra is contained between the perineal membrane and the membranous layer of the superficial fascia. As these are densely adherent to the ischiopubic rami, extravasated urine cannot pass posteriorly because the 2 layers are continuous around the superficial transverse perineal muscles.

209
Q

A 73 year old man presents with symptoms of mesenteric ischaemia. As part of his diagnostic work up a diagnostic angiogram is performed .The radiologist is attempting to cannulate the coeliac axis from the aorta. At which of the following vertebral levels does this is usually originate?

	T10
	L2
	L3
	T8
	T12
A

Ans: E

Coeliac trunk branches:

Left Hand Side (LHS)

Left gastric
Hepatic
Splenic

210
Q

A 45 year old man presents with a lipoma located posterior to the posterior border of the sternocleidomastoid muscle, approximately 4cm superior to the middle third of the clavicle. During surgical excision of the lesion troublesome bleeding is encountered. Which of the following is the most likely source?

	Internal jugular vein
	External jugular vein
	Common carotid artery
	Vertebral artery
	Second part of the subclavian artery
A

Ans: B

The external jugular vein runs obliquely in the superficial fascia of the posterior triangle. It drains into the subclavian vein. During surgical exploration of this area the external jugular vein may be injured and troublesome bleeding may result. The internal jugular vein and carotid arteries are located in the anterior triangle. The third, and not the second, part of the subclavian artery is also a content of the posterior triangle

211
Q

The sciatic nerve lies deep to the following structures except:

	Gluteus maximus
	The femoral cutaneous nerve
	Long head of biceps femoris
	Gluteus medius
	Branch of the inferior gluteal artery
A

Ans: D

212
Q

Which of the following upper limb muscles is not innervated by the radial nerve?

	Extensor carpi ulnaris
	Abductor digiti minimi
	Anconeus
	Supinator
	Brachioradialis
A

Ans: B

Mnemonic for radial nerve muscles: BEST

B rachioradialis
E xtensors
S upinator
T riceps

213
Q

During a liver resection a surgeon performs a pringles manoeuvre to control bleeding. Which of the following structures will lie posterior to the epiploic foramen at this level?

	Hepatic artery
	Cystic duct
	Greater omentum
	Superior mesenteric artery
	Inferior vena cava
A

Ans: E

Bleeding from liver trauma or a difficult cholecystectomy can be controlled with a vascular clamp applied at the epiploic foramen.

The epiploic foramen has the following boundaries:
Anteriorly (in the free edge of the lesser omentum): Bile duct to the right, portal vein behind and hepatic artery to the left.
PosteriorlyInferior vena cava
Inferiorly1st part of the duodenum
SuperiorlyCaudate process of the liver

214
Q

A 72 year old lady is suspected of having a femoral hernia. At which of the following sites is it most likely to be identifiable clinically?

	Mid inguinal point
	Above and medial to the pubic tubercle
	Below and lateral to the pubic tubercle
	Mid point of the inguinal ligament
	3 cm superomedially to the superficial inguinal ring
A

Ans: C

215
Q

Which muscle is responsible for causing flexion of the distal interphalangeal joint of the ring finger?

	Flexor digitorum superficialis
	Lumbricals
	Palmar interossei
	Flexor digitorum profundus
	Flexor digiti minimi brevis
A

A: D

Flexor digitorum superficialis and flexor digitorum profundus are responsible for causing flexion. The superficialis tendons insert on the bases of the middle phalanges; the profundus tendons insert on the bases of the distal phalanges. Both tendons flex the wrist, MCP and PIP joints; however, only the profundus tendons flex the DIP joints.

216
Q

A 34 year old lady undergoes a thyroidectomy for Graves disease. Post operatively she develops a tense haematoma in the neck. In which of the following fascial planes will it be contained?

	Gerotas fascia
	Waldeyers fascia
	Pretracheal fascia
	Sibsons fascia
	Clavipectoral fascia
A

C)

The pretracheal fascia encloses the thyroid and is unyielding. Therefore tense haematomas can develop.

217
Q

A 32 year old lady complains of carpal tunnel syndrome. The carpal tunnel is explored surgically. Which of the following structures will lie in closest proximity to the hamate bone within the carpal tunnel?

The tendon of abductor pollicis longus
The tendons of flexor digitorum profundus
The tendons of flexor carpi radialis longus
Median nerve
Radial artery
A

B)

The carpal tunnel contains nine flexor tendons:
Flexor digitorum profundus
Flexor digitorum superficialis
Flexor pollicis longus

The tendon of flexor digitorum profundus lies deepest in the tunnel and will thus lie nearest to the hamate bone.

218
Q

A 45 year old man sustains a significant head injury and a craniotomy is performed. The sigmoid sinus is bleeding profusely, into which of the following structures does it drain?

	Internal jugular vein
	Straight sinus
	Petrosal sinus
	Inferior sagittal sinus
	External jugular vein
A

A)

The sigmoid sinus is joined by the inferior petrosal sinus to drain into the internal jugular vein.

219
Q

Which nerve supplies the interossei of the fourth finger?

	Radial
	Median
	Superficial ulnar
	Deep ulnar
	Posterior interosseous
A

D)

Mnemonic:
PAD and DAB
Palmer interossei ADduct
Dorsal interossei ABduct

220
Q

In which of the following cranial bones does the foramen spinosum lie?

	Sphenoid bone
	Frontal bone
	Temporal bone
	Occipital bone
	Parietal bone
A

A)

The foramen spinosum (which transmits the middle meningeal artery and vein) lies in the sphenoid bone.

221
Q

Which of the following is not considered a major branch of the descending thoracic aorta?

	Bronchial artery
	Mediastinal artery
	Inferior thyroid artery
	Posterior intercostal artery
	Oesophageal artery
A

C)

The inferior thyroid artery is usually derived from the thyrocervical trunk, a branch of the subclavian artery.

222
Q

An 18 year old lady with troublesome hyperhidrosis of the hands and arms is due to undergo a sympathectomy to treat the condition. Which of the following should the surgeons divide to most effectively treat her condition?

	Sympathetic ganglia at T1, T2 and T3
	Sympathetic ganglia at T2 and T3
	Sympathetic ganglia at T1 and T2
	Stellate ganglion
	Superior cervical ganglion
A

B)

To treat hyperhidrosis the sympathetic ganglia at T2 and T3 should be divided. Dividing the other structures listed would either carry a risk of Horners syndrome or be ineffective.

223
Q

A 44 year old lady is recovering following a transphenoidal hypophysectomy. Unfortunately there is a post operative haemorrhage. Which of the following features is most likely to occur initially?

	Cavernous sinus thrombosis
	Abducens nerve palsy
	Bi-temporal hemianopia
	Inferior homonymous hemianopia
	Central retinal vein occlusion
A

C)

The pituitary is covered by a sheath of dura and an expanding haematoma at this site may compress the optic chiasm in the same manner as an expanding pituitary tumour.

224
Q

During a right hemicolectomy the caecum is mobilised. As the bowel is retracted medially a vessel is injured, posterior to the colon. Which of the following is the most likely vessel?

	Right colic artery
	Inferior vena cava
	Aorta
	External iliac artery
	Gonadal vessels
A

E)

The key in this question is that its during the caecal mobilization. The gonadal vessels and ureter are important posterior relations that are at risk during a right hemicolectomy. During latter stages of the procedure, the ileocolic artery and vein are traced along the anterior aspect of the duodenum. At this point it is possible to injure these, the superior mesenteric vein or the middle colic vein, injury to any of these can result in torrential bleeding that is very difficult to control.

225
Q

A 53 year old man with a carcinoma of the lower third of the oesophagus is undergoing an oesophagogastrectomy. As the surgeons mobilise the lower part of the oesophagus, where are they most likely to encounter the thoracic duct?

	Anterior to the oesophagus
	On the left side of the oesophagus
	On the right side of the oesophagus
	Immediately anterior to the azygos vein
	Posterior to the oesophagus
A

E)

The thoracic duct lies posterior to the oesophagus and passes to the left at the level of the Angle of Louis. It enters the thorax at T12 together with the aorta.

226
Q

Which of the following represents the root values of the sciatic nerve?

	L4 to S3
	L1 to L4
	L3 to S1
	S1 to S4
	L5 to S1
A

Ans: A

The sciatic nerve most commonly arises from L4 to S3.

227
Q

An 83 year old lady presents with a femoral hernia and undergoes a femoral hernia repair. Which of the following forms the posterior wall of the femoral canal?

	Pectineal ligament
	Lacunar ligament
	Inguinal ligament
	Adductor longus
	Sartorius
A

A)

Borders of the femoral canal
Laterally	Femoral vein
Medially	Lacunar ligament
Anteriorly	Inguinal ligament
Posteriorly	Pectineal ligament
228
Q

A 45 year man presents with hand weakness. He is given a piece of paper to hold between his thumb and index finger. When the paper is pulled, the patient has difficulty maintaining a grip. Grip pressure is maintained by flexing the thumb at the interphalangeal joint. What is the most likely nerve lesion?

	Posterior interosseous nerve
	Deep branch of ulnar nerve
	Anterior interosseous nerve
	Superficial branch of the ulnar nerve
	Radial nerve
A

B)

This is a description of Froment’s sign, which tests for ulnar nerve palsy. It mainly tests for the function of adductor pollicis. This is supplied by the deep branch of the ulnar nerve. Remember the anterior interosseous branch (of the median nerve), which innervates the flexor pollicis longus (hence causing flexion of the thumb IP joint), branches off more proximally to the wrist.

229
Q

Which of the following cranial foramina pairings are incorrect?

	Carotid canal and internal carotid artery.
	Foramen ovale and mandibular nerve.
	Optic canal and ophthalmic artery.
	Optic canal and ophthalmic nerve.
	Foramen rotundum and maxillary nerve.
A

D)

The optic canal transmits the optic nerve. The ophthalmic nerve traverses the superior orbital fissure.

230
Q

A 22 year old man is involved in a fight and sustains a stab wound in his upper forearm. On examination there is a small, but deep laceration. There is an obvious loss of pincer movement involving the thumb and index finger with minimal loss of sensation. The most likely nerve injury is to the:

	Ulnar nerve
	Radial nerve
	Anterior interosseous nerve
	Axillary nerve
	Median nerve
A

C)

The anterior interosseous nerve is a motor branch of the median nerve just below the elbow. When damaged it classically causes:
Pain in the forearm
Loss of pincer movement of the thumb and index finger (innervates the long flexor muscles of flexor pollicis longus & flexor digitorum profundus of the index and middle finger)
Minimal loss of sensation due to lack of a cutaneous branch

231
Q

A 66 year old man is undergoing a left nephro-ureterectomy. The surgeons remove the ureter, which of the following is responsible for the blood supply to the proximal ureter?

	Branches of the renal artery
	External iliac artery
	Internal iliac artery
	Direct branches from the aorta
	Common iliac artery
A

A)

The proximal ureter is supplied by branches from the renal artery. For the other feeding vessels - see below.

232
Q

Which of the following structures does not pass behind the lateral malleolus?

	Peroneus brevis tendon
	Sural nerve
	Short saphenous vein
	Peroneus longus tendon
	Tibialis anterior tendon
A

E) Tibialis anterior tendon passes anterior to the medial malleolus

Structures posterior to the lateral malleolus and superficial to superior peroneal retinaculum
Sural nerve
Short saphenous vein

Structures posterior to the lateral malleolus and deep to superior peroneal retinaculum
Peroneus longus tendon
Peroneus brevis tendon

The calcaneofibular ligament is attached at the lateral malleolus

233
Q

A 63 year old man is admitted with severe headache, nausea and recent epileptic fit. Fundoscopy shows papilloedema. He is also noted to have diplopia. Which of the cranial nerves listed is responsible?

	Abducens
	Optic
	Oculomotor
	Facial
	Trigeminal
A

A)

The long intracranial course of this nerve makes it susceptible to damage early in the course of raised ICP.

234
Q

A 78 year old man presents with symptoms consistent with intermittent claudication. To assess the severity of his disease you decide to measure his ankle brachial pressure index. To do this you will identify the dorsalis pedis artery. Which of the following statements relating to this vessel is false?

It originates from the peroneal artery
It is crossed by the tendon of extensor hallucis brevis
Two veins are usually closely related to it
It passes under the inferior extensor retinaculum
The tendon of extensor digitorum longus lies lateral to it
A

A)

The dorsalis pedis artery is a direct continuation of the anterior tibial artery.

235
Q

Which of the following is not a content of the anterior triangle of the neck?

	Vagus nerve
	Submandibular gland
	Phrenic nerve
	Internal jugular vein
	Hypoglossal nerve
A

C)

The phrenic nerve is a content of the posterior triangle. The anterior triangle contains the carotid sheath and its contents.

236
Q

A 32 year old attends neurology clinic complaining of tingling in his hand. He has radial deviation of his wrist and there is mild clawing of his fingers, with the 4th and 5th digits being relatively spared. What is the most likely lesion?

	Ulnar nerve damage at the wrist
	Ulnar nerve damage at the elbow
	Radial nerve damage at the elbow
	Median nerve damage at the wrist
	Median nerve damage at the elbow
A

B)

The ulnar paradox- the higher the lesion, the less the clawing of the fingers seen clinically.

At the elbow the ulnar nerve lesion affects the flexor carpi ulnaris and flexor digitorum profundus.

237
Q

A 22 year old man is undergoing an endotracheal intubation. Which of the following vertebral levels is consistent with the origin of the trachea?

	C2
	T1
	C6
	C4
	C3
A

C)

The trachea commences at C6. It terminates at the level of T5 (or T6 in tall subjects in deep inspiration).

238
Q

A young child undergoes a difficult craniotomy for fulminant mastoiditis and associated abscess. During the procedure the trigeminal nerve is severely damaged within Meckels cave. Which deficit is least likely to be present?

Anaesthesia over the ipsilateral anterior aspect of the scalp
Loss of the corneal reflex
Weakness of the ipsilateral masseter muscle
Anaesthesia of the anterior aspect of the lip
Anaesthesia over the entire ipsilateral side of the face
A

E)

The angle of the jaw is not innervated by sensory fibres of the trigeminal nerve and is spared in this type of injury.
Remember the trigeminal nerve provides motor innervation to the muscles of mastication. The close proximity of the site of injury to the motor fibres is likely to result in at least some compromise of motor muscle function.

239
Q

Which of the following is not a branch of the external carotid artery?

	Facial artery
	Lingual artery
	Superior thyroid artery
	Mandibular artery
	Maxillary artery
A

D)

External carotid artery branches mnemonic:

‘Some Angry Lady Figured Out PMS’

Superior thyroid (superior laryngeal artery branch)
Ascending pharyngeal 
Lingual 
Facial (tonsillar and labial artery)
Occipital 
Posterior auricular 
Maxillary (inferior alveolar artery, middle meningeal artery)
Superficial temporal
240
Q

A 23 year old man is stabbed in the groin, several structures are injured and the adductor longus muscle has been lacerated. Which of the following nerves is responsible for the innervation of adductor longus?

	Femoral nerve
	Obturator nerve
	Sciatic nerve
	Common peroneal nerve
	Ilioinguinal nerve
A

B) The adductors are innervated by the obturator nerve

241
Q

Which of the following statements relating to the basilar artery and its branches is false?

The superior cerebellar artery may be decompressed to treat trigeminal neuralgia
Occlusion of the posterior cerebral artery causes contralateral loss of the visual field
The oculomotor nerve lies between the superior cerebellar and posterior cerebral arteries
The posterior inferior cerebellar artery is the largest of the cerebellar arteries arising from the basilar artery
The labyrinthine branch is accompanied by the facial nerve
A

D)

The posterior inferior cerebellar artery is the largest of the cerebellar arteries arising from the vertebral artery. The labyrinthine artery is long and slender and may arise from the lower part of the basilar artery. It accompanies the facial and vestibulocochlear nerves into the internal auditory meatus. The posterior cerebral artery is often larger than the superior cerebellar artery and it is separated from the vessel, near it’s origin, by the oculomotor nerve. Arterial decompression is a well established therapy for trigeminal neuralgia.

242
Q

Which of the following muscles does not recieve any innervation from the sciatic nerve?

	Semimembranosus
	Quadriceps femoris
	Biceps femoris
	Semitendinosus
	Adductor magnus
A

B)

The sciatic nerve is traditionally viewed as being a nerve of the posterior compartment. It is known to contribute to the innervation of adductor magnus (although the main innervation to this muscle is from the obturator nerve). The quadriceps femoris is nearly always innervated by the femoral nerve.

243
Q

A 23 year old man is involved in a fight and is stabbed in his upper arm. The ulnar nerve is transected. Which of the following muscles will not demonstrate compromised function as a result?

	Flexor carpi ulnaris
	Medial half of flexor digitorum profundus
	Palmaris brevis
	Hypothenar muscles
	Pronator teres
A
E) M edial lumbricals
A dductor pollicis
F lexor digitorum profundus/Flexor digiti minimi
I nterossei
A bductor digiti minimi and opponens

Innervates all intrinsic muscles of the hand (EXCEPT 2: thenar muscles & first two lumbricals - supplied by median nerve)

244
Q

Which of the following pairings are incorrect?

Aortic bifurcation and L4
Transpyloric plane and L1
Termination of dural sac and L4
Oesophageal passage through diaphragm and T10
Transition between pharynx and oesophagus at C6
A

C) Vena cava T8 (eight letters)
Oesophagus T10 (ten letters)
Aortic hiatus T12 (twelve letters)

It terminates at S2, which is why it is safe to undertake an LP at L4/5 levels. The spinal cord itself terminates at L1.

245
Q

A 22 year old man is involved in a fight. He sustains a laceration to the posterior aspect of his wrist. In the emergency department the wound is explored and the laceration is found to be transversely orientated and overlies the region of the extensor retinaculum, which is intact. Which of the following structures is least likely to be injured in this scenario?

	Dorsal cutaneous branch of the ulnar nerve
	Tendon of extensor indicis
	Basilic vein
	Superficial branch of the radial nerve
	Cephalic vein
A

B)

The extensor retinaculum attaches to the radius proximal to the styloid, thereafter it runs obliquely and distally to wind around the ulnar styloid (but does not attach to it). The extensor tendons lie deep to the extensor retinaculum and would therefore be less susceptible to injury than the superficial structures.

Structures superficial to the retinaculum
Basilic vein
Dorsal cutaneous branch of the ulnar nerve
Cephalic vein
Superficial branch of the radial nerve

246
Q

Which of the following is not a content of the porta hepatis?

	Portal vein
	Hepatic artery
	Cystic duct
	Lymph nodes
	None of the above
A

C)

The cystic duct lies outside the porta hepatis and is an important landmark in laparoscopic cholecystectomy. The structures in the porta hepatis are:
Portal vein
Hepatic artery
Common hepatic duct

247
Q

A 21 year old develops tonsillitis. He is in considerable pain. Which of the following nerves is responsible for the sensory innervation of the tonsillar fossa?

	Facial nerve
	Trigeminal nerve
	Glossopharyngeal nerve
	Hypoglossal nerve
	Vagus
A

C) The glossopharyngeal nerve is the main sensory nerve for the tonsillar fossa. A lesser contribution is made by the lesser palatine nerve. Because of this otalgia may occur following tonsillectomy.

248
Q

A man has an incision sited that runs 8cm from the deltopectoral groove to the midline. Which of the following is not at risk of injury?

	Cephalic vein
	Shoulder joint capsule
	Axillary artery
	Pectoralis major
	Trunk of the brachial plexus
A

B)
This region will typically lie medial to the joint capsule. The diagram below illustrates the plane that this would transect and as it can be appreciated the other structures are all at risk of injury.

249
Q

A surgeon is due to perform a laparotomy for perforated duodenal ulcer. An upper midline incision is to be performed. Which of the following structures is the incision most likely to divide?

	Rectus abdominis muscle
	External oblique muscle
	Linea alba
	Internal oblique muscle
	None of the above
A

C)

250
Q

A 59 year old man is undergoing an extended right hemicolectomy for a carcinoma of the splenic flexure of the colon. The surgeons divide the middle colic vein close to its origin. Into which of the following structures does this vessel primarily drain?

	Superior mesenteric vein
	Portal vein
	Inferior mesenteric vein
	Inferior vena cava
	Ileocolic vein
A

A) The middle colonic vein drains into the SMV, if avulsed during mobilisation then dramatic haemorrhage can occur and be difficult to control.

251
Q

What is the lymphatic drainage of the female urethra?

	Superficial inguinal nodes
	Deep inguinal nodes
	Internal iliac nodes
	External iliac nodes
	Para-aortic nodes
A

C)

252
Q

A 23 year old man is stabbed in the chest approximately 10cm below the right nipple. In the emergency department a abdominal ultrasound scan shows a large amount of intraperitoneal blood. Which of the following statements relating to the likely site of injury is untrue?

Part of its posterior surface is devoid of peritoneum.
The quadrate lobe is contained within the functional right lobe.
Its nerve supply is from the coeliac plexus.
The hepatic flexure of the colon lies posterio-inferiorly.
The right kidney is closely related posteriorly.
A

B) The right lobe of the liver is the most likely site of injury. Therefore the answer is B as the quadrate lobe is functionally part of the left lobe of the liver. The liver is largely covered in peritoneum. Posteriorly there is an area devoid of peritoneum (the bare area of the liver). The right lobe of the liver has the largest bare area (and is larger than the left lobe).

253
Q

A 22 year old man is involved in a fight and sustains a skull fracture with an injury to the middle meningeal artery. A craniotomy is performed, and with considerable difficulty the haemorrhage from the middle meningeal artery is controlled by ligating it close to its origin. What is the most likely sensory impairment that the patient may notice post operatively?

Parasthesia of the ipsilateral external ear
Loss of taste sensation from the anterior two thirds of the tongue
Parasthesia overlying the angle of the jaw
Loss of sensation from the ipsilateral side of the tongue
Loss of taste from the posterior two thirds of the tongue
A

A) The auriculotemporal nerve is closely related to the middle meningeal artery and may be damaged in this scenario. The nerve supplied sensation to the external ear and outermost part of the tympanic membrane. The angle of the jaw is innervated by C2,3 roots and would not be affected. The tongue is supplied by the glossopharyngeal nerve.

254
Q

A 72 year old man presents with haemoptysis and undergoes a bronchoscopy. The carina is noted to be widened. At which level does the trachea bifurcate?

	T3
	T5
	T7
	T2
	T8
A

B)

The trachea bifurcates at the level of the fifth thoracic vertebra. Or the sixth in tall subjects.

255
Q

A 23 year old man is injured during a game of rugby. He suffers a fracture of the distal third of his clavicle, it is a compound fracture and there is evidence of arterial haemorrhage. Which of the following vessels is most likely to be encountered first during subsequent surgical exploration?

	Posterior circumflex humeral artery
	Axillary artery
	Thoracoacromial artery
	Sub scapular artery
	Lateral thoracic artery
A

C) The thoracoacromial artery arises from the second part of the axillary artery. It is a short, wide trunk, which pierces the clavipectoral fascia, and ends, deep to pectoralis major by dividing into four branches.

256
Q

The following are true of the femoral nerve, except:

It is derived from L2, L3 and L4 nerve roots
It supplies sartorius
It supplies quadriceps femoris
It gives cutaneous innervations via the saphenous nerve
It supplies adductor longus
A

E) Adductor longus is supplied by the obturator nerve.

257
Q

Where is the vomiting centre located?

	Medulla oblongata
	Substantia nigra
	Antrum of stomach
	Pons
	Midbrain
A

A)

258
Q

Which of the following nerves conveys sensory information from the laryngeal mucosa?

	Glossopharyngeal
	Laryngeal branches of the vagus
	Ansa cervicalis
	Laryngeal branches of the trigeminal
	None of the above
A

B) The laryngeal branches of the vagus supply sensory information from the larynx.

259
Q

Which of the following nerves passes through the greater sciatic foramen and innervates the perineum?

	Pudendal
	Sciatic
	Superior gluteal
	Inferior gluteal
	Posterior cutaneous nerve of the thigh
A
A) 3 divisions of the pudendal nerve:
Rectal nerve
Perineal nerve
Dorsal nerve of penis/ clitoris
All these pass through the greater sciatic foramen.

The pudendal nerve innervates the perineum. It passes between piriformis and coccygeus medial to the sciatic nerve.

260
Q

Which of the following is true in relation to the sartorius muscle?

Innervated by the deep branch of the femoral nerve
Inserts at the fibula
It is the shortest muscle in the body
Forms the Pes anserinus with Gracilis and semitendinous muscle
Causes extension of the knee
A

D)

It is innervated by the superficial branch of the femoral nerve. It is a component of the pes anserinus.

261
Q

A 25 year old man sustains a severe middle cranial fossa basal skull fracture. Once he has recovered it is noticed that he has impaired tear secretion. This is most likely to be the result of damage to which of the following?

	Stellate ganglion
	Ciliary ganglion
	Otic ganglion
	Trigeminal nerve
	Greater petrosal nerve
A

E) The greater petrosal nerve may be injured and carries fibres for lacrimation

262
Q

Which of the following structures passes through the quadrangular space near the humeral head?

	Axillary artery
	Radial nerve
	Axillary nerve
	Median nerve
	Transverse scapular artery
A

C)

The quadrangular space is bordered by the humerus laterally, subscapularis superiorly, teres major inferiorly and the long head of triceps medially. It lies lateral to the triangular space. It transmits the axillary nerve and posterior circumflex humeral artery.

263
Q

Which of the following pairings of foramina and their contents is not correct?

Superior orbital fissure and the oculomotor nerve
Foramina rotundum and the maxillary nerve
Jugular foramen and the hypoglossal nerve
Foramina spinosum and the middle meningeal artery
Carotid canal and the internal carotid artery
A

C) The hypoglossal nerve passes through the hypoglossal canal.

264
Q

A 55 year old man with carcinoma of the larynx is undergoing a difficult laryngectomy. The surgeons divide the thyrocervical trunk, from which of the following vessels does this structure most commonly originate?

	Subclavian artery
	Common carotid artery
	Vertebral artery
	External carotid artery
	Internal carotid artery
A

A) The thyrocervical trunk is a branch of the subclavian artery. It arises from the first part between the subclavian artery and the inner border of scalenus anterior. It branches off the subclavian distal to the vertebral artery.

265
Q

What is the first branch of the axillary artery?

	Subscapular artery
	Lateral thoracic artery
	Thoraco acromial artery
	Superior thoracic artery
	Anterior circumflex humeral artery
A

D)

The superior thoracic artery is the first branch of the axillary artery arises from the first part
Two branches arise from the second part, thoraco acromial and lateral thoracic
Three branches from the third part, subscapular artery, anterior and posterior circumflex humeral arteries

266
Q

The following structures are closely related to the brachiocephalic artery except:

	Trachea posteriorly
	Right brachiocephalic vein
	Inferior thyroid vein
	Right recurrent laryngeal nerve
	None of the above
A

D)

There is no brachiocephalic artery on the left, however the left brachiocephalic vein lies anteriorly to the roots of all the 3 great arteries (including the brachiocephalic artery). The right recurrent laryngeal nerve has no relation to the brachiocephalic artery.

267
Q

Which of the following structures separates the ulnar artery from the median nerve?

	Brachioradialis
	Pronator teres
	Tendon of biceps brachii
	Flexor carpi ulnaris
	Brachialis
A

B)

It lies deep to pronator teres and this separates it from the median nerve.

268
Q

Which muscle is supplied by the superficial peroneal nerve?

	Peroneus tertius
	Sartorius
	Adductor magnus
	Peroneus brevis
	Gracilis
A

D)

269
Q

A man develops an infection in his external ear. The infection is extremely painful. Which of the following nerves conveys sensation from this region?

Occipital branch of the trigeminal nerve
Vestibulocochlear nerve
Facial nerve
Auriculotemporal nerve
Maxillary branch of the trigeminal nerve
A

D) Tensor tympania and stapedius are the only two muscles of the middle ear. Contraction of tensor tympani will tend to dampen the vibrations produced by loud sounds, it is innervated by a branch of the trigeminal nerve. The stapedius dampens movements of the ossicles in response to loud sounds and is innervated by a branch of the facial nerve.

The auriculotemporal nerve, which is derived from the mandibular branch of the trigeminal nerve supplies this area.

270
Q

Which muscle is responsible for causing flexion of the interphalangeal joint of the thumb?

	Flexor pollicis longus
	Flexor pollicis brevis
	Flexor digitorum superficialis
	Flexor digitorum profundus
	Adductor pollicis
A

A) There are 8 muscles:

  1. Two flexors (flexor pollicis brevis and flexor pollicis longus)
  2. Two extensors (extensor pollicis brevis and longus)
  3. Two abductors (abductor pollicis brevis and longus)
  4. One adductor (adductor pollicis)
  5. One muscle that opposes the thumb by rotating the CMC joint (opponens pollicis).

Flexor and extensor longus insert on the distal phalanx moving both the MCP and IP joints.

271
Q

Which of the following structures separates the posterior cruciate ligament from the popliteal artery?

	Oblique popliteal ligament
	Transverse ligament
	Popliteus tendon
	Biceps femoris
	Semitendinosus
A

A)
The posterior cruciate ligament is separated from the popliteal vessels at its origin by the oblique popliteal ligament. The transverse ligament is located anteriorly.

272
Q

How many compartments are there in the lower leg?

	2
	1
	3
	5
	4
A

E)
The posterior compartment of the lower leg has both superficial and deep posterior layers, together with the anterior and lateral compartments this allows for four compartments. Decompression of the deep posterior compartment during fasciotomy may be overlooked with significant sequelae.

273
Q

Which structure is least likely to be found at the level of the sternal angle?

	Left brachiocephalic vein
	Intervertebral discs T4-T5
	Start of aortic arch
	2nd pair of costal cartilages
	Bifurcation of the trachea into left and right bronchi
A

A)
The left brachiocephalic vein lies posterior to the manubrium, at the level of its upper border. The sternal angle refers to the transition between manubrium and sternum and therefore will not include the left brachiocephalic vein.

274
Q

What is the most useful test to clinically distinguish between an upper and lower motor neurone lesion of the facial nerve?

	Blow cheeks out
	Loss of chin reflex
	Close eye
	Raise eyebrow
	Open mouth against resistance
A

D)

Upper motor neurone lesions of the facial nerve- Paralysis of the lower half of face.
Lower motor neurone lesion- Paralysis of the entire ipsilateral face.

275
Q

An 18 year old man is stabbed in the axilla during a fight. His axillary artery is lacerated and repaired. However, the surgeon neglects to repair an associated injury to the upper trunk of the brachial plexus. Which of the following muscles is least likely to demonstrate impaired function as a result?

	Palmar interossei
	Infraspinatus
	Brachialis
	Supinator brevis
	None of the above
A

A)

The palmar interossei are supplied by the ulnar nerve. Which lies inferiorly and is therefore less likely to be injured.

276
Q

A 23 year old man is involved in a fight, during the dispute he sustains a laceration to the posterior aspect of his right arm, approximately 2cm proximal to the olecranon process. On assessment in the emergency department he is unable to extend his elbow joint. Which of the following tendons is most likely to have been cut?

	Triceps
	Pronator teres
	Brachioradialis
	Brachialis
	Biceps
A

A) The triceps muscle extends the elbow joint. The other muscles listed all produce flexion of the elbow joint.

277
Q

You excitedly embark on your first laparoscopic cholecystectomy and during the operation the anatomy of Calots triangle is more hostile than anticipated. Whilst trying to apply a haemostatic clip you avulse the cystic artery. This is followed by brisk haemorrhage. From which source is this most likely to originate ?

	Right hepatic artery
	Portal vein
	Gastroduodenal artery
	Liver bed
	Common hepatic artery
A

A) The cystic artery is a branch of the right hepatic artery.

278
Q

A 43 year old man suffers a pelvic fracture which is complicated by an injury to the junction of the membranous urethra to the bulbar urethra. In which of the following directions is the extravasated urine most likely to pass?

Posteriorly into extra peritoneal tissues
Laterally into the buttocks
Into the abdomen
Anteriorly into the connective tissues surrounding the scrotum
None of the above
A

D) The superficial perineal pouch is a compartment bounded superficially by the superficial perineal fascia, deep by the perineal membrane (inferior fascia of the urogenital diaphragm), and laterally by the ischiopubic ramus. It contains the crura of the penis or clitoris, muscles, viscera, blood vessels, nerves, the proximal part of the spongy urethra in males, and the greater vestibular glands in females.
When urethral rupture occurs as in this case the urine will tend to pass anteriorly because the fascial condensations will prevent lateral and posterior passage of the urine.

279
Q

Which of the following does not pass through the superior orbital fissure?

	Oculomotor nerve
	Abducens nerve
	Ophthalmic artery
	Ophthalmic division of the trigeminal nerve
	Ophthalmic veins
A

C)

The ophthalmic artery, a branch of the internal carotid enters the orbit with the optic nerve in the canal.

280
Q

Which nerve supplies the 1st web space of the foot?

	Popliteal nerve
	Superficial peroneal nerve
	Deep peroneal nerve
	Tibial nerve
	Saphenous nerve
A

C) The first web space is innervated by the deep peroneal nerve.

281
Q

During the course of a radical gastrectomy the surgeons detach the omentum and ligate the right gastro-epiploic artery. From which vessel does it originate?

	Superior mesenteric artery
	Inferior mesenteric artery
	Coeliac axis
	Common hepatic artery
	Gastroduodenal artery
A

E)

282
Q

Which of the following is not an intrinsic muscle of the hand?

	Opponens pollicis
	Palmaris longus
	Flexor pollicis brevis
	Flexor digiti minimi brevis
	Opponens digiti minimi
A

B) Palmaris longus originates in the forearm.

283
Q

Through which of the following foramina does the genital branch of the genitofemoral nerve exit the abdominal cavity?

	Superficial inguinal ring
	Sciatic notch
	Obturator foramen
	Femoral canal
	Deep inguinal ring
A

E) The genitofemoral nerve divides into two branches as it approaches the inguinal ligament. The genital branch passes anterior to the external iliac artery through the deep inguinal ring into the inguinal canal. It communicates with the ilioinguinal nerve in the inguinal canal (though this is seldom of clinical significance).

284
Q

A 63 year old lady is diagnosed as having an endometrial carcinoma arising from the uterine body. To which nodal region will the tumour initially metastasise?

	Para aortic nodes
	Iliac lymph nodes
	Inguinal nodes
	Pre sacral nodes
	Mesorectal lymph nodes
A

B) Tumours of the uterine body will tend to spread to the iliac nodes initially.

Lymphatic drainage of the ovaries, uterus and cervix

The ovaries drain to the para-aortic lymphatics via the gonadal vessels.
The uterine fundus has a lymphatic drainage that runs with the ovarian vessels and may thus drain to the para-aortic nodes. Some drainage may also pass along the round ligament to the inguinal nodes.
The body of the uterus drains through lymphatics contained within the broad ligament to the iliac lymph nodes.
The cervix drains into three potential nodal stations; laterally through the broad ligament to the external iliac nodes, along the lymphatics of the uterosacral fold to the presacral nodes and posterolaterally along lymphatics lying alongside the uterine vessels to the internal iliac nodes.

285
Q

Which of the following structures is not located in the superficial perineal space in females?

	Posterior labial arteries
	Pudendal nerve
	Superficial transverse perineal muscle
	Greater vestibular glands
	None of the above
A

B)
The pudendal nerve is located in the deep perineal space and then branches to innervate more superficial structures.

Urogenital triangle

The urogenital triangle is formed by the:
Ischiopubic inferior rami
Ischial tuberosities
A fascial sheet is attached to the sides, forming the inferior fascia of the urogenital diaphragm.

It transmits the urethra in males and both the urethra and vagina in females. The membranous urethra lies deep to this structure and is surrounded by the external urethral sphincter.

Superficial to the urogenital diaphragm lies the superficial perineal pouch. In males this contains:
Bulb of penis
Crura of the penis
Superficial transverse perineal muscle
Posterior scrotal arteries
Posterior scrotal nerves

In females the internal pudendal artery branches to become the posterior labial arteries in the superficial perineal pouch.

286
Q

Which of the following is not a branch of the hepatic artery?

	Pancreatic artery
	Cystic artery
	Right gastric artery
	Right hepatic artery
	Gastroduodenal artery
A

A)

The pancreatic artery is a branch of the splenic artery.

287
Q

Which of the following structures does not pass close to the piriformis muscle in the greater sciatic foramen?

	Sciatic nerve
	Posterior cutaneous nerve of the thigh
	Inferior gluteal artery
	Obturator nerve
	None of the above
A

D)

Contents

Nerves	
Sciatic Nerve
Superior and Inferior Gluteal Nerves
Pudendal Nerve
Posterior Femoral Cutaneous Nerve
Nerve to Quadratus Femoris
Nerve to Obturator internus

Vessels
Superior Gluteal Artery and vein
Inferior Gluteal Artery and vein
Internal Pudendal Artery and vein

288
Q

A 56 year old man is undergoing a right nephrectomy. The surgeons divide the renal artery. At what level does this usually branch off the abdominal aorta?

	T9
	L2
	L3
	T10
	L4
A

B)

The renal arteries usually branch off the aorta on a level with L2.

289
Q

A 23 year old man is shot in the chest during a robbery. The left lung is lacerated and is bleeding. An emergency thoracotomy is performed. The surgeons place a clamp over the hilum of the left lung. Which of the following structures lies most anteriorly at this level?

	Vagus nerve
	Oesophagus
	Descending aorta
	Phrenic nerve
	Azygos vein
A

D) The phrenic nerve lies anteriorly at this point. The vagus passes anteriorly and then arches backwards immediately superior to the root of the left bronchus, giving off the recurrent laryngeal nerve as it does so.

290
Q

Which of the following muscles does not adduct the shoulder?

	Teres major
	Pectoralis major
	Coracobrachialis
	Supraspinatus
	Latissimus dorsi
A

D)

Supraspinatus is an abductor of the shoulder.

291
Q

Which of these muscles is innervated by the cervical branch of the facial nerve?

	Masseter
	Sternocleidomastoid
	Platysma
	Geniohyoid
	Sternothyroid
A

C) The cervical branch of the facial nerve innervates platysma.

292
Q

A 56 year old man is left impotent following an abdomino-perineal excision of the colon and rectum. What is the most likely explanation?

Psychosexual issues related to an end colostomy
Damage to the sacral venous plexus during total mesorectal excision
Damage to the left ureter during sigmoid mobilisation
Damage to the hypogastric plexus during mobilisation of the inferior mesenteric artery
Damage to the internal iliac artery during total mesorectal excision
A

D) Autonomic nerve injury is the most common cause.

293
Q

A 73 year old man is due to undergo a radical prostatectomy for carcinoma of the prostate gland. To which of the following lymph nodes will the tumour drain primarily?

	Para aortic
	Internal iliac
	Superficial inguinal
	Meso rectal
	None of the above
A

B) The prostate lymphatic drainage is primarily to the internal iliac nodes and also the sacral nodes. Although internal iliac is the first site.

294
Q

Which of the following statements relating to the vertebral column is false?

There are 7 cervical vertebrae
The cervical and lumbar lordosis are secondary curves developing after birth due to change in shape of the intervertebral discs
The lumbar vertebrae do not have a transverse process foramina
The lumbar vertebrae receive blood directly from the aorta
The spinous process is formed by the junction of the pedicles posteriorly
A

D) The spinous process is formed by 2 laminae posteriorly.

295
Q

A 78 year old lady falls over in her nursing home and sustains a displaced intracapsular fracture of the femoral neck. A decision is made to perform a hemi arthroplasty through a lateral approach. Which of these vessels will be divided to facilitate access?

	Saphenous vein
	Superior gluteal artery
	Superficial circumflex iliac artery
	Profunda femoris artery
	Transverse branch of the lateral circumflex artery
A

E) During the Hardinge style lateral approach the transverse branch of the lateral circumflex artery is divided to gain access.

296
Q

A 73 year old man undergoes a sub total oesophagectomy with anastomosis of the stomach to the cervical oesophagus. Which vessel will be primarily responsible for the arterial supply to the oesophageal portion of the anastomosis?

	Superior thyroid artery
	Internal carotid artery
	Direct branches from the thoracic aorta
	Inferior thyroid artery
	Subclavian artery
A

D) The cervical oesophagus is supplied by the inferior thyroid artery. The thoracic oesophagus (removed in this case) is supplied by direct branches from the thoracic aorta.

297
Q

Which of the following structures is not closely related to the brachial artery?

	Ulnar nerve
	Median nerve
	Cephalic vein
	Long head of triceps
	Median cubital vein
A

C) The cephalic vein lies superficially and on the contralateral side of the arm to the brachial artery.

298
Q

A 38 year old man presents to the clinic with shoulder weakness. On examination he has an inability to initiate shoulder abduction. Which of the nerves listed below is least likely to be functioning normally?

	Suprascapular nerve
	Medial pectoral nerve
	Axillary nerve
	Median nerve
	Radial nerve
A

A)

299
Q

Which of the following statements relating to the Cavernous Sinus is false?

The pituitary gland lies medially
The internal carotid artery passes through it
The temporal lobe of the brain is a lateral relation
The mandibular branch of the trigeminal and optic nerve lie on the lateral wall
The ophthalmic veins drain into the anterior aspect of the sinus
A

D) The veins that drain into the sinus are important as sepsis can cause cavernous sinus thrombosis. The maxillary branch of the trigeminal and not the mandibular branches pass through the sinus

300
Q

Which of the following is not a branch of the subclavian artery?

	Superior thyroid artery
	Vertebral artery
	Thyrocervical trunk
	Internal thoracic artery
	Dorsal scapular artery
A

A) Mnemonic for the branches of the subclavian artery: VIT C & D

V ertebral artery
I nternal thoracic
T hyrocervical trunk

C ostalcervical trunk
D orsal scapular

Superior thyroid artery is a branch of the external carotid artery.

301
Q

During the repair of an atrial septal defect the surgeons note that blood starts to leak from the coronary sinus. Which structure forms the largest tributary of the coronary sinus?

	Thebesian veins
	Great cardiac vein
	Oblique vein
	Small cardiac veins
	None of the above
A

B) The great cardiac vein runs in the anterior interventricular groove, and is the largest tributary of the coronary sinus. The thebesian veins drain into the heart directly.

Coronary sinus
This lies in the posterior part of the coronary groove and receives blood from the cardiac veins. The great cardiac vein lies at its left and the middle and small cardiac veins lie on its right. The smallest cardiac vein (anterior cardiac vein) drains into the right atrium directly.

302
Q

Which of the following vessels provides the greatest contribution to the arterial supply of the breast?

	External mammary artery
	Thoracoacromial artery
	Internal mammary artery
	Lateral thoracic artery
	Subclavian artery
A

C) 60% of the arterial supply to the breast is derived from the internal mammary artery. The external mammary and lateral thoracic arteries also make a significant (but lesser) contribution. This is of importance clinically in performing reduction mammoplasty procedures.

303
Q

Which of the following muscles is supplied by the external laryngeal nerve?

	Transverse arytenoid
	Cricothyroid
	Thyro-arytenoid
	Posterior crico-arytenoid
	Oblique arytenoid
A

B) The others are all supplied by the recurrent laryngeal nerve.

304
Q

A 28 year old man has sustained a non salvageable testicular injury to his left testicle. The surgeon decides to perform an orchidectomy and divides the left testicular artery. From which of the following does this vessel originate?

	Abdominal aorta
	Internal iliac artery
	Inferior epigastric artery
	Inferior vesical artery
	External iliac artery
A

A) The testicular artery is a branch of the abdominal aorta.

305
Q

During a carotid endarterectomy the internal carotid artery is cross clamped. Assuming that no shunt is inserted, which of the following vessels will not have diminished or absent flow as a result?

	Anterior cerebral artery
	Ophthalmic artery
	Middle cerebral artery
	Maxillary artery
	None of the above
A

D) Mnemonic for branches of the cerebral portion of the internal carotid artery ‘Only Press Carotid Arteries Momentarily’

Only = Opthalmic
Press = Posterior communicating
Carotid = Choroidal
Arteries = Anterior cerebral
Momentarily = Middle cerebral

The maxillary artery is a branch of the external carotid artery.

306
Q

A 72 year old lady with osteoporosis falls and sustains an intracapsular femoral neck fracture. The fracture is completely displaced. Which of the following vessels is the main contributor to the arterial supply of the femoral head?

	Deep external pudendal artery
	Superficial femoral artery
	External iliac artery
	Circumflex femoral arteries
	Superficial external pudendal artery
A

D) The vessels which form the anastomoses around the femoral head are derived from the medial and lateral circumflex femoral arteries. These are usually derived from the profunda femoris artery.

307
Q

A 21 year old man is hit with a hammer and sustains a depressed skull fracture at the vertex. Which of the following sinuses is at risk in this injury?

	Superior sagittal sinus
	Inferior petrosal sinus
	Transverse sinus
	Inferior sagittal sinus
	Straight sinus
A

A) The superior sagittal sinus is at greatest risk in this pattern of injury. This sinus begins at the front of the crista galli and courses backwards along the falx cerebri. It becomes continuous with the right transverse sinus near the internal occipital protuberance.

308
Q

A 44 year old man is stabbed in the back and the left kidney is injured. A haematoma forms, which of the following fascial structures will contain the haematoma?

	Waldeyers fascia
	Sibsons fascia
	Bucks fascia
	Gerotas fascia
	Denonvilliers fascia
A

D) Waldeyers fascia- Posterior ano-rectum
Sibsons fascia- Lung apex
Bucks fascia- Base of penis
Gerotas fascia- Surrounding kidney
Denonvilliers fascia- Between rectum and prostate

309
Q

A baby is found to have a Klumpke’s palsy post delivery. Which of the following is most likely to be present?

	Loss of flexors of the wrist
	Weak elbow flexion
	Pronation of the forearm
	Adducted shoulder
	Shoulder medially rotated
A

A) Features of Klumpkes Paralysis
Claw hand (MCP joints extended and IP joints flexed)
Loss of sensation over medial aspect of forearm and hand
Horner’s syndrome
Loss of flexors of the wrist

A C8, T1 root lesion is called Klumpke’s paralysis and is caused by delivery with the arm extended.

310
Q

A 22 year old man undergoes a superficial parotidectomy for a pleomorphic adenoma. The operation does not proceed well and a diathermy malfunction results in division of the buccal branch of the facial nerve. Which of the following muscles will not demonstrate impaired function as a result?

	Zygomaticus minor
	Mentalis
	Buccinator
	Levator anguli oris
	Risorius
A

B)

311
Q

At which of the following vertebral body levels does the common carotid artery typically bifurcate into the external and internal carotid arteries?

	C4
	C2
	C1
	C6
	C7
A

A) It terminates at the upper border of the thyroid cartilage, Which is usually located at C4.

312
Q

A man is stabbed in the chest to the right of the manubriosternal angle. Which structure is least likely to be injured in this case?

	Right pleura
	The trachea
	Right phrenic nerve
	Right recurrent laryngeal nerve
	Brachiocephalic vein
A

D) The right recurrent laryngeal nerve branches off the right vagus more proximally and arches posteriorly round the subclavian artery. So of the structures given it is the least likely to be injured.

313
Q

An 18 year old man is stabbed in the neck and has to undergo repair of a laceration to the internal carotid artery. Post operatively he is noted to have a Horners syndrome. Which of the following will not be present?

Apparent enopthalmos
Loss of sweating on the entire ipsilateral side of the face
Constricted pupil
Mild ptosis
Normal sympathetic activity in the torso
A

B)? The anhidrosis will be mild as this is a distal lesion and at worst only a very limited area of the ipsilateral face will be anhidrotic.

314
Q

Which of the following types of epithelium lines the lumenal surface of the normal oesophagus?

Non keratinised stratified squamous epithelium
Ciliated columnar epithelium
Keratinised stratified squamous epithelium
Non ciliated columnar epithelium
None of the above
A

A) The oesphagus is lined by non keratinised stratified squamous epithelium. Changes to glandular type epithelium occur as part of metaplastic processes in reflux.

315
Q

A 23 year old man is stabbed in the neck, in the region between the omohyoid and digastric muscles, the injury is explored surgically. At operation a nerve injury is identified immediately superior to the lingual artery as is branches off the external carotid artery. Which of the following is the most likely result of this injury?

Paralysis of the ipsilateral side of the tongue
Abduction of the ipsilateral vocal cord
Winging of the scapula
Paralysis of the ipsilateral hemi diaphragm
Inability to abduct the shoulder
A

A)
The hypoglossal nerve runs anterior to the external carotid, above the lingual arterial branch. If damaged then ipsilateral paralysis of the genioglossus, hyoglossus and styloglossus muscles will occur. If the patient is asked to protrude their tongue then it will tend to point to the affected side.

316
Q

Which of the following structures is not directly related to the right adrenal gland?

	Diaphragm posteriorly
	Kidney inferiorly
	Right renal vein
	Inferior vena cava
	Hepato-renal pouch
A

C)

The right renal vein is very short and lies more inferiorly.

317
Q

Which of the following aortic branches leaves the aorta approximately 1cm below the coeliac axis?

	Renal artery
	Inferior mesenteric artery
	Superior mesenteric artery
	Lumbar artery
	Gonadal artery
A

C) The SMA leaves the aorta approximately 1cm below the coeliac axis. This is usually a level of L1. It is crossed anteriorly by the splenic vein and the body of the pancreas. It runs downwards and forwards anterior to the uncinate process.

318
Q

Through which of the structures listed below does the axillary nerve pass?

	Quadrangular space
	Triangular space
	Subclavicular space
	Sub pectoral space
	Intercostal space
A

A)

319
Q

The oesophagus is constricted at the following levels apart from:

	Cricoid cartilage
	Arch of the aorta
	Lower oesophageal sphincter
	Left main stem bronchus
	Diaphragmatic hiatus
A
C) Constrictions of the oesophagus : ABCD 
A- Arch of the Aorta 
B- Left main Bronchus 
C- Cricoid Cartilage 
D- Diaphragmatic Hiatus

The oesophagus is not constricted at the level of the lower oesophageal sphincter.

320
Q

A 19 year old man is playing rugby when he suddenly notices a severe pain at the posterolateral aspect of his right thigh. Which of the following muscle groups is most likely to have been injured?

	Semimembranosus
	Semitendinosus
	Long head of biceps femoris
	Gastrocnemius
	Soleus
A

C) The biceps femoris is the laterally located hamstring muscle. The semitendinosus and semimembranosus are located medially. Rupture of gastrocnemius and soleus may occur but is less common.

321
Q

Which of the following is a branch of the third part of the axillary artery?

	Superior thoracic
	Lateral thoracic
	Dorsal scapular
	Thoracoacromial
	Posterior circumflex humeral
A

E) The other branches include:
Subscapular
Anterior circumflex humeral

322
Q

Which of the following structures separates the intervertebral disks from the spinal cord?

	Anterior longitudinal ligament
	Posterior longitudinal ligament
	Supraspinous ligament
	Interspinous ligament
	Ligamentum flavum
A

B) The posterior longitudinal ligament overlies the posterior aspect of the vertebral bodies. It also overlies the posterior aspect of the intervertebral disks.

323
Q

At what level does the aorta bifurcate into the left and right common iliac arteries?

	L1
	L2
	L3
	L4
	L5
A

D) The aorta typically bifurcates at L4. This level is usually fairly constant and is often tested in the exam.

324
Q

A 23 year old man is due to undergo a mitral valve repair for mitral regurgitation. Which of the following is a feature of the mitral valve?

Its closure is marked by the first heart sound
It has two anterior cusps
The chordae tendinae attach to the anterior cusps only
The chordae tendinae anchor the valve directly to the wall of the left ventricle
It is best auscultated in the left third interspace
A

A) The mitral valve is best auscultated over the cardiac apex, where its closure marks the first heart sound. It has only two cusps. These are attached to chordae tendinae which themselves are linked to the wall of the ventricle by the papillary muscles.

325
Q

A 23 year old lady with sialolithiasis of the submandibular gland is undergoing excision of the gland. Which of the following nerves is at risk as the duct is mobilised?

	Lingual nerve
	Buccal nerve
	Facial nerve
	Glossopharyngeal
	Vagus
A

A) The lingual nerve wraps around Whartons duct. The lingual nerve provides sensory supply to the anterior 2/3 of the tongue.

326
Q

Which of the following is true in connection with the phrenic nerves?

They both lie anterior to the hilum of the lungs
They are derived from spinal roots C 2,3,4
They pierce the diaphragm at the level of T7
They consist of motor fibres only
None of the above
A

A) C3,4,5
Keeps the diaphragm alive!

They both lie anterior to the hilum of the lung. The phrenic nerves have both motor and sensory functions. For this reason sub diaphragmatic pathology may cause referred pain to the shoulder.

327
Q

A 32 year old man presents with an inguinal hernia and undergoes an open surgical repair. The surgeons decide to place a mesh on the posterior wall of the inguinal canal to complete the repair, which of the following structures will lie posterior to the mesh?

	Transversalis fascia
	External oblique
	Rectus abdominis
	Obturator nerve
	None of the above
A

A) Inguinal canal walls: ‘MALT: 2M, 2A, 2L, 2T’:

Starting from superior, moving around in order to posterior:
Superior wall (roof): 2 Muscles:Internal oblique, transversus abdominis 
Anterior wall: 2 Aponeuroses: Aponeurosis of external oblique, Aponeurosis of internal oblique 
Lower wall (floor): 2 Ligaments: Inguinal Ligament, Lacunar Ligament Posterior wall: 2Ts: Transversalis fascia, Conjoint Tendon
328
Q

A 22 year old man is involved in a fight and is stabbed in the posterior aspect of his right leg. The knife passes into the popliteal fossa. He sustains an injury to his tibial nerve. Which muscle is least likely to be compromised as a result?

	Tibialis posterior
	Flexor hallucis longus
	Flexor digitorum brevis
	Soleus
	Peroneus tertius
A

E) Peroneus tertius is innervated by the deep peroneal nerve.

329
Q

Which of the following overlies the outer muscular layer of the intrathoracic oesophagus?

	Serosa
	Meissners plexus
	Auerbach's plexus
	Loose connective tissue
	None of the above
A

D) The oesophagus has no serosal covering and hence holds sutures poorly. The Auerbach’s and Meissner’s nerve plexuses lie in between the longitudinal and circular muscle layers and submucosally. The sub mucosal location of the Meissner’s nerve plexus facilitates its sensory role.

330
Q

Which nerve lies medially on the thyroid gland, in the groove between the oesophagus and trachea?

	Vagus nerve
	External laryngeal nerve
	Recurrent laryngeal nerve
	Ansa cervicalis
	Phrenic nerve
A

C) The recurrent laryngeal nerve may be injured at this site during ligation of the inferior thyroid artery.

331
Q

Which of the following nerve roots contribute nerve fibres to the ansa cervicalis?

	C1 only
	C1, C2 and C3
	C2, C3 and C6
	C2, C4 and C5
	C4, C5 and C6
A

B)

332
Q

The anterior interosseous nerve is a branch of which of the following?

	Ulnar nerve
	Superficial branch of the radial nerve
	Axillary nerve
	Deep branch of the radial nerve
	Median nerve
A

E)

333
Q

At which of the following anatomical locations does the common peroneal nerve bifurcate into the superficial and deep peroneal nerves?

Immediately anterior to the linea aspera
At the lateral aspect of the neck of the fibula
Within the substance of tibialis anterior muscle
At the inferomedial aspect of the popliteal fossa
Under the medial head of gastrocnemius
A

B)

334
Q

A 48 year old motor cyclist sustains a complex lower limb fracture in a motor accident. For a time the popliteal artery is occluded and eventually repaired. Subsequently he develops a compartment syndrome and the anterior and superficial posterior compartments of the lower leg are decompressed. Unfortunately, the operating surgeon neglects to decompress the deep posterior compartment. Which of the following muscles is least likely to be affected as a result?

	Flexor digitorum longus
	Plantaris
	Tibialis posterior
	Flexor hallucis longus
	None of the above
A
B) Muscles of the deep posterior compartment:
Tibialis posterior
Flexor hallucis longus
Flexor digitorum longus
Popliteus

The plantaris muscle lies within the superficial posterior compartment of the lower leg.

335
Q

Considering the pituitary gland, which of the following is false?

The anterior pituitary secretes thyroid stimulating hormone
The anterior pituitary develops from Rathkes pouch
Patients with craniopharyngioma may develop bi temporal hemianopia
The pituitary is in direct contact with the optic chiasm
The posterior pituitary secretes oxytocin via a positive feedback loop
A

D) Although the optic chiasm is closely related to the pituitary, and craniopharyngiomas may compress this structure leading to bitemporal hemianopia, it is separated from the chiasm itself by a dural fold.

336
Q

A 24 year old man is involved in a fight and his face is cut with a knife. The wound lies immediately anterior to the tragus of the ear and extends anteriorly. The wound is surgically explored and the laceration is found to be mainly superficial. It extends slightly more deeply immediately inferior to the main trunk of the facial nerve. Bleeding is observed, from which of the following is it most likely to originate?

	External carotid artery
	Retromandibular vein
	Occipital artery
	Maxillary artery
	Ascending pharyngeal artery
A

B) The retromandibular vein lies slightly more deeply than the facial nerve in the parotid gland. It is formed from the maxillary and superficial temporal vein.

337
Q

A 22 year old man is involved in a fight outside a nightclub. He is stabbed in the back, on the left side, approximately 3cm below the 12th rib in the mid scapular line. The structure most likely to be injured first as a result is the:

	Spleen
	Left kidney
	Left adrenal gland
	Left ureter
	None of the above
A

B) The left kidney lies in this location and is the most likely structure to be injured. The Spleen lies more superiorly, and the left adrenal and ureter are unlikely to be injured in isolation.

338
Q

A 28 year old man has a pleomorphic adenoma and the decision is made to resect this surgically. Which of the following structures is least likely to be encountered during surgical resection of the parotid gland?

	External carotid artery
	Retromandibular vein
	Auriculotemporal nerve
	Mandibular nerve
	Zygomatic branch of the facial nerve
A

D) Structures passing through the parotid gland
Facial nerve and branches
External carotid artery (and its branches; the maxillary and superficial temporal)
Retromandibular vein
Auriculotemporal nerve

The mandibular nerve is well separated from the parotid gland.
The maxillary vein joins to the superficial temporal vein and they form the retromandibular vein which then runs through the parotid gland.
The auriculotemporal nerve runs through the gland. Following a parotidectomy this nerve may be damaged and during neuronal regrowth may then attach to sweat glands in this region. This can then cause gustatory sweating (Freys Syndrome).
The facial nerve branch is the marginal mandibular branch and this is related to the gland.

339
Q

A 23 year old man is undergoing a hernia repair and the mesh is to be sutured to the inguinal ligament. From which of the following does the inguinal ligament arise?

	Transversus abdominis fascia
	Internal oblique
	Rectus sheath
	Rectus abdominis muscle
	External oblique aponeurosis
A

E) The inguinal ligament is formed by the external oblique aponeurosis. It runs from the pubic tubercle to the anterior superior iliac spine.

340
Q

A 56 year old man is undergoing a carotid endarterectomy. The internal carotid artery is mobilised. How many branches does this vessel give off in the neck?

	0
	1
	2
	3
	6
A

A) The internal carotid does not have any branches in the neck.

341
Q

Which of the following is a content of the adductor canal?

	Saphenous nerve
	Sural nerve
	Femoral nerve
	Profunda branch of the femoral artery
	Saphenous vein
A

A) It contains the saphenous nerve and the superficial branch of the femoral artery.

342
Q

A 56 year old man is undergoing a high anterior resection. Which of the following structures is at greatest risk of injury in this procedure?

	Superior mesenteric artery
	Left ureter
	External iliac vein
	External iliac artery
	Inferior vena cava
A

B) A careless surgeon may damage all of these structures. However, the structure at greatest risk and most frequently encountered is the left ureter.

343
Q

From which of these foraminae does the opthalmic branch of the trigeminal nerve exit the skull?

	Foramen ovale
	Foramen rotundum
	Foramen spinosum
	Superior orbital fissure
	Foramen magnum
A

D) Mnemonic:

Standing Room Only -Exit of branches of trigeminal nerve from the skull

V1 -Superior orbital fissure
V2 -foramen Rotundum
V3 -foramen Ovale

The opthalmic branch of the trigeminal nerve exits the skull through the superior orbital fissure.

344
Q

A 56 year old lady with metastatic breast cancer develops an oestolytic deposit in the proximal femur. One morning whilst getting out of bed she notices severe groin pain. X-rays show that the lesser trochanter has been avulsed. Which muscle is the most likely culprit?

	Vastus lateralis
	Psoas major
	Piriformis
	Gluteus maximus
	Gluteus medius
A

B) The psoas major inserts into the lesser trochanter and contracts when raising the trunk from the supine position. When oestolytic lesions are present in the femur the lesser trochanter may be avulsed.

345
Q

Which of the following nerves is responsible for the motor innervation of the sternocleidomastoid muscle?

	Ansa cervicalis
	Accessory nerve
	Hypoglossal nerve
	Facial nerve
	Vagus nerve
A

B) The motor supply to the sternocleidomastoid is from the accessory nerve. The ansa cervicalis supplies sensory information from the muscle.

346
Q

A 42 year old lady undergoes a difficult cholecystectomy and significant bleeding is occurring. The surgeons place a vascular clamp transversely across the anterior border of the epiploic foramen. Which of the following structures will be occluded in this manoeuvre?

	Cystic artery
	Cystic duct
	Left gastric artery
	Portal vein
	None of the above
A

D) The portal vein, hepatic artery and common bile duct are occluded.

347
Q

Which of the nerves listed below is at greatest risk of injury with a laceration to the upper lateral margin of the popliteal fossa?

	Common peroneal nerve
	Sural nerve
	Sciatic nerve
	Saphenous nerve
	Tibial nerve
A

A) The sural nerve exits at the lower infero-lateral aspect of the fossa and is more at risk in short saphenous vein surgery. The tibial nerve lies more medially and is even less likely to be injured in this location.

348
Q

Which option is false in relation to the trigeminal nerve?

The nerve originates at the pons
The posterior scalp is supplied by the trigeminal nerve
The maxillary nerve exits via the foramen rotundum
The maxillary nerve is purely sensory
The motor root is not in the trigeminal ganglion
A

B) The posterior scalp is supplied by C2-C3.

349
Q

A 45 year old man is undergoing a lymph node biopsy from the posterior triangle of his neck. Which structure forms the posterior border of this region?

	Trapezius muscle
	Diagastric muscle
	External jugular vein
	Omohyoid muscle
	Sternocleidomastoid muscle
A

A) Posterior triangle of the neck

Boundaries
Apex
Sternocleidomastoid and the Trapezius muscles at the Occipital bone

Anterior Posterior border of the Sternocleidomastoid

Posterior Anterior border of the Trapezius

Base Middle third of the clavicle

350
Q

A 42 year old lady has had an axillary node clearance for breast malignancy. Post operatively she reports weakness of the shoulder. She is unable to push herself forwards from a wall with the right arm and the scapula is pushed out medially from the chest wall. What is the most likely nerve injury?

	C5, C6
	C8, T1
	Axillary nerve
	Long thoracic nerve
	Spinal accessory nerve
A

D) The patient has a winged scapula caused by damage to the long thoracic nerve (C5,6,7) during surgery. The long thoracic nerve innervates serratus anterior. Serratus anterior causes pushing out of the scapula during a punch.

351
Q

A 36 year old male is admitted for elective surgery for a lymph node biopsy in the supraclavicular region. Post operatively the patient has difficulty shrugging his left shoulder. What is the most likely reason?

	Phrenic nerve lesion
	Axillary nerve lesion
	C5, C6 root lesion
	C8, T1 root lesion
	Accessory nerve lesion
A

E) The accessory nerve lies in the posterior triangle and may be injured in this region. Apart from problems with shrugging the shoulder, he may also have difficulty lifting his arm above his head.

352
Q

How many fissures are present within the right lung?

	One
	Three
	Two
	Four
	Five
A

C) The right lung has an oblique and horizontal fissure. The upper oblique fissure separates the inferior from the middle and upper lobes. The short horizontal fissure separates the superior and middle lobes.

353
Q

Which of the following muscles is supplied by the musculocutaneous nerve?

	Brachialis
	Latissimus dorsi
	Flexor carpi ulnaris
	Teres minor
	Triceps
A

A) Muscles innervated by the musculocutaneous nerve BBC:

Biceps brachii
Brachialis
Coracobrachialis

354
Q

Which cranial nerve supplies the motor fibres of styloglossus?

	Facial
	Trigeminal
	Vagus
	Hypoglossal
	Glossopharyngeal
A

D) The hypoglossal nerve supplies motor innervation to all extrinsic and intrinsic muscles of the tongue. The only possible exception to this is palatoglossus, which is jointly innervated by the vagus and accessory nerves.

355
Q

What is the most inferior anterior aortic branch?

	Median sacral artery
	Inferior mesenteric artery
	Lumbar artery
	Superior mesenteric artery
	Gonadal artery
A

B) The IMA leaves the front of the aorta usually about 3 to 4cm superior to its bifurcation. The median sacral is not an anterior branch.

356
Q

Which of the following statements relating to the posterior cerebral artery is false?

It supplies the visual cortex
It is closely related to the 3rd cranial nerve
It is a branch of the basilar artery
It is connected to the circle of Willis via the superior cerebellar artery
When occluded may result in contralateral loss of field of vision
A

D) The posterior cerebral arteries are formed by the bifurcation of the basilar artery and is connected to the circle of Willis via the posterior communicating artery.

The posterior cerebral arteries supply the occipital lobe and part of the temporal lobe.

357
Q

An elderly lady falls and lands on her hip. On examination, her hip is tender to palpation and x-rays are taken. There are concerns that she may have an intertrochanteric fracture. What is the normal angle between the femoral neck and the femoral shaft?

	90o
	105o
	80o
	130o
	180o
A

D) The normal angle between the femoral head and shaft is 130o. Changes to this angle may occur as a result of disease or pathology and should be investigated.

358
Q

A 22 year old man suffers a compound fracture of the tibia. During attempted surgical repair the deep peroneal nerve is divided. Which of the following muscles will not be affected as a result?

	Tibialis anterior
	Peroneus longus
	Extensor hallucis longus
	Extensor digitorum longus
	Peroneus tertius
A

B) Peroneus longus is innervated by the superficial peroneal nerve (L4, L5, S1).

359
Q

A 17 year old male presents to the clinic. He complains of difficulty using his left hand. It has been a persistent problem since he sustained a distal humerus fracture as a child. On examination, there is diminished sensation overlying the hypothenar eminence and medial one and half fingers. What is the most likely nerve lesion?

	Anterior interosseous nerve
	Posterior interosseous nerve
	Ulnar nerve
	Median nerve
	Radial nerve
A

C)

360
Q

An 18 year old athlete attends orthopaedic clinic reporting pain and swelling over the medial aspect of the knee joint. The pain occurs when climbing the stairs, but is not present when walking on flat ground. Clinically there is pain over the medial, proximal tibia and the McMurray test is negative. What is the most likely cause of this patient’s symptoms?

	Anterior cruciate ligament tear
	Prepatellar bursitis
	Medial meniscus injury
	Pes Anserinus Bursitis
	Fracture of tibia
A

D) Pes anserinus: GOOSE’S FOOT

Combination of sartorius, gracilis and semitendinous tendons inserting into the anteromedial proximal tibia.

Pes Anserinus Bursitis is common in sportsmen due to overuse injuries. The main sign is of pain in the medial proximal tibia. As the McMurray test is negative, medial meniscal injury is excluded.

361
Q

Which of the following structures lies most posteriorly at the porta hepatis?

	Cystic artery
	Common hepatic artery
	Left hepatic artery
	Portal vein
	Common bile duct
A

D) The portal vein is the most posterior structure at the porta hepatis.The common bile duct is a continuation of the common hepatic duct and is formed by the union of the common hepatic duct and the cystic duct.

362
Q

Which of the following nerves innervates the long head of the biceps femoris muscle?

	Inferior gluteal nerve
	Tibial division of sciatic nerve
	Superior gluteal nerve
	Common peroneal division of sciatic nerve
	Obturator nerve
A

B) The short head of biceps femoris, which may occasionally be absent, is innervated by the common peroneal component of the sciatic nerve. The long head is innervated by the tibial division of the sciatic nerve.

363
Q

A 28 year old man is shot in the right chest and develops a right haemothorax necessitating a thoracotomy. The surgeons decide to place a vascular clamp across the hilum of the right lung. Which of the following structures will lie most anteriorly at this point?

	Thoracic duct
	Phrenic nerve
	Vagus nerve
	Pulmonary artery
	Pulmonary vein
A

B) The phrenic nerve lies anteriorly at the root of the right lung.

364
Q

A 56 year old man is undergoing a pancreatectomy for carcinoma. During resection of the gland which of the following structures will the surgeon not encounter posterior to the pancreas itself?

	Left crus of the diaphragm
	Superior mesenteric vein
	Common bile duct
	Portal vein
	Gastroduodenal artery
A

E) The gastroduodenal artery divides into the gastro-epiploic and pancreaticoduodenal arteries at the superior aspect of the pancreas.

365
Q

From which structure is the central tendon of the diaphragm derived?

	Septum transversum
	Pleuroperitoneal folds
	Diaphragmatic crura
	Dorsal mesocardium
	Oropharyngeal membrane
A

A) The septum transversum is a thick ridge of mesodermal tissue in the developing embryo that separates the thoracic and abdominal cavities and forms the central tendon of the diaphragm.

366
Q

Where does Stensens duct primarily open?

	Immediately lateral to the foramen caecum
	Floor of mouth
	Opposite the second molar tooth
	Opposite the fifth molar tooth
	Into the post nasal space
A

C) Stensens duct conveys secretions from the parotid gland and these enter the oral cavity at the level of the second molar tooth.

367
Q

Which of the following nerves is responsible for the innervation of the posterior belly of the digastric muscle?

	Facial nerve
	Hypoglossal nerve
	Trigeminal nerve
	Ansa cervicalis
	Mylohoid nerve
A

A) The posterior belly of digastric is innervated by the facial nerve and the anterior belly by the mylohoid nerve.

368
Q

Which of the following carpal bones is a sesamoid bone in the tendon of flexor carpi ulnaris?

	Triquetrum
	Lunate
	Pisiform
	Scaphoid
	Capitate
A

C) This small bone has a single articular facet. It projects from the triquetral bone at the ulnar aspect of the wrist where most regard it as a sesamoid bone lying within the tendon of flexor carpi ulnaris.

369
Q

A 43 year old woman is due to undergo an axillary node clearance following a positive sentinel node biopsy. Which of the nerves below is at greatest risk?

	Accessory nerve
	Thoracodorsal
	Upper trunk of the brachial plexus
	Medial pectoral
	Axillary
A

B) The long thoracic nerve is also at risk. The thoracodorsal nerve traverses the level 2 axillary nodes to supply latissimus dorsi and may be divided or damaged with diathermy.

370
Q

Which of the muscles listed below would be affected if the radial nerve were injured in a humeral shaft fracture?

	Supinator
	Flexor digitorum profundus
	Brachialis
	Abductor pollicis brevis
	Abductor digiti minimi
A

A)

371
Q

A 73 year old lady is hit by a car. She suffers a complex fracture of the distal aspect of her humerus with associated injury to the radial nerve. Which of the following movements will be most impaired as a result?

	Elbow extension
	Elbow flexion
	Shoulder abduction
	Wrist extension
	None of the above
A

D) The triceps will not be affected so elbow extension will be preserved. Loss of wrist extension will be the most obvious effect.

372
Q

Which of the following is not a content of the rectus sheath?

	Pyramidalis
	Superior epigastric artery
	Inferior epigastric vein
	Internal iliac artery
	Rectus abdominis
A

D) The rectus sheath also contains:
superior epigastric vein
inferior epigastric artery

373
Q

Which of the following vessels does not drain directly into the inferior vena cava?

	Superior mesenteric vein
	Right common iliac
	Right hepatic vein
	Left hepatic vein
	Right testicular vein
A

A) The superior mesenteric vein drains into the portal vein. The right and left hepatic veins drain into it directly, this can account for major bleeding in more extensive liver shearing type injuries.

374
Q

A 17 year old male has a suspected testicular torsion and the scrotum is to be explored surgically. The surgeon incises the skin and then the dartos muscle. What is the next tissue layer that will be encountered during the dissection?

	Visceral layer of the tunica vaginalis
	Cremasteric fascia
	Parietal layer of the tunica vaginalis
	External spermatic fascia
	Internal spermatic fascia
A

D) The layers that will be encountered are (in order):

  1. Skin
  2. Dartos fascia and muscle
  3. External spermatic fascia
  4. Cremasteric muscle and fascia
  5. Internal spermatic fascia
  6. Parietal layer of the tunica vaginalis
375
Q

A well toned weight lifter attends clinic reporting weakness of his left arm. There is weakness of flexion and supination of the forearm. Which of the following nerves is likely to have been affected?

	Axillary
	Suprascapular
	Medial
	Radial
	Musculocutaneous
A

E) Musculocutaneous nerve compression due to entrapment of the nerve between biceps and brachialis. Elbow flexion and supination of the arm are affected. This is a rare isolated injury.

376
Q

A 25 year old man is stabbed in the groin and the area, which lies within the femoral triangle is explored. Which structure forms the lateral wall of the femoral triangle?

	Adductor longus
	Pectineus
	Adductor magnus
	Sartorius
	Conjoint tendon
A

D) The sartorius forms the lateral wall of the femoral triangle

377
Q

An 18 year old man develops a severe spreading sepsis of the hand. The palm is explored surgically and the flexor digiti minimi brevis muscle is mobilised to facilitate drainage of the infection. Which of the following structures is not closely related to this muscle?

	The hook of hamate
	Median nerve
	Superficial palmar arterial arch
	Digital nerves arising from the ulnar nerve
	None of the above
A

B) The flexor digiti minimi brevis originates from the Hamate, on its under- surface lie the ulnar contribution to the superficial palmar arterial arch and digital nerves derived from the ulnar nerve. The median nerve overlies the flexor tendons.

378
Q

A 19 year old man undergoes an open inguinal hernia repair. The cord is mobilised and the deep inguinal ring identified. Which of the following structures forms its lateral wall?

	External oblique aponeurosis
	Transversalis fascia
	Conjoint tendon
	Inferior epigastric artery
	Inferior epigastric vein
A

B) The transversalis fascia forms the superolateral edge of the deep inguinal ring. The epigastric vessels form its inferomedial wall.

379
Q

A 22 year old man develops an infection in the pulp of his little finger. What is the most proximal site to which this infection may migrate?

	The metacarpophalangeal joint
	The distal interphalangeal joint
	The proximal interphalangeal joint
	Proximal to the flexor retinaculum
	Immediately distal to the carpal tunnel
A

D) The 5th tendon sheath extends from the little finger to the proximal aspect of the carpal tunnel. This carries a significant risk of allowing infections to migrate proximally.

380
Q

Which of the following muscles is not innervated by the deep branch of the ulnar nerve?

	Adductor pollicis
	Hypothenar muscles
	All the interosseous muscles
	Opponens pollicis
	Third and fourth lumbricals
A

D)

381
Q

Which of the following muscles is not innervated by the deep branch of the ulnar nerve?

	Adductor pollicis
	Hypothenar muscles
	All the interosseous muscles
	Opponens pollicis
	Third and fourth lumbricals
A

D)

382
Q

During an inguinal hernia repair the surgeon identifies a small nerve whilst mobilising the cord structures at the level of the superficial inguinal ring. Which nerve is this most likely to be?

	Subcostal
	Iliohypogastric
	Ilioinguinal
	Obturator
	Pudendal
A

C) Ilioinguinal nerve entrapment may be a cause of neuropathic pain following inguinal hernia surgery.

383
Q

A 25 year old man undergoes an excision of a pelvic chondrosarcoma, during the operation the obturator nerve is sacrificed. Which of the following muscles is least likely to be affected as a result?

	Adductor longus
	Pectineus
	Adductor magnus
	Sartorius
	Gracilis
A

D) Sartorius is supplied by the femoral nerve. In approximately 20% of the population, pectineus is supplied by the accessory obturator nerve. Adductor magnus has a composite supply from the obturator and sciatic nerves.

384
Q

During an arch aortogram the brachiocephalic artery is entered with an angiography catheter. The radiologist continues to advance the catheter. Into which of the following vessels is it likely to enter?

	Left subclavian artery
	Left axillary artery
	Right subclavian artery
	Right axillary artery
	None of the above
A

C) The axillary artery is a branch of the subclavian artery and although developmental anomalies may occur they are rare. The catheter may also enter the right carotid. There is no brachiocephalic artery on the left side.

385
Q

Into which of the following structures does the superior part of the fibrous capsule of the shoulder joint insert?

	The surgical neck of the humerus
	The body of the humerus
	The bicipital groove
	Immediately distal to the greater tuberosity
	The anatomical neck of the humerus
A

E) The shoulder joint is a shallow joint, hence its great mobility. However, this comes at the expense of stability. The fibrous capsule attaches to the anatomical neck superiorly and the surgical neck inferiorly

386
Q

What is the lymphatic drainage of the membranous urethra?

	Deep inguinal nodes
	Superficial inguinal nodes
	Internal iliac nodes
	External iliac nodes
	Para-aortic nodes
A

C) The prostatic and membranous urethra drain to the internal iliac nodes.

387
Q

A 34 year old lady presents with symptoms of faecal incontinence. Ten years previously she gave birth to a child by normal vaginal delivery. Injury to which of the following nerves is most likely to account for this process?

	Genitofemoral
	Ilioinguinal
	Pudendal
	Hypogastric autonomic nerve
	Obturator
A

C) S2,3,4 keeps the poo up off the floor - POOdendal nerve

Damage to the pudendal nerve is classically associated with faecal incontinence and it is for this reason that sacral neuromodulation is a popular treatment for the condition. Injury to the hypogastric autonomic nerves is an aetiological factor in the development of constipation.

388
Q

During a difficult thyroidectomy haemorrhage is noted from the thyroidea ima vessel. From which structure does this vessel usually arise?

	External carotid artery
	Internal carotid artery
	Brachiocephalic artery
	Axillary artery
	Superior thyroid artery
A

C) Rhyme isthmus location:

Rings 2,3,4 make the isthmus floor

This accessory vessel which usually lies at the inferior aspect of the gland is derived either from the brachiocephalic artery or the arch of the aorta.

389
Q

The cephalic vein pierces the clavipectoral fascia to terminate in which of the veins listed below?

	External jugular
	Axillary
	Internal jugular
	Azygos
	Brachial
A

B)

390
Q

A 43 year old lady develops a cerebello-pontine angle lesion. Which of the nerves listed below is likely to be affected first?

	CN X
	CN III
	CN V
	CN IX
	CN XII
A

C) The most likely lesion to occur in the cerebello-pontine angle is an acoustic neuroma.
The trigeminal nerve has a broad base and involvement of at least part of this nerve is the most likely initial finding. The defect may be subtle such as loss of the ipsilateral corneal reflex. Ipsilateral hearing loss will also occur. Untreated, progressive lesions, may ultimately affect cranial nerve roots in this region.

391
Q

Which of the following is not a branch of the abdominal aorta?

	Inferior mesenteric artery
	Inferior phrenic artery
	Superior mesenteric artery
	Superior phrenic artery
	Renal artery
A

D) Mnemonic for the Descending abdominal aorta branches from diaphragm to iliacs:

‘Prostitutes Cause Sagging Swollen Red Testicles [in men] Living In Sin’:

Phrenic [inferior]
Celiac
Superior mesenteric
Suprarenal [middle]
Renal
Testicular ['in men' only]
Lumbars
Inferior mesenteric
Sacral

The superior phrenic artery branches from the aorta in the thorax.

392
Q

A 23 year old man is admitted with a suspected ureteric colic. A KUB style x-ray is obtained. In which of the following locations is the stone most likely to be visualised?

The tips of the transverse processes between L2 and L5
The tips of transverse processes between T10-L1
At the crest of the ilium
Over the S3 foramina
Over the sacrococcygeal joint
A

A) The ureter lies anterior to L2 to L5 and stones may be visualised at these points, they may also be identified over the sacro-iliac joints.

393
Q

In a patient with an ectopic kidney where is the adrenal gland most likely to be located?

	In the pelvis
	On the contralateral side
	In its usual position
	Superior to the spleen
	It will be absent
A

C) Because the kidney is present, rather than absent, the adrenal will usual develop and in the normal location.

394
Q

Which of the following structures is not closely related to the posterior tibial artery?

	Soleus posteriorly
	Tibial nerve medially
	Deep peroneal nerve laterally
	Flexor hallucis longus postero-inferiorly
	Popliteus
A

C) The deep peroneal nerve lies in the anterior compartment. The tibial nerve lies medially. At its termination it lies deep to the flexor retinaculum.

395
Q

Which cranial nerve provides general sensation to the anterior two thirds of the tongue?

	Facial
	Trigeminal
	Hypoglossal
	Vagus
	Glossopharyngeal
A

B) Taste to the anterior two thirds of the tongue is supplied by the facial nerve, the trigeminal supplies general sensation, this is mediated by the mandibular branch of the trigeminal nerve (via the lingual nerve).

396
Q

A 62 year old male complains of back pain. He has had a recent fall. Walking causes pain of the left lower leg. On examination, he is noted to have reduced sensation over the knee. Which of the spinal levels listed below is most likely to have been affected?

	L1
	L3
	L2
	L4
	L5
A

B) Sensation over the knee is equivalent to the L3 dermatome. The four nerves involved include the infrapatellar branch of the saphenous nerve, the lateral cutaneous nerve of the thigh, anterior cutaneous nerve of the thigh (both lateral and medial branches).

397
Q

Which of the following statements relating to the tympanic membrane is false?

The umbo marks the point of attachment of the handle of the malleus to the tympanic membrane
The lateral aspect of the tympanic membrane is lined by stratified squamous epithelium
The chorda tympani nerve runs medial to the pars tensa
The medial aspect of the tympanic membrane is lined by mucous membrane
The tympanic membrane is approximately 1cm in diameter
A

C) The chorda tympani runs medially to the pars flaccida.

398
Q

Which nerve supplies sensation to the nail bed of the index finger?

	Median
	Ulnar
	Radial
	Musculocutaneous
	Axillary
A

A)

399
Q

An injury to the spinal accessory nerve will have the greatest affect on which of the following movements?

	Lateral rotation of the arm
	Adduction of the arm at the glenohumeral joint
	Protraction of the scapula
	Upward rotation of the scapula
	Depression of the scapula
A

D) The spinal accessory nerve innervates trapezius. The entire muscle will retract the scapula. However, its upper and lower fibres act together to upwardly rotate it.

400
Q

Which of the following is not contained within the middle mediastinum?

	Main bronchi
	Arch of the azygos vein
	Thoracic duct
	Pericardium
	Aortic root
A

C) The thoracic duct lies within the posterior and superior mediastinum.

401
Q

55 year old man is due to undergo a radical prostatectomy for carcinoma of the prostate gland. Which of the following vessels directly supplies the prostate?

	External iliac artery
	Common iliac artery
	Internal iliac artery
	Inferior vesical artery
	None of the above
A

D) The arterial supply to the prostate gland is from the inferior vesical artery, it is a branch of the prostatovesical artery. The prostatovesical artery usually arises from the internal pudendal and inferior gluteal arterial branches of the internal iliac artery.

402
Q

Which nerve directly innervates the sinoatrial node?

	Superior cardiac nerve
	Right vagus nerve
	Left vagus nerve
	Inferior cardiac nerve
	None of the above
A

E) No single one of the above nerves is responsible for direct cardiac innervation (which those who have handled the heart surgically will appreciate).
The heart receives its nerves from the superficial and deep cardiac plexuses. The cardiac plexuses send small branches to the heart along the major vessels, continuing with the right and left coronary arteries. The vagal efferent fibres emerge from the brainstem in the roots of the vagus and accessory nerves, and run to ganglia in the cardiac plexuses and within the heart itself.

The background vagal discharge serves to limit heart rate, and loss of this background vagal tone accounts for the higher resting heart rate seen following cardiac transplant.

403
Q

A 56 year old man suddenly develops severe back pain. His pain has a radicular pattern. On examination, he is unable to extend his great toe. Which of the spinal levels listed below is most likely to have been affected?

	L5
	L4
	L3
	L2
	L1
A

A) Extensor hallucis longus is derived from L5 and loss of EHL function is a useful test to determine whether this level is involved.

404
Q

A 30 year old man presents with back pain and the surgeon tests the ankle reflex. Which of the following nerve roots are tested in this manoeuvre?

	S3 and S4
	L4 and L5
	L3 and L4
	S1 and S2
	S4 only
A

D) Ankle reflex

The ankle reflex is elicited by tapping the Achilles tendon with a tendon hammer. It tests the S1 and S2 nerve roots. It is typically delayed in L5 and S1 disk prolapses.

405
Q

Which of the following structures accompanies the posterior interventricular artery within the posterior interventricular groove?

	Great cardiac vein
	Middle cardiac vein
	Small cardiac vein
	Anterior cardiac vein
	Coronary sinus
A

B) Coronary sinus
This lies in the posterior part of the coronary groove and receives blood from the cardiac veins. The great cardiac vein lies at its left and the middle and small cardiac veins lie on its right. The smallest cardiac vein (anterior cardiac vein) drains into the right atrium directly.

406
Q

An 18 year old male presents to casualty with a depressed skull fracture. This is managed surgically. Over the next few days he complains of double vision on walking down stairs and reading. On testing ocular convergence, the left eye faces downwards and medially, but the right side does not do so. Which of the nerves listed below is most likely to be responsible?

	Facial
	Oculomotor
	Abducens
	Trochlear
	Trigeminal nerve
A

D) The trochlear nerve has a relatively long intracranial course and this makes it vulnerable to injury in head trauma. Head trauma is the commonest cause of an acute fourth nerve palsy. A 4th nerve palsy is the commonest cause of a vertical diplopia. The diplopia is at its worst when the eye looks medially which it usually does as part of the accommodation reflex when walking down stairs.

407
Q

A 77 year old man with symptoms of intermittent claudication is due to have his ankle brachial pressure indices measured. The dorsalis pedis artery is impalpable. Which of the following tendinous structures lies medial to it, that may facilitate its identification?

	Extensor digitorum longus tendon
	Peroneus tertius tendon
	Extensor hallucis longus tendon
	Extensor digitorum brevis tendon
	Flexor digitorum longus
A

C) The extensor hallucis longus tendon lies medial to the dorsalis pedis artery.

408
Q

A 28 year teacher reports difficulty with writing. There is no sensory loss. She is known to have an aberrant Gantzer muscle. Which of the following nerves has been affected?

	Posterior interosseous
	Anterior interosseous
	Median
	Ulnar
	Musculocutaneous
A

B) Anterior interosseous lesions occur due to fracture, or rarely due to compression. The Gantzer muscle is an aberrant accessory of the flexor pollicis longus and is a risk factor for anterior interosseous nerve compression. Remember loss of pincer grip and normal sensation indicates an interosseous nerve lesion.

409
Q

What is the level of the hyoid bone?

	C1
	C2
	C3
	C4
	C5
A

C)

410
Q

A 23 year old man falls over whilst intoxicated and a shard of glass transects his median nerve at the proximal border of the flexor retinaculum. His tendons escape injury. Which of the following features is least likely to be present?

Weakness of thumb abduction
Loss of sensation on the dorsal aspect of the thenar eminence
Loss of power of opponens pollicis
Adduction and lateral rotation of the thumb at rest
Loss of power of abductor pollicis brevia
A

B) The median nerve may be injured proximal to the flexor retinaculum. This will result in loss of abductor pollicis brevis, flexor pollicis brevis, opponens pollicis and the first and second lumbricals. When the patient is asked to close the hand slowly there is a lag of the index and middle fingers reflecting the impaired lumbrical muscle function. The sensory changes are minor and do not extend to the dorsal aspect of the thenar eminence.
Abductor pollicis longus will contribute to thumb abduction (and is innervated by the posterior interosseous nerve) and therefore abduction will be weaker than prior to the injury.

411
Q

The following muscles are supplied by the recurrent laryngeal nerve except:

	Transverse arytenoid
	Posterior crico-arytenoid
	Cricothyroid
	Oblique arytenoid
	Thyroarytenoid
A

C) Innervates: all intrinsic larynx muscles (excluding cricothyroid)

The external branch of the superior laryngeal nerve innervates the cricothyroid muscle.

412
Q

Which of the following relationship descriptions regarding the scalene muscles is incorrect?

The brachial plexus passes anterior to the middle scalene muscle
The phrenic nerve lies anterior to the anterior scalene muscle
The subclavian artery passes posterior to the middle scalene
The subclavian vein lies anterior to the anterior scalene muscle at the level of the first rib
The anterior scalene inserts into the first rib
A

C) The subclavian artery passes anterior to the middle scalene.

413
Q

A 23 year old man falls and injures his hand. There are concerns that he may have a scaphoid fracture as there is tenderness in his anatomical snuffbox on clinical examination. Which of the following forms the posterior border of this structure?

	Basilic vein
	Radial artery
	Extensor pollicis brevis
	Abductor pollicis longus
	Extensor pollicis longus
A

A) It’s boundaries are extensor pollicis longus, medially (posterior border) and laterally (anterior border) by the tendons of abductor pollicis longus and extensor pollicis brevis.

Anatomical snuffbox

Posterior border Tendon of extensor pollicis longus
Anterior border Tendons of extensor pollicis brevis and abductor pollicis longus
Proximal border Styloid process of the radius
Distal border Apex of snuffbox triangle
Floor Trapezium and scaphoid
Content Radial artery

414
Q

Which of the following structures attaches periosteum to bone?

	Sharpeys fibres
	Peripheral lamellae
	Elastic fibres
	Fibrolamellar bundles
	Purkinje fibres
A

A) Periosteum is attached to bone by strong collagenous fibers called Sharpey’s fibres, which extend to the outer circumferential and interstitial lamellae. It also provides an attachment for muscles and tendons.

415
Q

A 22 year man is shot in the groin. On examination, he has weak hip flexion, weak knee extension, and impaired quadriceps tendon reflex, as well as sensory deficit in the anteromedial aspect of the thigh. Which structure has been affected?

	Femoral nerve
	Sciatic nerve
	Superior gluteal nerve
	Ilioinguinal nerve
	Genitofemoral nerve
A

A)

416
Q

A man is undergoing excision of a sub mandibular gland. As the gland is mobilised, a vessel is injured lying between the gland and the mandible. Which of the following is this vessel most likely to be?

	Lingual artery
	Occipital artery
	Superior thyroid artery
	Facial artery
	External jugular vein
A

D) The high salivary viscosity of submandibular gland secretions favors stone formation.
Most stones are radio-opaque.
The marginal mandibular nerve is the most superficial structure.

The facial artery lies between the gland and mandible and is often ligated during excision of the gland. The lingual artery may be encountered but this is usually later in the operative process as Whartons duct is mobilised.

417
Q

A 52 year old female renal patient needs a femoral catheter to allow for haemodialysis. Which of the structures listed below is least likely to be encountered during its insertion?

	Great saphenous vein
	Deep circumflex iliac artery
	Superficial circumflex iliac artery
	Femoral vein
	Femoral branch of the genitofemoral nerve
A

B) Femoral access catheters are typically inserted in the region of the femoral triangle. Therefore the physician may encounter the femoral, vein, nerve, branches of the femoral artery and tributaries of the femoral vein. The deep circumflex iliac artery arises above the inguinal ligament and is therefore less likely to be encountered than the superficial circumflex iliac artery which arises below the inguinal ligament.

418
Q

A 44 year old lady who works as an interior decorator has undergone a mastectomy and axillary node clearance to treat breast cancer. Post operatively, she comments that her arm easily becomes fatigued when she is painting walls. What is the most likely explanation?

	Injury to the axillary nerve
	Injury to the long thoracic nerve
	Injury to the intercostobrachial nerve
	Injury to the thoracodorsal nerve
	Injury to the median pectoral nerve
A

D) The most likely explanation for this is that the thoracodorsal nerve has been injured. This will result in atrophy of latissimus dorsi and this will become evident with repetitive arm movements where the arm is elevated and moving up and down (such as in painting). Injury to the pectoral nerves may produce a similar picture but this pattern of injury is very rare and the pectoral nerves are seldom injured in breast surgery.

419
Q

A 53 year old man with a chronically infected right kidney is due to undergo a nephrectomy. Which of the following structures would be encountered first during a posterior approach to the hilum of the right kidney?

	Right renal artery
	Ureter
	Right renal vein
	Inferior vena cava
	Right testicular vein
A

B) The ureter is the most posterior structure at the hilum of the right kidney and would therefore be encountered first during a posterior approach.

Renal arteries

The right renal artery is longer than the left renal artery
The renal vein/artery/pelvis enter the kidney at the hilum

Relations
Right Anterior- IVC, right renal vein, the head of the pancreas, and the descending part of the duodenum
Left Anterior- left renal vein, the tail of the pancreas

420
Q

A 28 year old man is stabbed outside a nightclub in the upper arm. The median nerve is transected. Which of the following muscles will demonstrate impaired function as a result?

	Palmaris brevis
	Second and third interossei
	Adductor pollicis
	Abductor pollicis longus
	Abductor pollicis brevis
A
E) 
Palmaris brevis - Ulnar nerve
Palmar interossei- Ulnar nerve
Adductor pollicis - Ulnar nerve
Abductor pollicis longus - Posterior interosseous nerve
Abductor pollicis brevis - Median nerve

The median nerve innervates all the short muscles of the thumb except the adductor and the deep head of the short flexor. Palmaris and the interossei are innervated by the ulnar nerve.

421
Q

A 22 year old man sustains a blow to the side of his head with a baseball bat during a fight. He is initially conscious. However, he subsequently loses consciousness and then dies. Post mortem examination shows an extradural haematoma. The most likely culprit vessel is a branch of which of the following?

	Middle cerebral artery
	Internal carotid artery
	Anterior cerebral artery
	Maxillary artery
	Mandibular artery
A

D) The middle meningeal artery is the most likely source of the extradural haematoma in this setting. It is a branch of the maxillary artery. The middle cerebral artery does not give rise to the middle meningeal artery. Note that the question is asking for the vessel which gives rise to the middle meningeal artery (“the likely culprit vessel is a branch of which of the following”)

422
Q

A 72 year old man with carcinoma of the lung is undergoing a left pneumonectomy. The left main bronchus is divided. Which of the following thoracic vertebrae lies posterior to this structure?

	T3
	T7
	T6
	T10
	T1
A

C) The left main bronchus lies at T6. Topographical anatomy of the thorax is important as it helps surgeons to predict the likely structures to be injured in trauma scenarios (so popular with examiners)

423
Q

Which of the following regions of the male urethra is entirely surrounded by Bucks fascia?

	Preprostatic part
	Prostatic part
	Membranous part
	Spongiose part
	None of the above
A

D) Bucks fascia is a layer of deep fascia that covers the penis it is continuous with the external spermatic fascia and the penile suspensory ligament. The membranous part of the urethra may partially pass through Bucks fascia as it passes into the penis. However, the spongiose part of the urethra is contained wholly within Bucks fascia.

424
Q

In the distal third of the upper arm, where is the musculocutaneous nerve located?

Between the biceps brachii and brachialis muscles
Between the brachialis and brachioradialis muscles
Between the brachioradialis and triceps muscles
Between the brachialis and triceps muscles
Between the humerus and brachialis muscles
A

A) The musculocutaneous nerve lies between the biceps and brachialis muscles.

425
Q

A 48 year old lady is undergoing a left sided adrenalectomy for an adrenal adenoma. The superior adrenal artery is injured and starts to bleed, from which of the following does this vessel arise?

	Left renal artery
	Inferior phrenic artery
	Aorta
	Splenic
	None of the above
A

B) The superior adrenal artery is a branch of the inferior phrenic artery.

426
Q

Which of the following does not exit the pelvis through the greater sciatic foramen?

	Superior gluteal artery
	Internal pudendal vessels
	Sciatic nerve
	Obturator nerve
	Inferior gluteal nerve
A

D) The obturator nerve exits through the obturator foramen.

427
Q

Which of the following is not a branch of the posterior cord of the brachial plexus?

	Thoracodorsal nerve
	Axillary nerve
	Radial nerve
	Lower subscapular nerve
	Musculocutaneous nerve
A

E) Mnemonic branches off the posterior cord

S ubscapular (upper and lower)
T horacodorsal
A xillary
R adial

The musculocutaneous nerve is a branch off the lateral cord.

428
Q

A 18 year old man presents with an indirect inguinal hernia and undergoes surgery. The deep inguinal ring is exposed and held with a retractor at its medial aspect. Which structure is most likely to lie under the retractor?

	Ureter
	Inferior epigastric artery
	Internal iliac vein
	Femoral artery
	Lateral border of rectus abdominis
A

B) Boundaries of the deep inguinal ring:
Superolaterally - transversalis fascia
Inferomedially - inferior epigastric artery

The deep inguinal ring is closely related to the inferior epigastric artery. The inferior epigastric artery forms part of the structure referred to as Hesselbach’s triangle.

429
Q

A 73 year old man presents with a tumour at the tip of his tongue. To which of the following regions will the tumour initially metastasise?

	Sub mental nodes
	Ipsilateral deep cervical nodes
	Tonsil
	Ipsilateral superficial cervical nodes
	Contralateral deep cervical nodes
A

A)

430
Q

56 year old machinist has his arm entrapped in a steel grinder and is brought to the emergency department. On examination, he is unable to extend his metacarpophalangeal joints and abduct his shoulder. He has weakness of his elbow and wrist. What has been injured?

	Ulnar nerve
	Axillary nerve
	Medial cord of brachial plexus
	Lateral cord of brachial plexus
	Posterior cord of brachial plexus
A

E) The posterior cord gives rise to:
Radial nerve ((innervates the triceps, brachioradialis, wrist extensors, and finger extensors)
Axillary nerve (innervates deltoid and teres minor)
Upper subscapular nerve (innervates subscapularis)
Lower subscapular nerve (innervates teres major and subscapularis)
Thoracodorsal nerve (innervates latissimus dorsi)

This is a description of a posterior cord lesion. Remember that the posterior cord gives rise to the axillary and radial nerve.

431
Q

Which of the muscles below does not cause lateral rotation of the hip?

	Obturator internus
	Quadratus femoris
	Gemellus inferior
	Piriformis
	Pectineus
A

E) Mnemonic lateral hip rotators: P-GO-GO-Q (top to bottom)

Piriformis
Gemellus superior
Obturator internus
Gemellus inferior
Obturator externus
Quadratus femoris

Pectineus adducts and medially rotates the femur.

432
Q

Which of the following is not a content of the posterior triangle of the neck?

	Spinal accessory nerve
	Phrenic nerve
	External jugular vein
	Occipital lymph nodes
	Internal jugular vein
A

E)

433
Q

Which of these openings transmits the facial nerve into the temporal bone?

	Internal acoustic meatus
	Foramen lacerum
	Foramen spinosum
	Stylomastoid foramen
	Jugular foramen
A

A) It enters the temporal bone through the internal acoustic meatus and exits through the stylomastoid foramen.

434
Q

A motor cyclist is involved in a road traffic accident causing severe right shoulder injuries. He is found to have an adducted, medially rotated shoulder. The elbow is fully extended and the forearm pronated. Which is the most likely diagnosis?

	C8, T1 root lesion
	C5, C6 root lesion
	Radial nerve lesion
	Ulnar nerve lesion
	Axillary nerve lesion
A

B) Erbs Palsy C5, C6 lesion
The features include:
Waiter’s tip position
Loss of shoulder abduction (deltoid and supraspinatus paralysis)
Loss of external rotation of the shoulder (paralysis of infraspinatus)
Loss of elbow flexion (paralysis of biceps, brachialis and brachioradialis)
Loss of forearm supination (paralysis of Biceps)

The motorcyclist has had an Erb’s palsy (C5, C6 root lesion). This is commonly known to be associated with birth injury when a baby has a shoulder dystocia.

435
Q

A patient is due to undergo a right hemicolectomy for a carcinoma of the caecum. Which of the following vessels will require high ligation to provide optimal oncological control?

	Middle colic artery
	Inferior mesenteric artery
	Superior mesenteric artery
	Ileo-colic artery
	None of the above
A

D) The ileo - colic artery supplies the caecum and would require high ligation during a right hemicolectomy. The middle colic artery should generally be preserved when resecting a caecal lesion.
This question is essentially asking you to name the vessel supplying the caecum. The SMA does not directly supply the caecum, it is the ileocolic artery which does this.

436
Q

A 40-year-old man presents with pain in his lower back and ‘sciatica’ for the past three days. He describes bending down to pick up a washing machine when he felt ‘something go’. He now has severe pain radiating from his back down the right leg. On examination he describes paraesthesia over the anterior aspect of the right knee and the medial aspect of his calf. Power is intact and the right knee reflex is diminished. The femoral stretch test is positive on the right side. Which nerve or nerve root is most likely to be affected?

	Common peroneal nerve
	Lateral cutaneous nerve of the thigh
	L5
	L3
	L1
A

D)

437
Q

Which of the following statements relating to sartorius is untrue?

It is supplied by the femoral nerve
It forms the lateral boundary of the femoral triangle
The middle third forms the roof of the adductor canal
It is a flexor of the hip and knee
It inserts into the medial femoral condyle
A

E) It inserts into the medial aspect of the upper part of the tibia.

438
Q

A 63 year old man undergoes a radical cystectomy for carcinoma of the bladder. During the procedure there is considerable venous bleeding. What is the primary site of venous drainage of the urinary bladder?

	Vesicoprostatic venous plexus
	Internal iliac vein
	External iliac vein
	Gonadal vein
	Common iliac vein
A

A) The urinary bladder has a rich venous plexus surrounding it, this drains subsequently into the internal iliac vein. The vesicoprostatic plexus may be a site of considerable venous bleeding during cystectomy.

439
Q

A man sustains a laceration between the base of the little finger and wrist. Several weeks after the injury there is loss of thumb adduction power. Which nerve is most likely to have been injured?

	Superficial ulnar nerve
	Deep ulnar nerve
	Median nerve
	Radial nerve
	Recurrent branch of median nerve
A

B)

440
Q

A 60 year old female is undergoing a Whipples procedure for adenocarcinoma of the pancreas. As the surgeons begin to mobilise the pancreatic head they identify a large vessel passing inferiorly over the anterior aspect of the uncinate process. What is it likely to be?

	Superior mesenteric artery
	Coeliac axis
	Inferior mesenteric artery
	Aorta
	Left gastric artery
A

A) The superior mesenteric artery arises from the aorta and passes anterior to the lower part of the pancreas. Invasion of this structure is a relative contra indication to resectional surgery.
K

441
Q

A 23 year old man has a cannula inserted into his cephalic vein. Through which structure does the cephalic vein pass?

	Interosseous membrane
	Triceps
	Pectoralis major
	Clavipectoral fascia
	Tendon of biceps
A

D) The cephalic vein is a favored vessel for arteriovenous fistula formation and should be preserved in patients with end stage renal failure

The cephalic vein penetrates the calvipectoral fascia (but not the pectoralis major) prior to terminating in the axillary vein.

442
Q

An 18 year old man is undergoing an orchidectomy via a scrotal approach. The surgeons mobilise the spermatic cord. From which of the following is the outermost layer of this structure derived?

	Internal oblique aponeurosis
	External oblique aponeurosis
	Transversalis fascia
	Rectus sheath
	Campers fascia
A

B) The outermost covering of the spermatic cord is derived from the external oblique aponeurosis.This layer is added as the cord passes through the superficial inguinal ring.

443
Q

A 53 year old male presents with a carcinoma of the transverse colon. Which of the following structures should be ligated close to their origin to maximise clearance of the tumour?

	Superior mesenteric artery
	Inferior mesenteric artery
	Middle colic artery
	Ileo-colic artery
	Superior rectal artery
A

C) The middle colic artery supplies the transverse colon and requires high ligation during cancer resections. It is a branch of the superior mesenteric artery.
P